Sei sulla pagina 1di 253

2008 Oral and Maxillofacial Surgery

Self Assessment Tool (OMSSAT)

Administration Dates: April 12th - 26th


2008 Oral and Maxillofacial Surgery Self Assessment Tool (OMSSAT)

Which drug or drug class should be avoided if possible in patients with hypertrophic
cardiomyopathy?

A. - blockers

B. Angiotensin converting enzyme inhibitors

C. Calcium channel blocker

D. Diuretics

Answer: D

Rationale:
Dehydration in patients with this condition acts to increase the outflow tract pressure
gradients from the heart and generate an increase in symptoms. This can be exacerbated as
well with strenuous activity and result in sudden death. Dehydration and the use of
diuretics should be avoided if possible so as not to alter this gradient. Digitalis, nitrates,
vasodilators and adrenergic agonists are also to be avoided.

Beta-blockers are the mainstay of medical therapy for hypertrophic cardiomyopathy.


Angina, dyspnea, and presyncope may all be improved with beta-blockers. Calcium
channel blockers are an alternative therapy to beta blockers. ACE inhibitors are not
typically used in the treatment of hypertrophic cardiomyopathy.

Reference:
Kasper, D Braunwald, E et al Harrison's Principles of Internal Medicine 16th ed., McGraw-
Hill, New York, 2005 p1411.

Zipes: Braunwald's Heart Disease: A Textbook of Cardiovascular Medicine, 7th ed.,


2005, Elsevier, Chapter 59, Hypertrophic Cardiomyopathy.

American Board of Oral and Maxillofacial Surgery 2


2008 Oral and Maxillofacial Surgery Self Assessment Tool (OMSSAT)

Which type of cardiomyopathy is most often associated with high outflow tract pressures?

A. Restrictive

B. Dilated

C. Idiopathic

D. Hypertrophic

Answer: D

Rationale:
Patients with hypertrophic cardiomyopathy are most frequently associated with having
increased left ventricular outflow tract obstruction. This occurs in approximately 25% of
these patients. It is usually related to narrowing of the subaortic area as sequelae of the
apposition of the mitral valve leaflet in juxtaposition to the enlarged interventricular
septum. Hypertrophic cardiomyopathy is an autosomal dominant inherited disease at least
50% of the time. There are sporadic forms of the disease due to spontaneous mutations.

Dilated cardiomyopathy is characterized by cardiac enlargement and impaired systolic


function of one of both ventricles. Restrictive cardiomyopathy is the least common form in
the western hemisphere. The hallmark feature is abnormal diastolic function. The
ventricular walls are excessively rigid and impede ventricular filling. Systolic function is
often not impaired. Both dilated and restrictive cardiomyopathy can be idiopathic in
nature.

Reference:
Kasper, D Braunwald,E et al Harrison's Principles of Internal Medicine 16th ed., McGraw-
Hill, New York, 2005 pp1409-1412.

Zipes: Braunwald's Heart Disease: A Textbook of Cardiovascular Medicine, 7th ed, 2005,
Elsevier.

American Board of Oral and Maxillofacial Surgery 3


2008 Oral and Maxillofacial Surgery Self Assessment Tool (OMSSAT)

Caution should be given to administering local anesthetics containing vasoconstrictor to


hypertensive patients treated with which medication?

A. ACE inhibitors

B. Non-selective beta blockers

C. Calcium channel blockers

D. Angiotensin receptor blockers

Answer: B

Rationale:
Treatment of hypertension with a non-selective beta blocker will affect both beta-1 and
beta-2 receptors. Epinephrine or levonordefrin will normally cause sympathetic
stimulation of both alpha and beta adrenergic receptors. Alpha-1 mediated
vasoconstriction is unopposed with beta blockade and can result in severe hypertension and
possible reflex bradycardia.

ACE inhibitors affect the renin-angiotensin pathway leading to decreased angiotensin II


production. Angiotension II is a potent vasoconstrictor and therefore this reduces peripheral
vasoconstriction and afterload and will decrease blood pressure. Ca channel blockers will
lead to a decrease in the vasoconstriction of peripheral vasculature leading to a decrease in
BP.

Reference:
Ganzberg, Local Anesthetics and Vasoconstrictors. Oral and Maxillofacial Surgery Clinics of
North America, Volume 13, #1, p. 71, 2001.

American Board of Oral and Maxillofacial Surgery 4


2008 Oral and Maxillofacial Surgery Self Assessment Tool (OMSSAT)

The agent with the slowest onset of action when treating an acute.hypertensive crisis is:

A. esmolol (Brevibloc).

B. labetolol (Normodyne).

C. sublingual nitroglycerin (Nitrostat).

D. hydralazine (Apresoline).

Answer: D

Rationale:
Esmolol is a selective beta blocking agent against B1 receptors. The onset of action for IV
esmolol is 2-10 minutes. Labetalol has an onset of 2-5 minutes. It is a B1 and B2
blocker, and also blocks alpha receptors. Onset for sublingual nitroglycerin is 2-5
minutes. Nitroglycerin causes vasodilation and arterial dilation. Hydralazine has an onset
of 5-20 minutes. Hydralazine is a direct peripheral dilator, with prolonged duration of
action.

Reference:
Dym, The Hypertensive Patient. Oral and Maxillofacial Surgery Clinics of North America,
Volume 10, #3, p. 358, 1998.

Stoelting, Dierdorf, Anesthesia and Co-Existing Disease, 4th edition, p. 98, 2002.

American Board of Oral and Maxillofacial Surgery 5


2008 Oral and Maxillofacial Surgery Self Assessment Tool (OMSSAT)

A 56 year-old white male has been diagnosed with secondary hypertension associated with
hyperaldosteronism. The laboratory finding consistent with this diagnosis is:

A. hyperkalemia.

B. hypokalemia.

C. hypercalcemia.

D. hypocalcemia.

Answer: B

Rationale:
An adrenal adenoma or hyperplastic adrenal gland increases secretion of aldosterone from
the zona glomerulosa (outermost layer) of the gland. Primary aldosteronism produces
secondary hypertension associated with hypokalemia and suppressed renin activity.
Aldosterone causes potassium excretion and sodium resorption from the distal tubule and
collecting duct, which leads to the hypokalemia and hypernatremia with increased
extracellular fluid volume. Magnesium and chloride levels can also be reflexly altered.
Changes in calcium levels are not typically observed.

Reference:
Stoelting, Dierdorf, Anesthesia and Co-Existing Disease, 4th edition, p. 94-95, 429-430,
2002.

Andreoli, et al, Cecil Essentials of Medicine, 5th edition, p.245-246, 572, 2001.

American Board of Oral and Maxillofacial Surgery 6


2008 Oral and Maxillofacial Surgery Self Assessment Tool (OMSSAT)

A 70 year-old male with a history of mitral stenosis presents complaining of longstanding


fatigue, exertional dyspnea, and occasional chest pain. Clinically, he shows jugulovenous
distension, peripheral edema, and hepatosplenomegaly. Additional clinical findings most
consistent with this presentation are:

A. decreased pulmonary artery pressure by catheterization.

B. left ventricular hypertrophy by chest radiograph.

C. ECG findings of right axis deviation, peaked P waves in II, III, and AVF.

D. absent pulmonic component of the second heart sound.

Answer: C

Rationale:
Signs and symptoms of cor pulmonale/pulmonary hypertension are described. The ECG
findings in answer C are seen with right ventricular and atrial hypertrophy that results.
Pulmonary arterial pressure would be elevated in this scenario. LVH may be seen as late
sequelae, but not typically. The pulmonic component of the second heart sound is
accentuated rather than absent.

Reference:
Stoelting, Dierdorf, Anesthesia and Co-Existing Disease, 4th edition, p. 128, 2002.

Andreoli, et al, Cecil Essentials of Medicine, 5th edition, p.155-156, 2001.

American Board of Oral and Maxillofacial Surgery 7


2008 Oral and Maxillofacial Surgery Self Assessment Tool (OMSSAT)

Which of the following has a progressive PR interval increase?

A. First degree heart block

B. Mobitz I second degree heart block

C. Mobitz II second degree heart block

D. Third degree heart block

Answer: B

Rationale:
Mobitz type I second degree AV block is also associated with less than compensatory
pause, and a normal QRS duration. It may be seen with drug toxicity such as digitalis and
beta blockers.

First-degree heart block, or first-degree atrioventricular (AV) block, is defined as


prolongation of the PR interval on the ECG to more than 200 msec.

With first-degree AV block, every atrial impulse is transmitted to the ventricles, resulting
in a regular ventricular rate. This type of AV block can arise from delays in the conduction
system in the AV node itself, the His-Purkinje system, or a combination of both

Second-degree heart block, or second-degree atrioventricular (AV) block, refers to a


disorder of the cardiac conduction system in which some atrial impulses are not conducted
to the ventricles. Electrocardiographically, some P waves are not followed by a QRS
complex. Second-degree AV block is composed of 2 types: Mobitz I or Wenckebach block,
and Mobitz II.

The Mobitz I second-degree AV block is characterized by a progressive prolongation of the


PR interval, which results in a progressive shortening of the R-R interval. Ultimately, the
atrial impulse fails to conduct, a QRS complex is not generated, and there is no ventricular
contraction. The PR interval is the shortest in the first beat in the cycle, while the R-R
interval is the longest in the first beat in the cycle.

The Mobitz II second-degree AV block is characterized by an unexpected nonconducted


atrial impulse. Thus, the PR and R-R intervals between conducted beats are constant
Complete heart block, also referred to as third-degree heart block, or third-degree
atrioventricular (AV) block, is a disorder of the cardiac conduction system, where there is
no conduction through the AV node. Therefore, complete disassociation of the atrial and
ventricular activity exists. The ventricular escape mechanism can occur anywhere from the
AV node to the bundle-branch Purkinje system.

American Board of Oral and Maxillofacial Surgery 8


2008 Oral and Maxillofacial Surgery Self Assessment Tool (OMSSAT)

Reference:
Isselbacher, KJ ed. Harrison's Principles of Internal Medicine. McGraw-Hill, Inc. 1994.
p.1015.

American Board of Oral and Maxillofacial Surgery 9


2008 Oral and Maxillofacial Surgery Self Assessment Tool (OMSSAT)

A 64 year-old female is now two days postoperative from a iliac crest graft harvest for a
mandibular defect reconstruction. She continues to have dyspnea at rest since emergence from
anesthesia. Her B-type natriuretic peptide (BNP) assay is elevated. This may indicate that the
patient is suffering from:

A. pulmonary embolism.

B. chronic obstructive pulmonary disease.

C. metabolic acidosis.

D. congestive heart failure.

Answer: D

Rationale:
BNP is a neurohormone that is released by the ventricular myocardium in response to
ventricular volume and pressure overload. In patients who present with dyspnea of
unknown etiology, a plasma BNP > 100 picograms/milliliter (pg/mL) can be used as
evidence of heart failure as a cause of the dyspnea (diagnostic accuracy = 84%). This is
useful in differentiating dyspnea due to heart failure from noncardiac causes.

Reference:
Marino, L., The ICU Book, Lippincott Williams & Wilkins Philadelphia, 2006, p. 262

Angeja, B.G., Grossman, W., Evaluation and Management of Diastolic Heart Failure,
Circulation 2003; 107; 659-663 http://circ.ahajournals.org/cgi/content/full/107/5/659

ACC/AHA 2005 Guideline Update for the Diagnosis and Management of Chronic Heart
Failure in the AdultSummary Article: A Report of the American College of
Cardiology/American Heart Association Task Force on Practice Guidelines, Journal of the
American College of Cardiology, Volume 46, Issue 6, 20 September 2005, Pages 1122-
1123

American Board of Oral and Maxillofacial Surgery 10


2008 Oral and Maxillofacial Surgery Self Assessment Tool (OMSSAT)

A 70 year-old male is undergoing multiple tooth extractions. Shortly after administering local
anesthesia with epinephrine he complains of crushing substernal chest pain that radiates to his
left arm. What is the most appropriate initial therapeutic intervention?

A. Sublingual nitroglycerin

B. Supplemental oxygen

C. Crushed aspirin 325 mg PO

D. Intramuscular morphine sulfate

Answer: B

Rationale:
The patient being treated is likely suffering from an ischemic myocardial injury as
described with the symptom of substernal chest pain. Despite the common acronym
MONA, the initial treatment of ischemic heart disease is oxygen, nitroglycerin, aspirin, and
morphine, in that order. Oxygen administration may limit ischemic myocardial injury,
although its effects on morbidity and mortality of acute infarction are unknown. A short
period of initial routine oxygen supplementation is reasonable during initial stabilization of
the patient, given its safety and the potential for underrecognition of hypoxemia. Farther
down the chain of early treatment of chest pain, a 12 lead ECG would be obtained early in
the assessment of his chest pain. Later, cardiac marker enzymes levels (Troponin T,
Troponin I, CK-MB) would be used in the diagnosis of myocardial injury.

Reference:
American Heart Association: Handbook of Emergency Cardiovascular Care for Healthcare
Providers, American Heart Association, Dallas Texas, 2006, pages 22 - 28
ACC/AHA 2007 Guidelines for the Management of Patients With Unstable Angina/Non
ST-Elevation Myocardial Infarction: A Report of the American College of
Cardiology/American Heart Association Task Force on Practice Guidelines (Writing
Committee to Revise the 2002 Guidelines for the Management of Patients With Unstable
Angina/NonST-Elevation Myocardial Infarction), Journal of the American College of
Cardiology, Volume 50, Issue 7, 14 August 2007, page 39

American Board of Oral and Maxillofacial Surgery 11


2008 Oral and Maxillofacial Surgery Self Assessment Tool (OMSSAT)

What are the beneficial effects of using ACE inhibitors and diuretics in combination for the
treatment of congestive heart failure?

A. Positive chronotropic and inotropic effects

B. Negative chronotropic and inotropic effects

C. Increase preload and afterload

D. Decrease preload and afterload

Answer: D

Rationale:
ACE inhibitors and diuretics are recommended for routine use in treating CHF.
ACE inhibitors block the renin-angiotensin-aldosterone system producing vasodilation by
limiting angiotensin II-induced vasoconstriction. The vasodilation is predominantly
arterial which decreases afterload. This decreases myocardial work and decreases
myocardial energy requirements.

Diuretics decrease extracellular fluid thereby decreasing ventricular filling pressures or


preload. This treats the symptoms of CHF.

Reference:
ACC/AHA 2005 Guideline Update for the Diagnosis and Management of Chronic Heart
Failure in the AdultSummary Article: A Report of the American College of
Cardiology/American Heart Association Task Force on Practice Guidelines, Journal of the
American College of Cardiology, Volume 46, Issue 6, 20 September 2005, Pages 1128-
1129

Cooper, D., The Washington Manual of Medical Therapeutics, 32nd Edition, Chapter 6,
pages 171 and 174, 2007

American Board of Oral and Maxillofacial Surgery 12


2008 Oral and Maxillofacial Surgery Self Assessment Tool (OMSSAT)

Listen to the attached audio clip and identify the cause of the heart murmur.

Sound 1: Please note that sound is not available in this format but the item content is appropriate
for learning purposes.

A. Aortic valve regurgitation

B. Mitral valve regurgitation

C. Tricuspid valve regurgitation

D. Ventricular septal defect

Answer: A

Rationale:
Tricuspid valve regurgitation, mitral valve regurgitation, and ventricular septal defects all
cause murmurs that are audible during the systolic phase of heart function. Aortic valve
regurgitation occurs during the diastolic phase of heart function (represented by the longer
time period between heart sounds) and most frequently occurs due to aortic root dilation or
rheumatic heart disease. A portion of the LV stroke volume is expelled during systole and
regurgitates in the LV during diastole leading to AR regurgitation.

Aortic regurgitation occurs when the aortic valve fails to close completely and blood flows
back into the left ventricle after ejection into the aorta is complete (after S2). Normally,
there is a brief period of time after the aortic valve closes when the ventricle relaxes
isovolumetrically (the mitral valve is also closed during this phase). But when the aortic
valve is leaky, the ventricle begins to fill from the aorta after the incomplete closure of the
aortic valve. This leads to an increase in ventricular volume prior to the opening of the
mitral valve and normal ventricular filling. Because blood is leaving the aorta in two
directions (back into the heart as well as down the arterial network), the aortic diastolic
pressure falls more rapidly thereby leading to a decrease in arterial diastolic pressure.
Because the ventricle fills from both the aorta and the left atrium, there is a large increase
in left ventricular volume and pressure (increased preload), which is best depicted by
pressure-volume loops for this condition. The increased preload causes the left ventricle to
contract more forcefully (Frank-Starling mechanism), thereby increasing ventricular (and
aortic) systolic pressure and increasing stroke volume to help compensate for the
regurgitation. The increase in ventricular end-diastolic pressure, however, also leads to an
increase in left atrial pressure, which can result in pulmonary congestion and edema.
Regurgitation, coupled with enhanced left ventricular stroke volume, results in a
characteristic widening of the aortic pulse pressure. The backward flow of blood into the
ventricular chamber during diastole results in a diastolic murmur between S2 and S1.

Reference:

American Board of Oral and Maxillofacial Surgery 13


2008 Oral and Maxillofacial Surgery Self Assessment Tool (OMSSAT)

Isselbacher, KJ ed. Harrison's Principles of Internal Medicine. McGraw-Hill, Inc. 1994. p.


952-953.
Audio clip used with permission from Dr RS MacWalter, Consultant Physician & Reader
in Medicine, Ninewells Hospital & Medical School, Dundee DD1 9SY. Great examples of
other murmurs can be heard at http://www.dundee.ac.uk/medther/Cardiology/hsmur.html

American Board of Oral and Maxillofacial Surgery 14


2008 Oral and Maxillofacial Surgery Self Assessment Tool (OMSSAT)

Which of the following is the risk factor for sudden cardiac death in a patient with aortic
stenosis?

A 1%

B. 5%

C. 10%

D. 15%

Answer: B

Rationale:
Aortic stenosis (AS) is the obstruction of blood flow across the aortic valve. AS has several
etiologies: congenital unicuspid or bicuspid valve, rheumatic fever, and degenerative
calcific changes of the valve.

Pathophysiology: When the aortic valve becomes stenotic, resistance to systolic ejection
occurs and a systolic pressure gradient develops between the left ventricle and the aorta.
Stenotic aortic valves have a decreased aperture that leads to a progressive increase in left
ventricular systolic pressure. This leads to pressure overload in the left ventricle, which,
over time, causes an increase in ventricular wall thickness (ie, concentric hypertrophy). At
this stage, the chamber is not dilated and ventricular function is preserved, although
diastolic compliance may be affected.

Eventually, however, the left ventricle dilates. This, coupled with a decrease in compliance,
is associated with an increase in left ventricular end-diastolic pressure, which is increased
further by a rise in atrial systolic pressure. A sustained pressure overload eventually leads
to myocardial decompensation. The contractility of the myocardium diminishes, which
leads to a decrease in cardiac output. The elevated left ventricular end-diastolic pressure
causes a corresponding increase in pulmonary capillary arterial pressures and a decrease in
ejection fraction and cardiac output. Ultimately, congestive heart failure (CHF) develops.

In the US: This is a relatively common congenital cardiac defect. Incidence is 4 in 1000
live births.

Mortality/Morbidity: Sudden cardiac death occurs in 3-5% of patients with AS. Adults with
AS have a 9% mortality rate per year. Once symptoms develop the incidence of sudden
death increases to 15-20%, with average survival duration of less than 5 years. Patients
with exertional angina or syncope survive an average of 3 years. After the development of
left ventricular failure, life expectancy is slightly greater than 1 year.

Reference:

American Board of Oral and Maxillofacial Surgery 15


2008 Oral and Maxillofacial Surgery Self Assessment Tool (OMSSAT)

Fuster, V., Hurst's THE HEART, McGraw-Hill, New York, 2004, p2038

American Board of Oral and Maxillofacial Surgery 16


2008 Oral and Maxillofacial Surgery Self Assessment Tool (OMSSAT)

The above EKG shows which of the following rhythms?

A. Ventricular escape rhythm

B. First degree heart block

C. Second degree heart block

D. Third degree heart block

Answer: D

Rationale:
The ECG with third degree heart block has the following three characteristics: P waves are
present with a regular atrial rate faster than the ventricular rate, QRS complexes are present
with a slow ventricular rate, and the P waves bear no relation to the QRS complexes, and
the PR intervals are completely variable because the atria and ventricles are electrically
disconnected.

First-degree AV block, defined as a PR interval exceeding 200 milliseconds in an adult


(180 milliseconds in adolescents), is more accurately described as first-degree AV
conduction delay.

Second degree heart block is divided into Mobitz I and Mobitz II. Mobitz type I, also
called Wenckebach, results in progressive lengthening of the P-R interval with eventual
drop of a QRS complex. Mobitz II is characterized by the sudden loss of a QRS complex
without P-R elongation.

Reference:
Goldberger: Clinical Electrocardiography: A Simplified Approach, 7th ed., Copyright
2006 Mosby, An Imprint of Elsevier

Rakel: Conn's Current Therapy 2006, 58th ed., Copyright 2006 Saunders, An Imprint of
Elsevier

American Board of Oral and Maxillofacial Surgery 17


2008 Oral and Maxillofacial Surgery Self Assessment Tool (OMSSAT)

A trauma patient in the surgical intensive care unit has the following blood gas result.

pH 7.32 (normal range 7.36 7.44)


PaCO2 46 mm Hg (normal range 36-44 mm Hg)
HCO3 23 mEq/L (normal range 22-26 mEq/L)

This finding is indicative of which condition?

A. Respiratory acidosis

B. Respiratory alkalosis

C. Metabolic acidosis

D. Metabolic alkalosis

Answer: A

Rationale:
Respiratory acidosis occurs when the pH is below 7.36 and PaCO2 is above 44 mm/Hg.
Respiratory alkalosis occurs when the pH is above 7.44 and PaCO2 is below 36 mm/Hg.
Metabolic acidosis has a pH below 7.36 and HCO3 is below 22 mm/Hg. Metabolic
alkalosis has a pH above 7.44 and HCO3 is higher than 26 mm/Hg.

Respiratory acidosis occurs with impairment in the rate of alveolar ventilation. Acute
respiratory acidosis occurs with a sudden depression of the medullary respiratory center,
paralysis of the respiratory muscles, and with airway obstruction. Therapy is aimed at
treatment of the underlying disorder and ventilatory support. The question above refers to
uncompensated respiratory acidosis. The body does use metabolic and respiratory
compensatory mechanisms to maintain a constant PaCO2/ HCO3 ratio. For example in a
primary respiratory acidosis or alkalosis, the kidneys provide the compensation by
adjusting the HCO3 reabsorption. In a primary metabolic disorder, the ventilatory system
is mediated by H+ sensitive chemoreceptors in the carotid body which signal an increase or
decrease in ventilation to alter arterial PaCO2 levels. The amount of compensation can be
calculated using the pH, PaCO2, and bicarbonate level.

Respiratory alkalosis occurs with hyperventilation. It can occur in pregnancy, fever and
septic states, with pneumonia, pulmonary embolism, and congestive heart failure. Acute
hyperventilation is characterized by light-headedness, paresthesia, circumoral numbness,
and tingling of the extremities. Tetany occurs in severe cases.
Metabolic acidosis can be due to extrarenal loss of bicarbonate, as in diarrheal diseases, but
can also be caused by high renal excretion of bicarbonate.
Metabolic alkalosis is caused by things such as excessive vomiting and diuretic use leading
to volume depletion. It occurs due to a failure to the kidney to excrete excess bicarbonate.
Treatment is directed at correction of the metabolic disorder.

American Board of Oral and Maxillofacial Surgery 18


2008 Oral and Maxillofacial Surgery Self Assessment Tool (OMSSAT)

Reference:
The ICU Book, 2nd ed., Marino, Williams and Wilkins, 1997 p. 584-586

Cecil Essentials of Medicine, 7th ed. Andreoli and Carpenter, Saunders, 2007, 298-303.

American Board of Oral and Maxillofacial Surgery 19


2008 Oral and Maxillofacial Surgery Self Assessment Tool (OMSSAT)

Which of the following is the most important risk factor for developing nosocomial pneumonia?

A. Malnutrition

B. Mechanical ventilation

C. Nursing home residence

D. Tobacco abuse

Answer: B

Rationale:
While malnutrition, tobacco use, and residency in a nursing home facility are risk factors,
endotracheal intubation and mechanical ventilation is by far the most important contributor
leading to nosocomial pneumonia. Endotracheal intubation provides a pathway for
bacterial contamination in the lungs.

This is increased in the patient with chest trauma or lung injury. The aspiration of oral
secretions into the upper airways is the inciting event in most cases of pneumonia.
Averages of 1 billion bacteria are found in each milliliter of saliva.

Reference:
Kokko, J., Stein, S., The Emory University Comprehensive Board Review in Internal
Medicine. McGraw-Hill. New York, 2000, p. 286.

Mandell, Bennett, & Dolin: Principles and Practice of Infectious Diseases, 6th ed.,
Chapter 314 Infections in the Elderly, Pneumonia, Copyright 2005 Churchill
Livingstone, An Imprint of Elsevier

Marino, P., The ICU Book. Lippincott Williams Wilkins. Baltimore, 2004, p. 516-517.

American Board of Oral and Maxillofacial Surgery 20


2008 Oral and Maxillofacial Surgery Self Assessment Tool (OMSSAT)

Chronic complications of which disease process includes cirrhosis of the liver, pancreatic
dysfunction, sinusitis, and bronchial hyperreactivity?

A. Chronic bronchitis

B. Cystic fibrosis

C. Sarcoidosis

D. Tuberculosis

Answer: B

Rationale:
Cystic fibrosis is an autosomal recessive disorder. In the Untied States, it affects 1 in 2500
Caucasians, 1 in 17000 African Americans, and 1 in 90000 Asians. Over 230 mutations
have been identified with various degrees of disease severity, with the most common defect
on the long arm of chromosome 7. The genetic defect causes defective chloride transport
and increased sodium reabsorption in airway and ductal epithelia, creating abnormally
thick and viscous secretions in the respiratory, hepatobiliary, gastrointestinal, and
reproductive tracks. The thick secretions cause luminal obstruction and destruction of
exocrine ducts. The median survival age is 40 years.

The disease often manifests in childhood. Respiratory complications in addition to chronic


sinusitis and bronchial hyperreactivity include mucous plugging within the lungs. Mucous
plugs can also occur in the salivary glands, although not common.

A diagnostic criterion is a sweat chloride concentration > 60 mEq/L. Genetic testing can
also be completed to confirm the diagnosis. Clinical findings are also important in
diagnosis. Presence of mucoid Pseudomonas aeruginosa in sputum is characteristic of CF.
Chest radiograph abnormalities include bronchiectasis with upper lobe and right-sided
predominance. Digital clubbing and hypertrophic osteoarthropathy are present.
Spontaneous pneumothorax and pulmonary hypertension are negative prognostic findings.

Standard therapy for CF includes chest physical therapy, antibiotics as needed, adequate
nutrition, and exercise. Bronchodilators may be used for patients with bronchospasm or
airflow limitation. Lung transplants for patients with advanced CF are being performed.

Reference:
Kokko, J., Stein, S., The Emory University Comprehensive Board Review in Internal
Medicine. McGraw-Hill. New York, 2000, p. 305-306.

Andreoli T: Cecil Essentials of Medicine, W. B. Saunders Company, Philadelphia, 2001, p.


220-221.

American Board of Oral and Maxillofacial Surgery 21


2008 Oral and Maxillofacial Surgery Self Assessment Tool (OMSSAT)

Which of the following is commonly associated with asthma?

A. Decreased lung compliance

B. Obtunded cholinergic sensitivity

C. Gastroesophageal reflux disease

D. Hypocarbia

Answer: C

Rationale:
Upwards of 80% of asthmatics exhibit gastroesophageal reflux disease (GERD). Although
respiratory disease can induce GER by increasing abdominothoracic pressure gradients,
there is substantial evidence suggesting that the more common pathway is that for GER to
trigger respiratory symptoms, particularly of asthma. This may occur from microaspiration
with resultant direct inflammation and bronchoconstriction, from vagally mediated effects
from stimulation of upper airway receptors, or from esophageal afferent irritation causing a
neurogenic reflex.

Decreased lung compliance is associated with restrictive lung disease not obstructive, and
asthma is an obstructive pulmonary disease.
Asthmatics commonly have increased cholinergic sensitivity.
Hypercarbia is sequelae of an asthmatic attack.

Reference:
Behrman: Nelson Textbook of Pediatrics, 17th ed., Copyright 2004 Saunders, An
Imprint of Elsevier, Chapter 387, Gastroesophageal reflux and respiratory disorders.

Mason: Murray and Nadel's Textbook of Respiratory Medicine, 4th ed. Chapter 78
Pulmonary Complications of Abdominal Disease.

American Board of Oral and Maxillofacial Surgery 22


2008 Oral and Maxillofacial Surgery Self Assessment Tool (OMSSAT)

Which of the following best characterize restrictive lung disease?

A. Decrease in total lung capacity

B. Decrease in FEV1/FVC ratio

C. Increase in vital capacity

D. Increase in airway resistance

Answer: A

Rationale:
Restrictive disorders are best evaluated by measuring lung volumes. The severity of the
restrictive defect is based on the TLC (total lung capacity). In restrictive lung disorders,
the FEV1 may be decreased, but the FEV1/FVC ratio will be preserved. The TLC, FRC
(functional residual capacity) and RV (residual volume) will all be decreased. Airway
resistance is not affected in a restrictive lung disease.

Restrictive disorders can occur in three circumstances: lung disorders, disorders of the
chest wall, and neuromuscular disease. Lung disorders with interstitial infiltration typically
show restriction caused by increased elastic recoil such as observed in idiopathic
pulmonary fibrosis and sarcoidosis. Lung edema as a result of congestive heart failure can
also cause a restrictive pattern. Chest wall abnormalities may exhibit themselves with a
restrictive pattern by restricting lung expansion as observed in kyphoscoliosis, obesity , and
ankylosing spondylitis, Included in the chest wall abnormalities are the pleural diseases
(pleural effusion, pneumothorax,), space occupying lesions (tumors), and conditions
causing increased abdominal girth such as pregnancy, ascites, and large intra-abdominal
tumors. Some neuromuscular disorders cause restriction by preventing normal excursion
of the lung during breathing as observed in patients with myasthenia gravis, amyotrophic
lateral sclerosis, diaphragmatic paralysis, and the Guillain-Barre syndrome. Patients may
also restrict lung excursion during inspiration as a result of pain or somnolence (drug
overdose). Lung resection during lobectomy will also cause a restrictive defect.

Decrease in the FEV1/FVC ratio is the hallmark of obstructive lung disease. Airway
resistance is increased in obstructive lung disease (asthma).

Reference:
Andreoli T: Cecil Essentials of Medicine, W. B. Saunders Company, Philadelphia, 2001, p.
208-210.

Clinical Anesthesiology 4th ed., Lange Medical Books/McGraw Hill, New York, 2006
pp572-580

American Board of Oral and Maxillofacial Surgery 23


2008 Oral and Maxillofacial Surgery Self Assessment Tool (OMSSAT)

Rita K Cydulka: Emergency Medicine 6th ed., McGraw Hill, New York, 2004 pp 475-480

American Board of Oral and Maxillofacial Surgery 24


2008 Oral and Maxillofacial Surgery Self Assessment Tool (OMSSAT)

What is the most likely diagnosis for a patient with the following findings after pulmonary
function testing?

FVC - normal
FEV1 - decreased
FEV1/FVC - decreased

A. Respiratory muscle weakness

B. Obstructive lung disease

C. Restrictive lung disease

D. Spontaneous pneumothorax

Answer: B

Rationale:
Obstructive lung disease includes asthma, chronic bronchitis, emphysema, bronchiectasis,
cystic fibrosis, and upper airway obstruction.

Diseases such as asthma and bronchitis, which obstruct the airway, reduce expiratory flow
rates and therefore reduce FEV1 and FEV1/FVC. An obstructive defect is essentially
characterized by a disproportionate decrease in the airflow rate relative to the actual
volume exhaled (i.e., FVC) and indicates airway narrowing and flow limitation during
expiration. Values for FEV1/FVC that are lower than 70% reflect mild obstruction, those
lower than 60% suggest moderate obstruction, and those lower than 50% indicate severe
obstruction.

In restrictive lung disease and respiratory muscle weakness, the forced vital capacity and
FEV1 are decreased, while the FEV1/FVC ratio is normal. Pneumothorax can be
considered a restrictive lung disease.

Reference:
Andreoli T: Cecil Essentials of Medicine, W. B. Saunders Company, Philadelphia, 2001, p.
208-210.

Kokko, Jl, Stein,S., The Emory University Comprehensive Board Review in Internal
Medicine. Pg 271-283, 301-311 McGraw-Hill. New York, 2000.

American Board of Oral and Maxillofacial Surgery 25


2008 Oral and Maxillofacial Surgery Self Assessment Tool (OMSSAT)

Which is considered the most definitive in the diagnosis of pulmonary embolism?

A. Ultrasound of femoral-popliteal system

B. Nuclear ventilation-perfusion scan

C. Chest radiograph

D. Contrast-enhanced CT scan

Answer: D

Rationale:
Pulmonary embolism (PE) is usually a complication of venous thrombosis in the deep veins
of the lower extremities that originate in the femoral-iliac-pelvic veins. Causes of
thrombosis include: vessel injury, hypercoagulability, and stasis. The best treatment of PE
is prevention, especially in high risk patients (malignancy, obesity, age greater than 40, PE
history, extensive surgery, and immobilization). In hemodynamically unstable patients it is
important to definitively diagnose and treat. The test that was considered the gold
standard for many years was pulmonary angiogram. Helical or multi-detector CT scans
are now becoming the preferred test for definitive diagnosis.

In hemodynamically stable patients the following is a suggested treatment protocol.


1) Maintain high index of suspension
2) Signs and symptoms: - dyspnea, tachypnea, tachycardia, chest pain, PO2 less than 80
(none is completely diagnostic)
3) Chest XR: may show localized volume loss, atelectasis, peripheral infiltrates, and/or
pleural effusions
4) Ventilation-perfusion scan (V/Q): This scan uses lung ventilation with radiolabeled
tracer gas with lung perfusion by radiolabeled microocclusive particles. A normal scan
will rule out PE. Results of a low or intermediate scan gives likelihood between 15-55%
and usually indicates pulmonary angiogram for conformation. The results of V/Q are
dependent on quality of the scan, interpretation of the scan, and clinical probability of PE.
5) Impedance plethysmography and Doppler ultrasonograph are noninvasive techniques
for diagnosis of deep vein thrombosis in the lower extremity that may support diagnosis of
PE. These tests have a low sensitivity. Venous duplex scanning (ultrasonic venous
imaging with Doppler blood-flow imaging) has greater accuracy than the above
procedures.
6) Contrast-enhanced CT is increasingly used as the initial radiologic study in the
diagnosis of PE, especially in patients with abnormal chest radiographs in whom
scintigraphic results are more likely to be nondiagnostic.

CT show emboli directly, as does pulmonary angiography, and it is also noninvasive,


cheaper, and widely available. CT is the only test that can provide significant additional

American Board of Oral and Maxillofacial Surgery 26


2008 Oral and Maxillofacial Surgery Self Assessment Tool (OMSSAT)

information related to alternate diagnoses; this is a clear advantage of CT compared with


either pulmonary angiography or scintigraphy.

Because DVT and PE are part of the same disease process, CT venography can easily be
performed after CT pulmonary angiography.

When PE is identified, it is characterized as acute or chronic. An embolus is acute if it is


situated centrally within the vascular lumen or if it occludes a vessel (vessel cutoff sign).
Acute PE commonly causes distention of the involved vessel. An embolus is chronic if (1)
it is eccentric and contiguous with the vessel wall, (2) it reduces the arterial diameter by
more than 50%, (3) evidence of recanalization within the thrombus is present, and (4) an
arterial web is present.

PE is further characterized as central or peripheral, depending on the location or the arterial


branch involved. Central vascular zones include the main pulmonary artery, the left and
right main pulmonary arteries, the anterior trunk, the right and left interlobar arteries, the
left upper lobe trunk, the right middle lobe artery, and the right and left lower lobe arteries.
Peripheral vascular zones include the segmental and subsegmental arteries of the right
upper lobe, the right middle lobe, the right lower lobe, the left upper lobe, the lingula, and
the left lower lobe.

7) Pulmonary angiogram: The last test in this protocol because of its invasive nature and
inherent risk. The role of conventional angiograghy is limited to patients in which other
results are non-diagnostic and/or clinical suspicions are high.

D-dimer: Blood test that is used as an aid to diagnose thrombosis. It is best used to rule out
thromboembolic disease when probability is low. A negative result practically rules out
thrombosis and a positive test may indicate thrombosis but does not rule out other
etiologies (DIC) and requires further testing for DVT/PE.
D-dimer is unique in that they are breakdown products of fibrin mesh after being
stabilized/cross-linked by Factor XIII. This is the final step in the generation of a
thrombosis and indicate a thrombosis is present.

Reference:
Kokko, J. Stein, S., The Emory University Comprehensive Board Review in Internal
Medicine. Pg 327-331 McGraw-Hill New York, 2000.

Marino, P The ICU Book. Pg 106-115, 376-379 Lippincott Williams Wilkins. Baltimore,
2006.

Eng, j. Krishman, JA. Segal, J. Bolger, D. Tamariz, L. Accuracy of CT in the Diagnosis of


Pulmonary Embolism. Journal of Roentololgy. 2004; 183:1819-1827.

American Board of Oral and Maxillofacial Surgery 27


2008 Oral and Maxillofacial Surgery Self Assessment Tool (OMSSAT)

Which of the following pharmacologic groups is a risk factor for venous thromboembolism?

A. Hypercholesterolemics

B. Estrogen replacement therapy

C. Non steroidal anti-inflammatories

D. ACE inhibitors

Answer: B

Rationale:
Estrogens and birth control pills are can result in hypercoagulability and are a risk factor
for venous thromboembolism Pulmonary embolism is usually a complication of venous
thrombosis of the femoral-iliac-pelvic-popliteal system of the lower extremity. Factors that
may contribute to thrombosis include: venous stasis, hypercoagulability, endothelial injury.
Patient risk factors include: heart failure, malignancy, immobilization, trauma, obesity,
advanced age, pregnancy, extensive surgery, and oral contraceptives.
There is no data to support hypercholerterolemics, NSAID's or ACE inhibitors as risk
factors for venous thromboembolism.
Prevention is the best treatment which includes: intermittent compression device, anti-
embolism stocking, selective heparin use, and early post-operative ambulation.

Reference:
Kokko, J., Stein, S., The Emory University Comprehensive Board Review in Internal
Medicine. McGraw-Hill. New York, 2000, p.329-331.

Marino, P. The ICU Book. Lippincott Williams Wilkins. Baltimore, 2006, p. 106-108.

American Board of Oral and Maxillofacial Surgery 28


2008 Oral and Maxillofacial Surgery Self Assessment Tool (OMSSAT)

What is the first course of action for patients strongly suspected or documented to have a deep
vein thrombosis (DVT)?

A. Warfarin

B. Thrombectomy

C. Heparin

D. Streptokinase

Answer: C

Rationale:
Treatment should include supportive care for hypoxemia and hypotension. Anticoagulation
with IV heparin should be started as soon clinical suspicion of PE without waiting for
diagnostic studies. The goal is for 1.5-2.5 times the control value of PTT for 5-10 days.
Any delay in reaching therapeutic PTT range has been shown to increase the risk of
progressive thrombosis and recurrent pulmonary embolism. Then, warfarin therapy is
initiated with an INR goal of 2-3 for 3-6 months or indefinitely if patient is at high risk. If
contraindications exist for anticoagulation or if patient has reoccurrence of PE in spite of
anticoagulation the treatment of choice is vena caval interruption. (Greenfield filter).
Systemic thrombolytic therapy and embolectomy may be indicated in very rare cases.

Heparin (unfractionated) is a glycosaminoglycan with varying molecular size. It


potentiates the action of antithrombin III thereby inactivating thrombin and prevents the
conversion of fibrinogen to fibrin.

Low molecular weight heparin (LMWH) has been used in place of the unfractionated
heparin with comparable results for treatment of DVT's and non-massive PE. LMWH is
enzymatically broken down into smaller more uniform molecules with less variation in
anti-coagulation response. Because of the predictability of the response and route of
administration these patients are, in some cases, treated as outpatients, dramatically
reducing cost but not changing clinical outcomes. LMWH is given subcutaneously with a
slower onset then IV heparin but dosing is less frequent, there is less heparin-induced
thrombocytopenia, less heparin-induced osteoporosis, and slightly less risk of bleeding.

Reference:
Green, GB, Harris, IS., Lin, GA., Moylan, KC., The Washington Manual of Medical
Therapeutics, 31st Edition. Lippincott Williams Wilkins. Baltimore, 2004.

Marino, P., The ICU Book. Pg 112-115, 376-379 Lippincott Williams Wilkins. Baltimore,
2004.

American Board of Oral and Maxillofacial Surgery 29


2008 Oral and Maxillofacial Surgery Self Assessment Tool (OMSSAT)

Hauer, Karin Low Molecular Weight Heparin in the Treatment of DVT. West J Med Oct.
1998; 169:240-244.

Quilan, D. McQuillan, A Low molecular weight Heparin Compared with IV


Unfractionated Heparin for Treatment of Pulmonary Embolism. Annals of Internal
Medicine. 2004; 140:175-183.

American Board of Oral and Maxillofacial Surgery 30


2008 Oral and Maxillofacial Surgery Self Assessment Tool (OMSSAT)

A 60 year-old patient reports a history of smoking and frequent productive cough present for at
least three months in each of the last two years. What is the most likely diagnosis?

A. Emphysema

B. Cystic fibrosis

C. Asthma

D. Chronic bronchitis

Answer: D

Rationale:
Chronic bronchitis is defined as the presence of a productive cough for at least 3 months in
each of the past 2 years in a person with excessive mucous secretion not due to other
disease. The most common cause is cigarette smoking. Chronic bronchitis may result in
carbon dioxide retention, chronic hypoxemia, erythrocytosis, pulmonary hypertension and
right heart failure. The respiratory drive becomes less sensitive to carbon dioxide retention
and may potentially be depressed by oxygen administration. (Blue Bloater Syndrome)

Emphysema is characterized by irreversible enlargement of the airways distal to the


terminal bronchioles and destruction of the alveolar septa. Emphysema will usually have
little sputum production and a cough with exertion.

Asthma is characterized by airway inflammation and hyper-reactivity in response to


various stimuli. Asthma produces episodic attacks of dyspnea, cough, and wheezing.
Cystic fibrosis is characterized by secretion of highly viscous mucous with fibrosis and
chronic lung infections.

Reference:
Clinical Anesthesiology 4th ed., Lange Medical Books/McGraw Hill, New York, 2006
pp572-580

Rita K Cydulka: Emergency Medicine 6th ed., McGraw Hill, New York, 2004 pp 475-480

American Board of Oral and Maxillofacial Surgery 31


2008 Oral and Maxillofacial Surgery Self Assessment Tool (OMSSAT)

Which of the following pulmonary function tests is most useful in evaluating the asthmatic
patient?

A. TLC (Total lung capacity)

B. FEV1 (Forced expiratory volume in 1 second)

C. FRC (Functional residual capacity)

D. VC (Vital capacity)

Answer: B

Rationale:
The FEV1 measures the amount of air which can be forcibly expelled in 1 second and
reflects the degree of large airway obstruction. FEV1 values are normally around 3L for
men and 2L for women, but vary with each individual. FEV1/FVC should normally be
>70% and an FEV1 or FEV1/FRC ratio less than 50% indicates moderate to severe
asthma. TLC and FRC may be elevated in obstructive disease. Obstructive disease (such
as asthma) is characterized by difficult expiration. Either more force is required to expire
a given volume of air, or emptying of the lungs is slowed, or both. Spirometry
(Pulmonary function tests) measures both lung volume and gas flow. Pulmonary function
tests will often demonstrate partial reversibility (improvement in the FEV1 of at least
15%) after the administration of bronchodilators. The absence of improvement in
pulmonary function tests after the administration of a bronchodilator does not constitute
proof of irreversible airway obstruction.

Reference:
Clinical Anesthesiology 4th ed., Lange Medical Books/McGraw Hill, New York, 2006
pp574

Rita K Cydulka: Emergency Medicine 6th ed., McGraw Hill, New York, 2004 pp 468

Current Medical Diagnosis and Treatment 36th ed., Appleton and Lange, Stamford CT,
1997 pp242

Pathophysiology: The Biologic Basis for Disease in Adults and Children, Mosby, St.
Louis, 1990 pp 1048, 1049

American Board of Oral and Maxillofacial Surgery 32


2008 Oral and Maxillofacial Surgery Self Assessment Tool (OMSSAT)

Which of the following medications should be avoided in a patient with a history of nasal
polyposis and moderate asthma?

A. Acetaminophen (Tylenol)

B. Tramadol (Ultram)

C. Propoxyphene

D. Diclofenac

Answer: D

Rationale:
Aspirin induced asthma (AIA) is characterized by severe and sometimes life-threatening
episodes of asthma following the use of NSAIDS (nonsteroidal anti-inflammatory drugs).
Individuals with AIA suffer from chronic asthma that is often steroid dependent and
associated with a significant number of individuals with chronic rhinosinusitis and nasal
polyps (Sampter's Triad). AIA usually appears in the 3rd to 4th decade of life.

Patients with AIA react to NSAIDS that inhibit cyclooxygenase enzyme 1 (COX-1).
COX-1 catalyzes the formation of prostaglandins and thromboxanes from cell membrane
arachidonic acid. If the COX-1 is inhibited, it may cause a shift to an alternative pathway
of arachidonic acid metabolism involving 5 lipoxygenase. The products of 5 lipoxygenase
include various leukotrienes that are potent inflammatory mediators. These leukotrienes
can act as bronchoconstrictors. These leukotrienes may also increase mucus secretion,
induce swelling of the nasal mucosa, promote airway edema and attract eosinophils into
the airways. Increased leukotriene concentrations have been detected in AIA patients
compared to non AIA patients both at baseline and following ASA (Aspirin) challenge.

American Board of Oral and Maxillofacial Surgery 33


2008 Oral and Maxillofacial Surgery Self Assessment Tool (OMSSAT)

Reference:
Jonathan P. Arm, Lung Biology in Health and Disease Vol. 115., Fatal Asthma, Marcei
Dekker, 1998 pp335-345
Rita K Cydulka: Emergency Medicine 6th ed., McGraw Hill, New York, 2004 pp 475-480

Namazy J., Simon R., Current Review of Asthma., Respiratory Reactions to Anti-
inflammatory Drugs, Current Medicine Inc., 2003 pp91-98

Elliot Israel MD., Up To Date, 2007 Aspirin-Induced Asthma

American Board of Oral and Maxillofacial Surgery 34


2008 Oral and Maxillofacial Surgery Self Assessment Tool (OMSSAT)

An asthmatic patient begins to complain of severe dyspnea and a cough. Auscultation of the
lungs reveals wheezing. Which of the following drug classes reflects the preferred initial
treatment?

A. Xanthines

B. Adrenergics

C. Anticholinergics

D. Corticosteroids

Answer: B

Rationale:
Beta adrenergic agents are the preferred initial rescue medication for acute bronchospasm.
The ideal beta adrenergic agent would possess pure beta 2 selective activity without cardiac
side effects. Older catecholamine bronchodilators such as epinephrine and isoproterenol
are not beta 2 specific and have a short duration of action. Epinephrine (1:1000 or 1mg/ml
0.3mg-0.5mg SC every 20 minutes for 3 doses) may be given to patients unable to
coordinate aerosolized or MDI treatments. Newer agents such as the resorcinol
bronchodilators (metaproterenol, terbutaline and fenoterol) and saligenin bronchodilators
(albuterol and carbuterol) have greater beta 2 specificity and a longer duration of action.
Albuterol is an inhaled short-acting beta 2 agonist that can be administered as a nebulized
solution or as an MDI (metered dose inhaler). Albuterol via MDI may be given at
90ug/puff with 4-8 puffs every 20 minutes up to 4 hours for the adult patient. Xanthines
include medications such as theophylline. They are not considered a first line treatment for
acute asthma. Xanthines are frequently reserved for those patients whose symptoms persist
despite the use of an inhaled steroid and beta 2 antgonist. Therapeutic levels are often
associated with gastrointestinal, cardiac, and central nervous system side effects. There is a
narrow therapeutic index, toxicity may occur and be fatal. Anticholinergics such as
atropine or ipratropium bromide are bronchodilators, but their usage can produce side
effects such as tachycardia. Corticosteroids such as hydrocortisone and betamethasone are
highly effective in the chronic management of asthma by reduction of lumen mucosa
inflammation, and they may restore some beta adrenergic responsiveness, but the onset of
the effect may be delayed 4-8 hours.

Reference:
Clinical Anesthesiology 4th ed., Lange Medical Books/McGraw Hill, New York, 2006
pp572-576

Rita K Cydulka: Emergency Medicine 6th ed., McGraw Hill, New York, 2004 pp 467-473

American Board of Oral and Maxillofacial Surgery 35


2008 Oral and Maxillofacial Surgery Self Assessment Tool (OMSSAT)

Auscultation of the lungs reveals the finding on the attached audio clip. What is the finding that
you hear?

Sound 2: Please note that sound is not available in this format but the item content is appropriate
for learning purposes.

A. Normal breath sounds

B. Rales

C. Wheezing

D. Crackles

Answer: C

Rationale:
The breath sound in the clip is an expiratory wheeze. Wheezing is caused by high-velocity
air flow through a narrowed or obstructed airway. The longer the wheeze and the higher
the pitch, the worse the obstruction. If a wheeze is heard bilaterally, it may be caused by
asthma or acute or chronic bronchitis.

Crackles are heard most often during inspiration and are characterized by discrete
discontinuous sounds, each lasting a few milliseconds. Crackles may be fine and high
pitched or coarse and low pitched. They are caused by the disruptive passage of air
through the small airways in the respiratory tree. Rales is another term for crackles.
Crackles are due to abnormalities of the lungs (pneumonia, fibrosis, early congestive heart
failure), or of the airways (bronchitis, bronchiectasis).

Reference:
Mosby's Guide to Physical Examination, 6th ed. Seidel et al, 2006, p 386-387

Audio clip used with permission from Dr. Samuel Louie, University of California-Davis.
Other examples of respiratory sounds can be heard at
http://medocs.ucdavis.edu/IMD/420C/sounds/lngsound.htm

American Board of Oral and Maxillofacial Surgery 36


2008 Oral and Maxillofacial Surgery Self Assessment Tool (OMSSAT)

A 25 year-old male is undergoing general anesthesia for treatment of a mandible fracture. The
anesthesiologist notes difficulty ventilating the patient and obtains the attached radiograph.
What treatment is appropriate for the patients condition?

A. Placement of a needle on the right

B. An albuterol nebulizer treatment

C. Antibiotics

D. Placement of a chest tube on the left

Answer: A

Rationale:
A tension pneumothorax is the accumulation of air in the pleural space, creating positive
pressure. Because tension pneumothorax can cause hemodynamic compromise, this
medical emergency requires immediate decompression. In addition to resistance to
ventilation, the blood pressure decreases and central venous pressure increases. The
increased pressure should be relieved immediately with needle thoracostomy, which is
performed by inserting a large-bore (16 or 18-gauge) needle through the second or third
intercostal space anteriorly on the involved side. A chest tube is also appropriate treatment

American Board of Oral and Maxillofacial Surgery 37


2008 Oral and Maxillofacial Surgery Self Assessment Tool (OMSSAT)

for a tension pneumothorax, however in this case it would be on the wrong side of the
patient.

An albuterol nebulizer treatment would be used for acute asthma attack. An exacerbation
of asthma under anesthesia can cause difficulty with mechanical ventilation. Antibiotics
would be prescribed for pneumonia. The onset of pneumonia would not typically cause
acute difficulty with ventilation.

Reference:
Marx: Rosen's Emergency Medicine: Concepts and Clinical Practice, 6th ed, Chapter 42

Roberts: Clinical Procedures in Emergency Medicine, 4th ed. Saunders, 2004, Chapter 10.

Radiograph from Dr. Brian Mullan, MD, University of Iowa Hospitals and Clinics

American Board of Oral and Maxillofacial Surgery 38


2008 Oral and Maxillofacial Surgery Self Assessment Tool (OMSSAT)

A 32 year-old female presents with a chief complaint of weakness with chewing. Her symptoms
are worse during evening meals. Physical examination shows diplopia and lid ptosis. If her
clinical signs improved with the administration of edrophonium which of the following would
you suspect?

A. Mutation in the dystrophin gene

B. Antibodies to the presynaptic voltage-gated calcium channels

C. Antibodies to sarcoglycan proteins

D. Antibodies against the postsynaptic nicotinic acetylcholine receptor

Answer: D

Rationale:
The patient is demonstrating classic findings associated with myasthenia gravis. This
disease process is a result of autoantibodies to the postsynaptic nicotinic acetylcholine
receptor. Its prevalence is about 1 in 7500 and although it affects all age groups, it is most
common in males in their fifties and sixties, and females in their twenties and thirties.
Cardinal features include weakness and fatigability of muscles which is progressive and
may improve with rest. Cranial muscle weakness usually involves the eyelids and
extraocular muscles early on resulting in diplopia and ptosis. Weakness with chewing may
result after a prolonged effort, and nasal speech as well as difficulty swallowing may also
result from palatal and pharyngeal muscle weakness. Limb weakness is proximal and may
be asymmetric. Deep tendon reflexes are preserved. Occasionally, respiratory muscle
weakness may develop requiring ventilatory assistance; this is known as a myasthenic
crisis. Foil A is associated with Duchenne Muscular Dystrophy. Foil B represents the
pathophysiology of Lambert-Eaton Myasthenic Syndrome. Foil C is the etiology of Limb-
Girdle muscular dystrophy.

Reference:
Fauci et al., Harrison's Principles of Internal Medicine 14th ed., McGraw-Hill, 1998, pp.
2469-2472.

Goetz: Textbook of Clinical Neurology, 2nd ed., Copyright 2003 Saunders, An Imprint
of Elsevier

American Board of Oral and Maxillofacial Surgery 39


2008 Oral and Maxillofacial Surgery Self Assessment Tool (OMSSAT)

A 67 year-old gentleman presents to your office with xerostomia and keratoconjunctivitis sicca.
He has a long history of smoking and recently diagnosed lung cancer. You note intermittent
eyelid ptosis, and decreased deep tendon reflexes that improve briefly with exertion, but with
sustained activity worsen. Which disorder do you suspect?

A. Hyperthyroidism

B. Myasthenia gravis

C. Oculopharyngeal dystrophy

D. Lambert-Eaton myasthenic syndrome

Answer: D

Rationale:
Lambert-Eaton myasthenic syndrome is a disorder of the presynaptic neuromuscular
junction. It causes symptoms that resemble closely those of myasthenia gravis. However,
it is typically associated with a malignancy, most commonly small cell cancer of the lung.
This is thought to be caused by autoantibodies to the calcium channels at motor nerve
terminals which impairs the release of acetylcholine. Treatment of the disorder involves
immunosuppression often with steroids, as well as plasmapheresis, which is often
performed preoperatively, along with cessation of the anticholinesterase medications. A
differential diagnosis includes: hyperthyroidism, botulism, intracranial lesions,
neurasthenia, and progressive external ophthalmoplegia.

Reference:
Fauci et al., Harrison's Principles of Internal Medicine 14th ed., McGraw-Hill, 1998, pp.
2469-2472.

Ferri: Ferri's Clinical Advisor 2007: Instant Diagnosis and Treatment, 9th ed., Copyright
2007 Mosby, An Imprint of Elsevier

American Board of Oral and Maxillofacial Surgery 40


2008 Oral and Maxillofacial Surgery Self Assessment Tool (OMSSAT)

A 5 year-old male presents to your office with his mother. In the dental chair, the boy pushes his
hands against his knees to elevate his upper torso into an upright position. You also notice that
his calf muscles are rather large when compared with his thighs, and that he is intellectually
impaired. What is the disease process is most likely affecting this young person?

A. Myasthenia gravis

B. Dermatomyositis

C. Polymyositis

D. Duchenne muscular dystrophy

Answer: D

Rationale:
The boy performs a classic Gowers' maneuver when elevating himself to an erect position,
which is a giveaway for the diagnosis of Duchenne muscular dystrophy. This is a
hereditary myopathy that is also known as pseudohypertrophic muscular dystrophy. It is an
X-linked recessive disorder with an incidence of 30 per 100,000 males. While present at
birth, it usually becomes apparent between the ages of 3 and 5. The boys have difficulty
keeping up with physical activities with their peers, and have a progressive muscle
weakness. The enlarged calves are also a classic physical finding and while
inappropriately termed pseudohypertrophy, it is actually a result of fibrofatty tissue
replacing the muscle in the enlarged calves. These children typically are unable to walk by
age 12, and also suffer from scoliosis and contractures. They commonly suffer from
intellectual impairment and have an IQ approximately one standard deviation below their
peers. Cardiomyopathy is present in most of these patients, though death is usually due to
pulmonary infections and an increased incidence of aspiration pneumonia.

Reference:
Fauci et al., Harrison's Principles of Internal Medicine 14th ed., McGraw-Hill, 1998, pp.
2473-2475.

Behrman: Nelson Textbook of Pediatrics, 17th ed., Copyright 2004 Saunders, An


Imprint of Elsevier, Chapter 600.

American Board of Oral and Maxillofacial Surgery 41


2008 Oral and Maxillofacial Surgery Self Assessment Tool (OMSSAT)

What finding is commonly associated present with Sturge-Weber Syndrome?

A. Seizures

B. Lip pits

C. Caf-au-lait spots on trunk

D. Pronounced mandibular hypoplasia

Answer: A

Rationale:
The classic clinical manifestations of Sturge-Weber Syndrome are facial vascular nevi
(port-wine stain), epilepsy, cognitive deficits, and hemiparesis or hemiplegia, hemianopia,
glaucoma. Patients with Sturge-Weber syndrome may actually have enlargement of the
associated lip, maxilla, or mandible. Cerebral angiography of these patients reveals
capillary, venous and arteriovenous anomalies of the leptomeninges. The anomalous
circulation is responsible for the progressive degeneration and atrophy of the cerebral
hemispheres. Extensive leptomeningeal abnormalities can cause seizures, contralateral
hemiplegia, and delayed motor and cognitive skills.

Caf-au-lait spots on a patient's trunk are commonly associated with McCune-Albright


Syndrome. Pronounced mandibular hypoplasia can be associated with such conditions as
Pierre-Robin Sequence or Treacher-Collins Syndrome but is not associated with Sturge-
Weber Syndrome.

Reference:
Goldman L, Ausiello D: Cecil Textbook of Medicine 22nd ed., Saunders, Philadelphia,
2003 pp 2363

Cummings: Otolaryngology: Head & Neck Surgery, 4th ed., Copyright 2005 Mosby,
Inc, Chapter 174 Vascular tumors and malformations of the head and neck, capillary
malformation.

American Board of Oral and Maxillofacial Surgery 42


2008 Oral and Maxillofacial Surgery Self Assessment Tool (OMSSAT)

A 24 year-old patient with a panfacial fracture acutely develops speech difficulties and
confusion. You have been called to the surgical intensive care unit to examine him for a newly
discovered maxillary fracture. While talking with the patient, he acutely develops difficulty
speaking despite remaining fully conscious. His vital signs remain stable, and he recovers fully
after several minutes. What is the most likely explanation for his acute speech difficulties and
confusion?

A. Transient ischemic attack

B. Hypoglycemia

C. Seizure

D. Narcolepsy

Answer: C

Rationale:
Posttraumatic seizures can occur as an acute result of blunt or penetrating head trauma.
Immediate posttraumatic seizures occur with 24 hours of injury, and are more common in
children than in adults. Within the first year after significant head trauma, the incidence of
seizures is at least 12 times that of the general population. The incidence of seizures after
injury with neurologic deficit without dural violation is 7% to 39%. When the dura is
disrupted, the incidence is 20 57%. Most patients with pos-traumatic epilepsy have
partial seizures that can affect sensory, motor or autonomic areas of the brain. If the speech
centers are involved in the ictal discharge, speech can be disturbed or arrested. These
seizures are usually short lived, and the patients recover function quickly.

The other answers can be associated with speech difficulties but are not necessarily related
to head trauma and have other related symptoms. A transient ischemic attack could involve
speech disturbance, but the patient is in the wrong age group and did not have any other
symptoms of a TIA such as decreased consciousness, nausea, vomiting, visual disturbance,
or hemiplegia. Most patients who suffer TIAs do not fully recover in minutes.
Hypoglycemic attacks can cause speech alteration, but they are associated with other
symptoms such as confusion, loss of consciousness, nausea, and vomiting. These attacks
do not usually resolve spontaneously, but rather require intervention. Narcolepsy is
associated with decreased consciousness or sudden onset of a sleep state.

Reference:
Goldman L, Ausiello D: Cecil Textbook of Medicine 22nd ed., Saunders, Philadelphia,
2003 pp 2257-2269.

Marx: Rosen's Emergency Medicine: Concepts and Clinical Practice, 6th ed, 2006;
Chapter 100 Seizures Seizures caused by trauma.

American Board of Oral and Maxillofacial Surgery 43


2008 Oral and Maxillofacial Surgery Self Assessment Tool (OMSSAT)

A 25 year-old female with multiple sclerosis is undergoing surgery in the operating room under
general anesthesia. Which of the following neuromuscular blocking drugs is contraindicated in
this individual?

A. Vecuronium

B. Cisatracurium

C. Rocuronium

D. Succinylcholine

Answer: D

Rationale:
Neuromuscular blocking drugs should be used judiciously. Succinylcholine poses the risk
of hyperkalemia due to the release of intracellular potassium. Hyperkalemia after
depolarizing neuromuscular block represents a risk to all patients with muscle denervation
pathology, and may lead to cardiac arrest. Patients with lesions involving motor nuclei, as
evidenced by flaccidity, spasticity or hyperreflexia, are at risk for hyperkalemia.
Upregulation of acetylcholine receptors even by the fourth day after the onset of symptoms
leads to sensitivity to succinylcholine in multiple sclerosis patients. Patients remain at risk
for hyperkalemia for months or years and succinylcholine is best avoided. The use of
nondepolarizing muscle relaxants can also pose difficulties. In addition to their multiple
interactions with medications taken by multiple sclerosis patients, they have a variable
pharmacodynamic effect. Denervation with upregulation of acetylcholine receptors may
increase resistance to nondepolarizing neuromuscular blocking drugs. This resistance may
outlast the patients' sensitivity to succinylcholine. On the other hand, muscle weakness and
decreased muscle mass may be associated with increased sensitivity. Careful titration,
continual monitoring and use of the lowest necessary dose, represent prudent management
principles. The nerve twitch should be monitored, if possible, on an unaffected extremity so
that factors such as increased resistance to neuromuscular blocking drugs will not lead to
unintended overdoses.

Reference:
Dorotta IR, Schubert A. Multiple sclerosis and anesthetic implications. Current Opinion in
Anaesthesiology 2002, 15:365-370.

Miller's Anesthesia, 6th ed. 2005, Churchill Livingston, Chapter 13 Pharmacology of


Muscle Relaxants and their antagonists demyelinating diseases.

American Board of Oral and Maxillofacial Surgery 44


2008 Oral and Maxillofacial Surgery Self Assessment Tool (OMSSAT)

A definitive diagnosis of Alzheimers disease can only be made by which of the following tests
or exams?

A. Psychiatric examination

B. Neurologic examination

C. CNS imaging studies (CT scan, MRI, and PET scan)

D. Examination of brain tissue at autopsy

Answer: D

Rationale:
The dementing illness of Alzheimer's disease is insidious in its onset, very gradual in its
progression, and ultimately fatal. It manifests itself as changes in memory and daily
functioning and is characterized by anterograde amnesia, cognitive decline and dementia
which lead to the patient's inability to participate in self-care, disruptions in speech, and
swallowing abnormalities. Seven stages of disease progression have been developed to
better define the progression nature of the cognitive deterioration. A presumptive clinical
diagnosis of Alzheimer's disease is obtained by identifying the patient's clinical symptoms
and comparing them with a set of known criteria. Patients generally undergo complete
physical, neurologic and psychiatric examinations. An attempt is made to rule out treatable
illnesses such as Parkinson's disease, drug toxicity, metabolic diseases, dietary deficiency,
cerebrovascular accident, and CNS infective diseases (e.g. syphilis or HIV). Common
diagnostic tools include Mini Mental State Examination, CBC, serum electrolytes, liver
function tests, cholesterol, serum lipids, vitamin B12 and folate, thyroid function tests,
urinalysis, chest x-ray, ECG, EEG, and CNS imaging studies. Though each of these tests,
when taken as a whole, can lead identification of Alzheimer's disease through a process of
elimination, the diagnosis can only be made with certainty at time of the patient's death
when the brain tissue is examined for intra -cellular -amyloid plaques, extracellular -
amyloid and intracellular neurofibrillary tangles of tau protein in the entorhinal cortex,
hippocampus, basal forebrain, amygdale and parietotemporal cortex areas related to
memory, learning, language, and emotional behavior.

Reference:
Braak H, Braak E. Neuropathological staging of Alzheimer-related changes. Acta
Neuropathol 1991; 82:239.

Braak H, Braak E. Evolution of neuronal changes in the course of Alzheimers disease. J


Neural Transm Suppl 1998; 53:127.

Haaroutunian V, Perl DP, Purohid DP, et al. Regional distribution of neuritic plaques in the
nondemented elderly and subjects with very mild Alzheimer disease. Arch Neurol 1998;
55:1185

American Board of Oral and Maxillofacial Surgery 45


2008 Oral and Maxillofacial Surgery Self Assessment Tool (OMSSAT)

Walsh, DM, Selkoe DJ. Deciphering the molecular basis of memory failure in Alzheimer's
disease. Neuron 2004:44(1):181.

American Board of Oral and Maxillofacial Surgery 46


2008 Oral and Maxillofacial Surgery Self Assessment Tool (OMSSAT)

A 65 year-old male underwent general anesthesia for mandible fracture repair one day ago. His
family notes weakness of his right arm and slurring of speech over the last 30 minutes. Which of
the following studies should be initially performed?

A. Lumbar puncture

B. Carotid arteriogram

C. Carotid doppler

D. Noncontrast head CT

Answer: D

Rationale:
Brain imaging remains a required component of the emergency assessment of patients with
suspected stroke. Both CT and MRI are options for imaging the brain, but for most cases
and at most institutions, CT remains the most practical initial brain imaging test. Head CT
will determine if the cause of arm weakness and slurred speech is a CVA and whether it is
hemorrhagic or ischemic. Ischemic strokes comprise approximately 80% of strokes.
Thrombolytic therapy is ideally initiated within 3 hours of the event to decrease the long-
term disability of the individual.

Most patients with stroke do not need an examination of cerebrospinal fluid. The yield of
brain imaging is very high for detection of intracranial hemorrhage. The clinical course of
subarachnoid hemorrhage or acute central nervous system infections usually is distinct
from that of ischemic stroke. Examination of the cerebrospinal fluid may be indicated for
evaluation of a patient with a stroke that may be secondary to an infectious illness.
In addition to CT and MR angiography, transcranial Doppler ultrasonography, carotid
duplex sonography, and catheter angiography have been used to detect intracranial or
extracranial vessel abnormalities, but head CT would be the initial test for imaging.

Reference:
Kokko, J., Stein, S., The Emory University Comprehensive Board Review in Internal
Medicine. McGraw-Hill. New York, 2000, p711-712.

Adams H, et al. Guidelines for the Early Management of Adults with Ischemic Stroke;
Stroke 2007; 38: 1655.

American Board of Oral and Maxillofacial Surgery 47


2008 Oral and Maxillofacial Surgery Self Assessment Tool (OMSSAT)

A 32 year-old patient with rheumatoid arthritis presents for treatment of partially erupted tooth
#32 with pericoronitis. The patient takes methotrexate. Which medication should be avoided
in the post-operative management of this individual?

A. Hydrocodone

B. Codeine

C. Tylenol

D. Ibuprofen

Answer: D

Rationale:
Methotrexate is used for the management of rheumatoid arthritis. The mechanism of action
of methotrexate is that it is a folate antimetabolite that inhibits DNA synthesis.
Methotrexate irreversibly binds to dihydrofolate reductase, resulting in inhibition of or
purine and thymidylic acid synthesis.

Methotrexate has side-effects which include bone marrow suppression, aplastic anemia,
and GI toxicity with concomitant administration of NSAIDs. Penicillins may increase
methotrexate concentrations due to a reduction in renal tubular secretion. Tylenol and
opioids may be administered to this patient.

Severe, sometimes fatal, toxicity (including hematologic and GI toxicity) has occurred
following administration of a NSAID (e.g., indomethacin, ketoprofen) concomitantly with
methotrexate (particularly with high-dose therapy) in patients with various malignant
neoplasms, psoriasis, or rheumatoid arthritis. The toxicity was associated with elevated and
prolonged serum concentrations of methotrexate. The exact mechanism of the interaction
remains to be established, but it has been suggested that NSAIDs may inhibit renal
elimination of methotrexate, possibly by decreasing renal perfusion via inhibition of renal
prostaglandin synthesis or by competing for renal elimination.

NSAIDs should be avoided in patients receiving relatively high dosages of methotrexate


(e.g., those used in the treatment of neoplastic disease). The risk of concomitant low-dose,
intermittent (e.g., 5-15 mg weekly) methotrexate therapy and NSAIDs has not been fully
elucidated, but the drugs have been used concomitantly in many patients receiving
methotrexate for the management of rheumatoid arthritis. However, in clinical studies in
which the drugs were used concomitantly, the patients often were monitored closely and
were receiving relatively stable dosages of NSAIDs; in addition, those with conditions that
might predispose to methotrexate toxicity generally were excluded from the studies.
NSAIDs should be used with caution in patients receiving low-dose methotrexate regimens
such as those employed in the management of rheumatoid arthritis, and the possibility of
increased and prolonged serum methotrexate concentrations and resultant toxicity should

American Board of Oral and Maxillofacial Surgery 48


2008 Oral and Maxillofacial Surgery Self Assessment Tool (OMSSAT)

be considered. Although intermittent regimens also are used in the management of


psoriasis, methotrexate dosages in such regimens usually are higher than those used in the
management of rheumatoid arthritis and therefore are more likely to result in toxicity
during concomitant NSAID therapy; serious toxicity, including at least one death, has been
reported in several patients with psoriasis receiving combined therapy with the drugs.
Further study is needed to evaluate the interaction between NSAIDs and methotrexate.

Reference:
AHFS Drug Information, McEvoy, American Society of Health-System Pharmacists, Inc.,
2007, Methotrexate < Drug interactions < Non-steroidal anti-inflammatory agents
(electronic version).

Drug Information Handbook for Dentistry, 12th ed. Wynn, Meiller, Crossley. Lexi-comp,
2006, p. 1015.

American Board of Oral and Maxillofacial Surgery 49


2008 Oral and Maxillofacial Surgery Self Assessment Tool (OMSSAT)

Eye examination of a trauma patient in the emergency department reveals one pupil to be fixed
and dilated, with that same eye deviated laterally and downward, with ptosis of the eyelid.
Which of the following cranial nerves is injured?

A. Optic (II)

B. Oculomotor (III)

C. Trochlear (IV)

D. Abducens (VI)

Answer: B

Rationale:
The oculomotor nerve has been injured in this case. This nerve supplies motor function to
all of the extraocular muscles, except the lateral rectus (supplied by the abducens nerve)
and the superior oblique (supplied by the trochlear nerve). Thus the only nerves
functioning have caused a downward and lateral gaze in the affected eye. The oculomotor
nerve also supplies motor function to the levator palpebrae superioris. Loss of this function
has caused ptosis of the eyelid. Parasympathetic fibers carried by cranial nerve III from the
ciliary ganglion to the pupillary sphincter cause pupillary constriction. Injury to these
fibers causes dilation of the affected pupil.

Reference:
Seidel et al: Mosby's Guide to Physical Examination, 6th ed., Mosby Elsevier, St. Louis,
MO; 2006; p.308.

Romanes: Cunningham's Manual of Practical Anatomy, Oxford Medical Publications,


Oxford; 2006; p.113.

American Board of Oral and Maxillofacial Surgery 50


2008 Oral and Maxillofacial Surgery Self Assessment Tool (OMSSAT)

The mechanism of action of diazepam (Valium) when treating status epilepticus occurs via a
GABA mediated inhibition of what ion?

A. Calcium

B. Potassium

C. Sodium

D. Chloride

Answer: D

Rationale:
During seizures, extracellular potassium and intracellular calcium concentrations increase
and contribute to the overall excitability of the epileptic neuronal aggregate. During the
seizure itself, neurons are tonically depolarized and fire continuously in a sustained, high-
frequency discharge (corresponding to the tonic phase of the seizure). The seizure ends as
phasic repolarizations interrupt the continuous firing pattern (the cellular correlate of the
clinical clonic phase) and gradually restore membrane potentials to normal or to a
temporary hyperpolarized state (postictal depression).The benzodiazepines and barbiturates
exert their anticonvulsant effect by enhancing postsynaptic -aminobutyric acid (GABA)-
mediated inhibition through an effect on the chloride ionophore. Phenytoin and
carbamazepine are effective anticonvulsants because they produce a use-dependent block
of sodium channels, limiting the capability of neurons to fire at high-frequency rates.

Reference:
Goldman: Cecil Textbook of Medicine, 22nd ed.; 2004, WB Saunders, Chapter 426, Status
Epilepticus

Drug Information Handbook for Dentistry, 12th ed. Wynn, Meiller, Crossley. Lexi-comp,
2006, p. 455.

American Board of Oral and Maxillofacial Surgery 51


2008 Oral and Maxillofacial Surgery Self Assessment Tool (OMSSAT)

Horners syndrome results from damage to which nerve fibers?

A. Sympathetic fibers to the pupil

B. Parasympathetic fibers to the pupil

C. Optic nerve

D. Oculomotor nerve

Answer: A

Rationale:
A unilateral small pupil is commonly due to underactivity of the ipsilateral sympathetic
pathways. Miosis is commonly associated with ptosis (lip droop) due to sympathetic
denervation of the tarsal muscle and facial anhidrosis (loss of sweating). This combination
is known as Horner's syndrome. It may be due to a lesion of the hypothalamus, brain stem,
cervical spinal cord, or sympathetic fibers to the pupil. Horner's syndrome may be the first
sign of an apical lung tumor (Pancoast tumor) or may occur in diseases affecting the carotid
artery. The Pancoast tumor causes compression on the spinal cord at C8-T1. The
sympathetic fibers to the superior cervical ganglion arise from the T1 level.

Reference:
Andeoli, Carpenter, et al. Cecil Essentials of Medicine, 7th ed. Saunders Elsevier,
Philadelphia, 2007 p. 1080

Goetz: Textbook of Clinical Neurology, 2nd ed. Saunders Elsevier 2003, Chapter 21
Autonomic nervous system < Pupils < Horner's syndrome, and Chapter 15 Strength and
Reflexes > Lower Motor Neuron Syndromes

American Board of Oral and Maxillofacial Surgery 52


2008 Oral and Maxillofacial Surgery Self Assessment Tool (OMSSAT)

Which of the following are the vitamin-K dependent clotting factors?

A. III, VII, IX, X

B. II, VII, IX, X

C. III, VIII, IX, XII

D. II, VIII, IX, XII

Answer: B

Rationale:
There are 5 factors produced in the liver: II, V, VII, IX, and X. Of these, four are vitamin-
K dependent II, VII, IX, and X. Vitamin K is required for gamma-carboxylation of the
factors. Virtually all clotting factors are produced in the liver. Factor VIII is made in
several tissues, including the liver as well as the glomerular and tubular epithelial cells in
the kidney. Warfarin interferes with hepatic synthesis of vitamin K-dependent clotting
factors. The PT (Prothrombin time)/INR (International normalized ratio) are both
measures of the extrinsic pathway in the clotting cascade.

Reference:
Abubaker AO: Oral and Maxillofacial Surgery Secrets. Hanley & Belfus, Inc.,
Philadelphia, PA, 2001; Page 130

Andeoli, Carpenter, et al. Cecil Essentials of Medicine, 7th ed. Saunders Elsevier,
Philadelphia, 2007 p. 532-538

Tierney LM: 2005 Curent Medical Diagnosis and Treatment. McGraw-Hill, Inc, New
York, New York; 2005; Page 656

American Board of Oral and Maxillofacial Surgery 53


2008 Oral and Maxillofacial Surgery Self Assessment Tool (OMSSAT)

What is the etiology of thrombocytopenia associated with liver disease?

A. Reduced urea synthesis

B. Functional renal failure

C. Splenomegaly

D. Ascites

Answer: C

Rationale:
Portal hypertension is a common sequelae of chronic liver failure. Vascular congestion
within the portal system leads to splenomegaly. An enlarged spleen can sequester red
blood cells and platelets. Sequestration of platelets by the spleen will lead to
thrombocytopenia.

Reduced urea synthesis does not lead to thrombocytopenia. Functional renal failure, also
known as hepatorenal syndrome is a late complication of cirrhosis. It occurs in the
presence of liver failure without intrinsic kidney abnormalities. Ascites is an accumulation
of excess fluid in the abdomen and is caused by cirrhosis in at lest 80% of patients.
Complications of ascites include spontaneous bacterial peritonitis, hydrothorax (passage of
ascitic fluid into the pleural space) and abdominal wall hernias.

Reference:
Oral & Maxillofacial Surgery Knowledge Update. Vol 1, Part II, 1995, Patient Evaluation,
p. 39.

Mercer KW, Gail Macik B, Williams ME. Hematologic disorders in critically ill patients.
Semin Respir Crit Care Med. 2006 Jun; 27:286-96.

Goldman: Cecil Textbook of Medicine, 22nd ed, Chapter 157, Cirrhosis and its sequelae <
Complications of Cirrhosis.

American Board of Oral and Maxillofacial Surgery 54


2008 Oral and Maxillofacial Surgery Self Assessment Tool (OMSSAT)

Which of the following is a component of the modified Child-Pugh classification?

A. Albumin

B. Lactate dehydrogenase

C. Partial thromboplastin time

D. Ammonia

Answer: A

Rationale:
Modified Child-Pugh classification was designed to indicate the severity of cirrhosis in
patients with chronic liver disease. The components of the classification include: albumin,
bilirubin, PT, INR, ascites, and encephalopathy. PTT is not a component of the Child-Pugh
classification. PTT is used to measure the intrinsic pathway. PT is used as a measure of
the extrinsic pathway. Ammonia is detoxified into urea by the liver. Increased levels of
ammonia are thought to contribute to hepatic encephalopathy. Lactate dehydrogenase
levels can be elevated in hepatitis.

Modified Child-Pugh scoring system


Modified Child-Pugh Score *
Parameters 1 2 3
Albumin (g/dL) >3.5 1.83.5 <2.8
Prothrombin time
Seconds prolonged <4 46 >6
International normalized ratio <1.7 1.72.3 >2.3
Bilirubin (mg/dL) <2 23 >3
Ascites Absent Slight-moderate Tense
Encephalopathy None Grade III Grade IIIIV
From Pugh RNH, Murray-Lyon IM, Dawson JL, et al: Transection of oesophagus for
bleeding of oesophageal varices. Br J Surg 60:646649, 1973.
* Class A = 5 to 6 points, B = 7 to 9 points, and C = 10 to 15 points.

Reference:
Martins A, Cortez-Pinto H, Marques-Vidal P, Mendes N, et al. Treatment and prognostic
factors in patients with hepatocellular carcinoma. Liver Int. 2006 Aug; 26(6):680-7.

Miller: Miller's Anesthesia, 6th ed, Chapter 55 Anesthesia and the Hepatobiliary System
< Cirrhosis as a perioperative risk factor.

American Board of Oral and Maxillofacial Surgery 55


2008 Oral and Maxillofacial Surgery Self Assessment Tool (OMSSAT)

Detection of only Hepatitis B surface antibody (Anti-HBsAg) in the serum is associated with
which of the following?

A. Chronic infection

B. Acute infection

C. Immunity

D. Initial cellular response

Answer: C

Rationale:
Hepatitis B surface antigen encodes the viral envelope of the virus. Anti-HBsAg is the
antibody formed against this antigen and its presence confers immunity when the titer is >
10mIU/ml. The following table outlines the differences in immunity, acute, and chronic
infectious states of Hepatitis B in regards to the serologic markers.

Serologic marker Interpretation

Hepatitis Total Immunoglobulin M Antibodies to


B surface antibodies to antibodies to hepatitis B
antigen hepatitis B hepatitis B core surface
core antigen antigen antigen

Susceptible; never infected

+ Early acute infection; transient


(21 days) after vaccination

+ + + Acute infection

+ + Acute resolving infection

+ + Past infection; recovered and


immune

+ + Chronic infection

+ False-positive (i.e., susceptible);


past infection; "low-level"
chronic infection; passive
transfer to an infant born to a
mother who is positive for
hepatitis B surface antigen

American Board of Oral and Maxillofacial Surgery 56


2008 Oral and Maxillofacial Surgery Self Assessment Tool (OMSSAT)

+ Immune if titer is >10 mIU/ml

Reference:
Oral and Maxillofacial Surgery Clinics of North America, Volume 18, Number 2, May
2006, p. 217-219

Shepard CW, Simard EP, Finelli L, Fiore AE, Bell BP. Hepatitis B virus infection:
epidemiology and vaccination. Epidemiol Rev. 2006; 28(1):112-25. 2006

American Board of Oral and Maxillofacial Surgery 57


2008 Oral and Maxillofacial Surgery Self Assessment Tool (OMSSAT)

First line treatment for primary peptic ulcer disease would be:

A. H2 blockers and antacids.

B. mucosal protectants and antibiotics.

C. proton pump inhibitors and H2 blockers.

D. proton pump inhibitor and antibiotics.

Answer: D

Rationale:
Peptic ulcers are called primary when they occur without any predisposing factors such as
acute medical illness trauma, use of NSAIDS, alcohol, or smoking. Helicobacter pylori is
recognized as an important factor in the pathogenesis of primary peptic ulcer disease.
Inhibitors of gastric acid secretion and antimicrobial agents are recommended. This
combination therapy is first line treatment because antisecretory drugs provide rapid relief
of pain and accelerate healing; while antibiotics eradicate Helicobacter pylori.

In the United States, perhaps 80% of patients who have duodenal ulcers are infected with
the organism, as are more than 60% of those with gastric ulcers.

Reference:
Perioperative Management of the Oral and Maxillofacial Surgery Patient, Part II, Oral and
Maxillofacial Surgery Clinics of North America, Vol. 18, No. 2, May 2006,
pp. 245-246
Dental Management of the Medically Compromised Patient, 6th Ed., 2002, Little, J.W.,
Falace, D.A., Miller, C.S., Rhodus, N.L., pp. 192-193

Feldman: Sleisenger & Fordtran's Gastrointestinal and Liver Disease, 8th ed., Saunders,
2006, p.1094.

American Board of Oral and Maxillofacial Surgery 58


2008 Oral and Maxillofacial Surgery Self Assessment Tool (OMSSAT)

Barretts metaplasia is a complication associated with which of the following?

A. Chronic ulcerative colitis

B. Peptic ulcer

C. Gastroesophageal reflux

D. Crohns disease

Answer: C

Rationale:
Barrett's esophagus is not uncommon, and is found in approximately 6% to 12% of patients
undergoing endoscopy for symptomatic GERD and in 1% to 2% of unselected patients
undergoing endoscopy.

Barrett's esophagus occurs when the squamous epithelium of the esophagus is replaced by
metaplastic columnar epithelium. It develops due to chronic reflux-induced injury to the
esophageal squamous epithelium. These patients often have severe GERD, usually with
chronic reflux symptoms for more than 10 years, poor esophageal motility, large hiatal
hernias, and extensive acid and bile reflux. Patients with long-segment Barrett's esophagus
have an estimated 30 to 125 times increased risk of developing esophageal cancer
compared with the general population. Epidemiologic data suggest that the mean interval
from developing Barrett's esophagus to evolution to cancer may be 20 to 30 years.

The appropriate surveillance interval for patients with Barrett's esophagus has not been
studied prospectively. However, current programs, such as those proposed by the American
College of Gastroenterology, are based on the grade of dysplasia. Individuals with high-
grade dysplasia may undergo endoscopy every 3 months, while those with low grade
dysplasia may be followed yearly.

Reference:
Current Medical Diagnosis and Treatment, 42nd Ed., 2003, Tierney, L.M., McPhee, S.J.,
Papadakis, M.A., pp. 552-553

Feldman: Sleisenger and Fordtran's Gastrointestinal and Liver Disease, 8th ed., Saunders,
2006, 921-929.

American Board of Oral and Maxillofacial Surgery 59


2008 Oral and Maxillofacial Surgery Self Assessment Tool (OMSSAT)

Which of the following differentiates type I from type II diabetes mellitus?

A. Ketoacidosis does not occur in type II DM

B. There is a stronger familial link with type I DM than with type II DM

C. Insulin use is only indicated in the management of type I DM

D. Diabetic neuropathy is primarily a complication of type II DM

Answer: A

Rationale:
Since insulin production continues to be produced in individuals with type II DM,
ketoacidosis does not occur though insulin may ultimately be required to control
hyperglycemia. Genetic tendencies towards developing diabetes are associated with both
types, but the linkage is more prominent in type II DM where there is concordance rate of
greater than 90% between identical twins. Those with type II DM often progress to the
need for exogenous insulin to control hyperglycemia. Diabetic neuropathy is a frequent
complication of both types of DM.

Diabetic ketoacidosis (DKA) is a syndrome in which insulin deficiency and glucagon


excess combine to produce a hyperglycemic, dehydrated, acidotic patient with profound
electrolyte imbalance. Glucose in the renal tubules draws water, sodium, potassium,
magnesium, calcium, phosphorus, and other ions from the circulation into the urine. This
osmotic diuresis combined with poor intake and vomiting produces the profound
dehydration and electrolyte imbalance associated with DKA.

In uncontrolled type II DM, the patient develops a hyperosmolar nonketotic state.


Decreased insulin action results in glycogenolysis, gluconeogenesis, and decreased
peripheral uptake of glucose. The hyperglycemia pulls fluid from the intracellular space
into the extracellular space, transiently maintaining adequate perfusion. Soon, however,
this fluid is lost in a profound osmotic diuresis, limited finally by hypotension and a
subsequent drop in the glomerular filtration rate (GFR). The urine is extremely hypotonic,
with urine sodium concentration between 50 and 70 mEq/L, compared with 140 mEq/L in
extracellular fluid. This hypotonic diuresis produces profound dehydration, leading to
hyperglycemia, hypernatremia, and associated hypertonicity.

Reference:
Baker, Maurer, Wauman; Perioperative management of diabetes mellitus, Oral and
Maxillofacial Clinics of North America: 10:3:363-371; W.B.Saunders, Philadelphia, 1998

Powers: C.323, Diabetes mellitus, Harrison's Principles of Internal Medicine, 16th Edit.,
p.2169-2185; McGraw-Hill, New York, 2005

American Board of Oral and Maxillofacial Surgery 60


2008 Oral and Maxillofacial Surgery Self Assessment Tool (OMSSAT)

Marx: Rosen's Emergency Medicine: Concepts and Clinical Practice, 6th ed., 2006 Mosby,
Inc., p. 1962-1967.

American Board of Oral and Maxillofacial Surgery 61


2008 Oral and Maxillofacial Surgery Self Assessment Tool (OMSSAT)

Cushings syndrome is most commonly due to which of the following?

A. A functioning pituitary adenoma

B. Adrenal carcinoma

C. Exogenous steroid administration

D. Adrenal hyperplasia

Answer: A

Rationale:
Cushing's syndrome is due to chronic exogenous or endogenous glucocorticoid excess.
The most common cause of Cushing's syndrome is a functioning pituitary adenoma (termed
Cushing's disease). Approximately 80% of ACTH-dependent cases are due to this
etiology. Cushing's Syndrome resulting from endogenous sources may be ACTH-
dependent or ACTH-independent. Ectopic ACTH syndrome accounts for about 10% of
cases. Examples of ACTH-independent endogenous sources are: pituitary adenoma, small-
cell lung carcinoma, bronchial carcinoma. Cushing's Syndrome can also be due to
exogenous iatrogenic steroid use.

Signs and symptoms of Cushing's syndrome include: centripetal obesity, reproductive


dysfunction, psychiatric abnormalities, osteoporotic changes leading to pathologic
fractures, thinning of the skin, myopathy and bruising, hypertension, infections are
common, glucose intolerance with diabetes, electrolyte imbalance, water retention, and
increased intraocular pressure and exophthalmos.

Reference:
Andreoli T: Cecil Essentials of Medicine., Mosby, Philadelphia, 2001 pp569-570

Henderson K, Baranski T, Bickel P: The Washington Manual Endocrinology Subspecialty


Consult., Lippincott Williams & Wilkins, Philadelphia, 2005 pp76-77

Larsen: Williams Textbook of Endocrinology, 10th ed. 2003, p. 508-525.

American Board of Oral and Maxillofacial Surgery 62


2008 Oral and Maxillofacial Surgery Self Assessment Tool (OMSSAT)

The physical findings in a patient with untreated Addisons disease include:

A. skin striae.

B. hyperpigmentation of oral mucosa and skin.

C. centripetal obesity.

D. acne.

Answer: B

Rationale:
Primary adrenocortical insufficiency is known as Addison's disease. The cause is most
often the result of autoimmune destruction of the adrenal glands. The loss of ACTH
secretion seen in secondary adrenocortical insufficiency is due to pituitary suppression
resulting from long term use of exogenous glucocorticoids. Skin striae, centripetal obesity,
and adult onset acne are seen in patients diagnosed with Cushing's Syndrome
(adrenocortical excess). Hyperpigmentation of oral mucosa and skin is seen in patients
with primary adrenocortical deficiency (Addison's disease) because ACTH continues to be
produced by the normally functioning pituitary gland. ACTH directly activates
melanocortin receptors resulting in pigmentation of mucosa and skin. Other signs and
symptoms frequently seen in patient's with adrenocortical insufficiency (primary or
secondary) include; nausea, vomiting, anorexia, weight loss, and fatigue.

Adrenal crisis is characterized by: dehydration, hypotension or shock, nausea/vomiting,


severe abdominal pain, unexplained hypoglycemia and fever, hyperpigmentation,
electrolyte abnormalities (hyponatremia, hyperkalemia, azotemia, hypercalcemia), and
hypothyroidism.

Adrenal crisis is precipitated by stress. Acute adrenal insufficiency is a life-threatening


emergency, and treatment should not be delayed while waiting for definitive proof of
diagnosis. However, in addition to measurement of plasma electrolytes and blood glucose,
appropriate samples for ACTH and cortisol should be taken before giving corticosteroid
therapy. If the patient is not critically ill, an acute ACTH stimulation test can be performed.

Emergency measures for treatment of acute adrenal insufficiency include the establishment
of IV access, drawing blood for serum electrolytes and glucose, as well as measurement of
plasma cortisol and ACTH. An infusion of 0.9% normal saline, with or without 5%
dextrose is started. Hydrocortisone 100 mg is administered and repeated every 6 hours.

Prevention of an adrenal crisis in a minor illness or stressful situation is to increase the


glucocorticoid dose two- to three-fold for the few days of the illness. For minor procedures
under local anesthesia, no extra supplementation is needed. For moderately stressful

American Board of Oral and Maxillofacial Surgery 63


2008 Oral and Maxillofacial Surgery Self Assessment Tool (OMSSAT)

procedures, a single 100 mg intravenous dose of hydrocortisone just before the procedure is
given.
For major surgery, give hydrocortisone 100 mg intravenously just before induction of
anesthesia and continue every 8 hr for first 24 hr. Taper dose rapidly, decreasing by half
per day, to maintenance level.

Reference:
Greenspan F, Gardener D: Basic & Clinical Endocrinology., Lange Medical
Books/McGraw-Hill, New York, 2004 pp384-386

Henderson K, Baranski T, Bickel P: The Washington Manual Endocrinology Subspecialty


Consult., Lippincott Williams & Wilkins, Philadelphia, 2005 pp61-66

Larsen: Williams Textbook of Endocrinology, 10th ed. 2003, p. 525-527.

American Board of Oral and Maxillofacial Surgery 64


2008 Oral and Maxillofacial Surgery Self Assessment Tool (OMSSAT)

In the table below, which set of laboratory values would most commonly be noted in the primary
hyperthyroid patient?

Answer: B

Rationale:
The patient with primary hyperthyroidism has elevations of both circulating T3 and T4
from thyroid gland hyperplasia, nodule, or adenoma/adenocarcinoma. This elevation of
circulating hormones exerts an inhibitory influence on the release of TSH by the anterior
pituitary gland, via a negative feedback mechanism. By extension, the hypothyroid patient
tends to have decreased levels of circulating T3 and T4, with concomitant elevation of the
TSH level. An additional test which is of use is the Resin T3 Uptake test. The RT3U test
measures the level of thyroid hormone-binding proteins in the blood. It is typically
elevated in the hyperthyroid patient, and decreased in the hypothyroid patient.
If, however, there is a lesion in the pituitary gland, e.g., hyperplasia or adenoma, then TSH
is oversecreted. Because TSH is elevated, T3 and T4 secretion is elevated. This is
secondary hyperthyroidism (secondary to pituitary disease).
In most individuals with hypothyroidism, serum TSH results are clearly elevated, but
results may be inappropriately normal for the level of T4 and T3 in those with pituitary or
hypothalamic disorders.

Reference:
Braverman LE, Utiger RD: Werner and Ingbar's the Thyroid: A Fundamental and Clinical
Text. 7th ed. 2005, pp 197-199, 336-341.

American Board of Oral and Maxillofacial Surgery 65


2008 Oral and Maxillofacial Surgery Self Assessment Tool (OMSSAT)

Warofsky L, Ingbar SH: Diseases of the thyroid. In: Wilson JD, Brunwald E, et al, eds.
Harrison's Principles of Internal Medicine. McGraw-Hill; 1991:pp. 1692-1712.

McPherson & Pincus: Henry's Clinical Diagnosis and Management by Laboratory


Methods, 21st ed., pp. 333-341. 2006 W. B. Saunders Company

American Board of Oral and Maxillofacial Surgery 66


2008 Oral and Maxillofacial Surgery Self Assessment Tool (OMSSAT)

A 9 year-old male presents with oral ulcerations and generalized erythematous gingival
enlargement. A review of systems elicits a history of abdominal pain and intermittent diarrhea.
An endoscopic evaluation showed ulcerations in the ileum and cecum separated by areas of
normal appearing mucosa. This clinical presentation is most consistent with:

A. Crohns disease.

B. Wegeners granulomatosis.

C. ulcerative colitis.

D. irritable bowel syndrome.

Answer: A

Rationale:
Crohn's disease is an inflammatory bowel disease that can affect young adults and present
with oral lesions such as aphthous ulcers of the buccal mucosa, lips or gingiva.
Generalized gingival enlargement has also been described. Wegener's granulomatosis can
also present with oral lesions but is best described as a granulomatous vasculitis affecting
primarily the upper respiratory tract, lungs, and kidneys. Endoscopic findings of bowel
ulcerations separated by areas of normal mucosa or skip lesions are common in Crohn's
disease and are not characteristic of ulcerative colitis. Kaposi's sarcoma is a vascular
neoplasm seen often in patients with AIDS and oral lesions appear as red-blue macules
which progress to bluish nodules.

Reference:
Shiboski C.H., Winkler J.R., Gingival Kaposi's sarcoma and periodontitis. A case report
and suggested treatment approach to the combined lesions. Oral Surg Oral Med Oral Pathol
(1993) 76: pp 49-53.

Goldman: Cecil Textbook of Medicine.23nd. Edition, 2007, p. 1042-1050

Stricker T., Braegger C.P., Images in clinical medicine. Oral manifestations of Crohn's
disease. N Engl Med (2000) 342: pp 1644-.

Knight J.M., Hayduk M.J., Summerlin D.J., Mirowski G.W., Strawberry gingival
hyperplasia: a pathognomonic mucocutaneous finding in Wegener granulomatosis. Arch
Dermatol (2000) 136: pp 171-173.

American Board of Oral and Maxillofacial Surgery 67


2008 Oral and Maxillofacial Surgery Self Assessment Tool (OMSSAT)

A 25 year-old female with progressive mandibular prognathism presents with facial features,
widely spaced teeth and bulbous finger tips. Which of the following is the most likely cause?

A. Hyperthyroidism

B. Pheochromocytoma

C. Chronic hypercalcemia

D. Pituitary adenoma

Answer: D

Rationale:
This patient has findings highly suggestive of acromegaly, the result of excess levels of
circulating growth hormone (GH). GH exhibits anti-insulin actions and about 10 % of those
with acromegaly develop diabetes mellitus. GH levels should be measured. A GH level
greater than 10 ng/ml favors a diagnosis of acromegaly. If GH is not suppressed below 5
ng/ml one to two hours after ingestion of 100 g of glucose, the diagnosis of acromegaly is
more conclusive. The cause of acromegaly is often a pituitary adenoma which secretes GH.
Often the adenoma is large enough to cause distortion of the sella turcica and may be
evident on lateral cephalometric radiographs. Hypercalcemia does not cause the facies
described. Hyperthyroidism does not cause increased levels of GH. Pheochromocytoma
has only rarely been known to express growth hormone releasing factor (GHRF) activity
causing acromegaly.

Reference:
Melmed, Jameson: C.318, Disorders of the anterior pituitary and hypothalamus, Harrison's
Principles of Medicine, 16th Edit; p.2070, McGraw-Hill, New York 2005

Larsen: Williams Textbook of Endocrinology, 10th ed, Saunders, 2003, p. 230-243.

American Board of Oral and Maxillofacial Surgery 68


2008 Oral and Maxillofacial Surgery Self Assessment Tool (OMSSAT)

Diabetic neuropathy can directly lead to which of the following conditions?

A. Postural hypotension

B. Resting bradycardia

C. Arrhythmias

D. Cardiomyopathy

Answer: A

Rationale:
Diabetic neuropathy is not a single entity but a number of different syndromes, ranging
from subclinical to clinical manifestations depending on the classes of nerve fibers
involved.
Diabetic neuropathy can cause postural hypotension. Patients with type 2 diabetes mellitus
and orthostatic hypotension are hypovolemic and have sympathoadrenal insufficiency.
Both factors contribute to the pathogenesis of orthostatic hypotension. Postural
hypotension in the patient with diabetic autonomic neuropathy can present a difficult
management problem. Elevating the blood pressure in the standing position must be
balanced against preventing hypertension in the supine position.

It does not directly cause bradycardia, arrhythmias or cardiomyopathy unless brought about
indirectly by ischemic coronary athlerosclerosis. Diabetic neuropathy can also result in
dysphagia, gastroparesis, constipation, and abnormal sweating.

A constellation of metabolic derangements that are frequently seen in patients with insulin
resistance and type 2 diabetes are individually associated with an increased risk of
cardiovascular disease. These patients have been variously designated as having syndrome
X; the dysmetabolic syndrome; hypertension, obesity, noninsulin-dependent diabetes
mellitus (NIDDM), dyslipidemia, and atherosclerotic cardiovascular disease (HONDA); or
the "deadly quartet." Myocardial infarction, stroke, and nonischemic cardiovascular
disease are the cause of death in up to 80% of individuals with type 2 diabetes. Independent
of other risk factors, type 2 diabetes increases the risk for cardiovascular morbidity and
mortality but also provides a synergistic interaction with other risk factors such as smoking,
hypertension, and dyslipidemia.

Reference:
Powers: Diabetes mellitus, Harrison's Principles of Internal Medicine, 16th Edit. C.323,
p.2165-2166; McGraw-Hill, New York, 2005

American Board of Oral and Maxillofacial Surgery 69


2008 Oral and Maxillofacial Surgery Self Assessment Tool (OMSSAT)

Baker, Maurer, Warman: Perioperative management of diabetes mellitus, Oral and


Maxillofacial Surgery Clinics of North America, 10:3:363-371; W.B.Saunders,
Philadelphia, 1998

Larsen: Williams Textbook of Endocrinology, 10th ed., 2003, Saunders, p. 1440-1441,


1553-1554.

American Board of Oral and Maxillofacial Surgery 70


2008 Oral and Maxillofacial Surgery Self Assessment Tool (OMSSAT)

Intense dilute diuresis following head trauma is caused by:

A. increased vasopressin production.

B. decreased vasopressin production.

C. increased renin production.

D. decreased renin production.

Answer: B

Rationale:
Trauma to the hypothalamus produces a decrease in vasopressin synthesis and secretion. A
normal kidney will attempt to reabsorb or excrete solute-free water to preserve a normal
plasma osmolality of 275 to 290 mOsm/kg. The primary hormone regulating plasma
osmolality is arginine vasopressin. It is synthesized in the hypothalamus and released into
the systemic circulation by means of the posterior pituitary gland. Despite wide fluctuations
in water and sodium intake, the body normally can maintain serum osmolality in a narrow
range (275 to 290 mOsm/kg). Osmoreceptors near the hypothalamus sense plasma
osmolality and modulate vasopressin release. Vasopressin functions at the distal collecting
duct of the kidney to increase water reabsorption in this otherwise relatively water-
impermeable section of the nephron. In hypo-osmolar conditions for instance, vasopressin
levels fall to a low basal rate to reabsorb less free water, resulting in more dilute urine.

The situation described is consistent with diabetes insipidus, where there is decreased
release of vasopressin, also known as antidiuretic hormone (ADH). Patients with
diabetes insipidus and inadequate thirst can rapidly become dehydrated and may
experience severe hypernatremia with devastating effects on the CNS. Hypertonic
encephalopathy with obtundation, coma, and seizures may be produced by brain
shrinkage. A decreased volume of brain in the skull may lead to subarachnoid
hemorrhage, intracerebral bleeding, or petechial hemorrhage.

Because a head trauma patient must be given fluids parenterally, some clinicians prefer
to use a continuous infusion of low-dose vasopressin. The vasopressin can either be
added directly to the crystalloid solution that is being administered or infused separately
to maintain a constant antidiuresis while fluid intake is adjusted appropriately to any
persistent polyuria and to cover insensible water loss. Doses of 0.25 to 2.7 mU/kg per
hour have been described. With this method, there is a potential to produce
hyponatremia, and serum sodium levels must be checked regularly.

SIADH is produced when plasma levels of arginine vasopressin are elevated at times
during which the physiologic secretion of vasopressin from the posterior pituitary would
normally be suppressed. Because the clinical abnormality is a decrease in the osmotic
pressure of body fluids, the hallmark of SIADH is hypo-osmolality, with euvolemia.
Clinical criteria for diagnosis include: Plasma osmolality < 275mOsm/kg, urine

American Board of Oral and Maxillofacial Surgery 71


2008 Oral and Maxillofacial Surgery Self Assessment Tool (OMSSAT)

osmolality > 100 mOsm/kg with normal renal function, clinical euvolemia, elevated
urinary sodium excretion with normal salt and water intake, absence of other causes
(hypothyroidism, addisons disease, diuretic use, pituitary ACTH insufficiency).

The pathophysiology of primary polydipsia is essentially the reverse of that in central DI:
the excessive intake of water expands and slightly dilutes body fluids, suppresses ADH
secretion, and dilutes the urine. The resultant increase in the rate of water excretion
balances the increase in intake, and the osmolality of body water stabilizes at a new,
slightly lower level that approximates the osmotic threshold for ADH secretion. Primary
polydipsia can be produced by drugs that cause a dry mouth, or by any peripheral disorder
causing pathologic elevations of renin or angiotensin. Primary polydipsia is often due to a
severe mental illness such as schizophrenia, mania, or an obsessive-compulsive disorder, in
which case it is called psychogenic polydipsia.

Reference:
Zimmerman: Systemic diseases, Merrit's Textbook of Neurolgy, 8th Edit., p.825-826; Lea
& Febiger, Philadelphia, 1989

Brenner & Rector's The Kidney, 7th ed., 2004, Saunders, p.880-884.

American Board of Oral and Maxillofacial Surgery 72


2008 Oral and Maxillofacial Surgery Self Assessment Tool (OMSSAT)

An individual diagnosed with Cushings disease would demonstrate which abnormality?

A. Hypovolemia

B. Hyperkalemia

C. Hyperglycemia

D. Hypotension

Answer: C

Rationale:
Cushing's disease involves glucocorticoid excess resulting specifically from a pituitary
adenoma. Physical and biochemical abnormalities result from a state of prolonged
hypercortisolism. Metabolic abnormalities associated with hypercortisolism include;
glucose intolerance, hyperglycemia, hypokalemia, and hypertension. Hypovolemia and
hypotension may be seen in cortisol deficiency states (such as Addison's disease or
pituitary dysfunction); but not, however, in a patient diagnosed with Cushing's disease.

Reference:
Endocrine Physiology, Mosby, Philadelphia, 2001 pp144-145

Andreoli T: Cecil Essentials of Medicine., W. B. Saunders Company, Philadelphia, 2001


pp570-571

American Board of Oral and Maxillofacial Surgery 73


2008 Oral and Maxillofacial Surgery Self Assessment Tool (OMSSAT)

During extraction of erupted premolars to facilitate orthodontics, which tooth is most likely to
have a root fracture during extraction?

A. Maxillary first premolar

B. Maxillary second premolar

C. Mandibular first premolar

D. Maxillary second premolar

Answer: A

Rationale:
The maxillary first premolar root usually bifurcates into buccal and palatal roots which are
often thin and slightly curved. This predisposes this premolar to root fractures with
extraction. Maxillary second premolars and mandibular first and second premolars usually
have single roots.

Reference:
Peterson, LJ, Ellis, E, Hupp, JR, Tucker, MR. Contemporary Oral and Maxillofacial
Surgery, Mosby, St. Louis, 1988, p. 169.

Fonseca, et al. Oral and Maxillofacial Surgery: Anesthesia/Dentoalveolar Surgery/Office


Management. (Vol 1). WB Saunders Company. Philadelphia. 2000. p. 221.

American Board of Oral and Maxillofacial Surgery 74


2008 Oral and Maxillofacial Surgery Self Assessment Tool (OMSSAT)

Which of the following factors would be most important in deciding to remove a 2mm fractured
root tip of a maxillary molar?

A. Close proximity of the root tip to the floor of the maxillary sinus

B. Patients age less than 50

C. Operator skill and experience

D. Presence of periapical pathology with the root tip

Answer: D

Rationale:
In general, roots with periapical lesions should be removed whenever possible, even if they
are near the maxillary sinus floor. The patient's age less than 50 should not be a factor in
the decision. All OMSs should possess adequate skill and experience to remove root tips.

Reference:
Berman, SA, Basic Principles of Dentoalveolar Surgery in Principles of Oral and
maxillofacial Surgery, Peterson, LJ, Indresan, AT, Marciani, RD, Roser, SM (eds), Vol 1,
Lippincott, Philadelphia, 1992, p.94.

Fonseca, et al. Oral and Maxillofacial Surgery: Anesthesia/Dentoalveolar Surgery/Office


Management. (Vol 1). WB Saunders Company. Philadelphia. 2000. p. 432.

American Board of Oral and Maxillofacial Surgery 75


2008 Oral and Maxillofacial Surgery Self Assessment Tool (OMSSAT)

Which of the following conditions are most likely to be associated with an asymptomatic erupted
mandibular third molar in a young adult?

A. Resorption of the distal root of the adjacent tooth

B. Dental caries

C. Loss of periodontal support

D. Dentigerous cysts

Answer: C

Rationale:
The most common problems associated with retained third molars are loss of periodontal
support on the adjacent second molar and pericoronitis.

Numerous studies have documented the presence of periodontal pathogens and loss of
periodontal support at the distal of the second molar and third molar. This condition has
been shown to progress with age. Acute pericoronitis is also a relatively common finding
and if third molars are not removed, the condition is likely to recur and / or occur with
another third molar. 60% of patients with pericoronitis experienced symptoms associated
with the contralateral third molar within the previous 12 months.

Other types of pathology such as resorption of adjacent tooth roots or odontogenic cysts are
less common.

Reference:
Marciani R and White RP, Establishing disease and clinical outcomes in asymptomatic
third molars Oral and Maxillofacial Surgery Knowledge Update Vol IV. ALV 7, 12, 1996.

Marciani R, Third molar removal: an overview of indications, imaging, evaluation, and


assessment of risk. Oral and Maxillofacial Surgery Clinics of North America Vol 19(1), 1-
3, 2007.

Ash M, Costich ER, Hayward JR: A study of periodontal hazards of third molars. J
Periodontology, 33:209, 1962.

American Board of Oral and Maxillofacial Surgery 76


2008 Oral and Maxillofacial Surgery Self Assessment Tool (OMSSAT)

Which of the following is an indication to perform a sulcular incision instead of a scalloped


mucogingival junction incision when performing periapical surgery on a maxillary incisor?

A. Presence of a short root

B. Preserving anterior gingival esthetics

C. Avoidance of releasing incisions

D. Eliminate the need for suturing

Answer: A

Rationale:
Typically, in the anterior region where esthetics is a concern, a scalloped submarginal
incision is preferred. However, contraindications to this approach are periodontal
breakdown, a large periapical lesion, and a short root. In these cases, a full thickness
sulcular incision with one or two releasing incisions is preferred. A sulcular incision may
also used to avoid placing an incision over an underlying bony defect.

Reference:
Fonseca, et al. Oral and Maxillofacial Surgery: Anesthesia/Dentoalveolar Surgery/Office
Management. (Vol 1). WB Saunders Company. Philadelphia. 2000, pp 318-9.

Peterson, LJ, Ellis,E, Hupp,JR, Tucker, MR. Contemporary Oral and Maxillofacial
Surgery 4th edition, Mosby, St. Louis, 1998, pp. 385-394.

American Board of Oral and Maxillofacial Surgery 77


2008 Oral and Maxillofacial Surgery Self Assessment Tool (OMSSAT)

Which of the following factors are associated with a favorable outcome when uprighting second
molars?

A. Uprighting involving an arc of rotation of greater than 90 degrees

B. Incomplete vertical growth of the mandible

C. The need to correct the bucco-lingual position of the tooth

D. Second molar root formation is 2/3 complete

Answer: D

Rationale:
Incomplete root formation is a favorable factor when repositioning a tooth. All the other
factors listed would increase the difficulty of uprighting the second molar. The procedure
is best performed after 2/3 of root development is completed. At this stage the risk of root
fracture is minimal. Performing this procedure when less than 2/3 of root development has
been completed could result in the second molar floating in its new position. Although the
procedure has been performed when root development is complete, the incidence of
subsequent pulpal necrosis or calcification is increased.

Reference:
Fonseca, et al. Oral and Maxillofacial Surgery: Anesthesia/Dentoalveolar Surgery/Office
Management. (Vol 1). W.B. Saunders Company. Philadelphia. 2000. p308-316.

Dentoalveolar Surgery in Peterson's Principles of Oral and Maxillofacial Surgery, Miloro,


M. et al., Editors, Second Edition, BC Decker Inc, 2004, pp. 135

American Board of Oral and Maxillofacial Surgery 78


2008 Oral and Maxillofacial Surgery Self Assessment Tool (OMSSAT)

The CT scans below are of a patient who presented to the emergency department with a history
of instantaneous painless swelling during surgical extraction of an erupted maxillary molar by a
general dentist. What is the most appropriate management?

A. Observation

B. Transcutaneous puncture aspiration

C. Arteriogram & selective embolization

D. Surgical exploration

Answer: A

Rationale:
The CT scan depicts air emphysema. Although hemorrhage should be included in the
differential diagnosis, the scan clearly depicts air and not blood. Most cases of surgical
emphysema following dental treatment (72%) involved the use of high-speed air-turbine
drills and air syringes. Surgical emphysema is characterized by soft tissue swelling of
sudden onset, usually developing within seconds or minutes. Palpation of the affected
tissues shows crepitus or crackling, an important diagnostic feature. Discomfort is a
variable finding. Some patients complain of severe pain, but usually the discomfort is mild
and resolves within a few days. Most cases of subcutaneous emphysema will begin to
resolve after 2 to 3 days of supportive treatment, and residual swelling is usually minimal
after 7 to 10 days of observation. Treatment is usually conservative, and consists of
antibiotic coverage to prevent infection. Oral bacteria may possibly be carried with the
aerosol into the soft tissue and represent a potential nidus of infection. Additionally, a
course of systemic corticosteroids may promote faster resolution. Surgical decompression

American Board of Oral and Maxillofacial Surgery 79


2008 Oral and Maxillofacial Surgery Self Assessment Tool (OMSSAT)

of the extensive emphysema should not be routinely used, because it is likely to be


ineffective and may even worsen or spread the emphysema.

Reference:
Heymans SN, Babayof I. Emphysema complications in dentistry. Quint Int 1995; 26: 535-543

Stanton D, Balasanian E, Yepes JF. Subcutaneous cervicofacial emphysema and pneumo-


mediastinum: A rare complication after crown preparation. General Dentistry, 53(2), 122-4,
March-April 2005

American Board of Oral and Maxillofacial Surgery 80


2008 Oral and Maxillofacial Surgery Self Assessment Tool (OMSSAT)

The most frequent location for an impacted supernumerary tooth is:

A. maxillary central incisor region.

B. maxillary canine region.

C. maxillary third molar region.

D. mandibular premolar region.

Answer: A

Rationale:
The most frequent site for supernumerary teeth is the maxillary central region, followed
by the maxillary lateral incisor region and the maxillary canine region.

Reference:
Andreasen JO, Petersen JK, Laskin DM, Textbook and Color Atlas of Tooth Impactions,
Mosby, 1997, pp. 95.

Fonseca, RJ, Oral and Maxillofacial Surgery Vol 1, Anesthesia/Dentoalveolar


Surgery/Office Management, WB Saunders 2000, p.355

American Board of Oral and Maxillofacial Surgery 81


2008 Oral and Maxillofacial Surgery Self Assessment Tool (OMSSAT)

Which tooth orientation is generally the most difficult mandibular third molar impaction to
remove?

A. Mesioangular

B. Distoangular

C. Horizontal

D. Vertical

Answer: B

Rationale:
The distoangular impaction is the most difficult mandibular third molar to remove owing to
the path of delivery into the ascending ramus. This situation often necessitates a
considerable amount of ostectomy and multiple sectioning of the tooth.

Reference:
Fonseca, RJ, Oral and Maxillofacial Surgery Vol 1, Anesthesia/Dentoalveolar
Surgery/Office Management, WB Saunders 2000, p.262.

Perciaccante VJ, Haug RH, Oral and Maxillofacial Surgery Clinics of North America,
Management of Impacted Teeth, Vol 19, No 1, WB Saunders, 2007, p.24.

American Board of Oral and Maxillofacial Surgery 82


2008 Oral and Maxillofacial Surgery Self Assessment Tool (OMSSAT)

Coronectomy is an alternative technique for the management of an impacted mandibular third


molar when:

A. there is periapical infection.

B. the tooth is mobile.

C. the root is intimately associated with the inferior alveolar nerve.

D. the tooth is horizontally impacted along the course of the inferior alveolar nerve.

Answer: C

Rationale:
Coronectomy is a viable technique in those cases where removal of an impacted third
molar might put the inferior alveolar nerve at considerable risk of damage. Infection, tooth
mobility, and horizontally impacted teeth adjacent to the nerve are contraindications for
this technique.

Reference:

Pogrel MA, Lee JS, Muff DF, Coronectomy: A technique to protect the Inferior Alveolar
Nerve, JOMS, Vol 621, No 12, Elseveir, 2004, p1447-52.

Pogrel MA, Oral and Maxillofacial Surgery Clinics of North America, Management of
Impacted Teeth, Vol 19, No 1, WB Saunders, 2007, p. 85-91

American Board of Oral and Maxillofacial Surgery 83


2008 Oral and Maxillofacial Surgery Self Assessment Tool (OMSSAT)

The best technique for performance of a partial odontectomy (coronectomy) is to remove tooth
structure:

A. to a level approximately 3 mm above the level of the inferior alveolar canal and healing by
secondary intention.

B. so that the remaining roots are at least 3 mm below the crestal bone followed by healing by
secondary intention.

C. so that the remaining roots are at least 3mm below the crestal bone followed by watertight
primary closure.

D. to a level approximately 3 mm above the level of the inferior alveolar canal followed by
watertight primary closure.

Answer: C

Rationale:
The problem of inferior alveolar nerve involvement during the removal of lower third
molars is a clinical and, more recently, medicolegal issue. Because the results of damage to
the inferior alveolar nerve are unpredictable in that many cases do recover but some do not,
it is preferable to carry out a technique that may reduce the possibility of this involvement.
The technique of coronectomy, partial odontectomy, or deliberate root retention, is one
such technique. The best technique for partial odontectomy involves sectioning of the
tooth at a 45 degree angle (as measured bucco-lingually) followed by further reduction to
reduce the remaining fragments 3 mm below the crestal bone level. The distance of 3 mm
has been validated by animal studies to allow bone formation over the retained root
fragments. Primary closure is indicated to reduce the risk of postoperative infection.

The technique of coronectomy seems to be a safe and straightforward technique with few
complications or potential complications. In Pogrel's series, there has only been one case of
mild, transient (5 days) lingual paresthesia, presumably caused by the lingual retraction, but
no other cases of lingual nerve involvement were reported. Other studies, however, have
suggested a higher rate of transient lingual paresthesias from the use of the lingual retractor
but not permanent cases of lingual nerve involvement. There does not seem to be any need
to treat the exposed pulp of the tooth, and root treatment actually seems to be
contraindicated. Animal studies have shown that vital roots remain vital with minimal
degenerative changes.

Reference:
Pogrel MA, LEE JS, Muff DF. Coronectomy: a technique to protect the inferior alveolar
nerve. J Oral Maxillofac Surg 2004; 62 (12):1447-52.

Pogrel MA. Partial Odontectomy. Oral Maxillofacial Surg Clin N Am 19(2007)85-91.

American Board of Oral and Maxillofacial Surgery 84


2008 Oral and Maxillofacial Surgery Self Assessment Tool (OMSSAT)

The best time to provide intravenous preoperative antibiotic therapy prior to removal of impacted
third molars associated with pericoronal infection is:

A. immediately prior to surgery.

B. 0.5-2 hours prior to surgery.

C. 3-4 hours prior to surgery.

D. 6 hours prior to surgery.

Answer: B

Rationale:
Prophylactic antibiotic therapy for asymptomatic impacted third molars in healthy patients
is not indicated. For the compromised patient or one who is actively infected, antibiotic
administration should be timed so that incision is performed at the peak systemic
concentration.

Reference:

Mehrabi M, Allen JM, Roser SM.. Therapeutic Agents in Perioperative Third Molar
Surgical Procedures. Oral Maxillofacial Surg Clin N Am 19(2007), p. 71.

Woods RK, Dellinger EP. Current guidelines for antibiotic prophylaxis of surgical
wounds. Am Fam Physician 1998; 57 (11):2731-40.

American Board of Oral and Maxillofacial Surgery 85


2008 Oral and Maxillofacial Surgery Self Assessment Tool (OMSSAT)

Bleeding encountered during exploration for this displaced, previously impacted, maxillary third
molar would most likely emanate from the:

A. pterygoid venous plexus.

B. sphenopalatine artery.

C. descending palatine artery.

D. masseteric artery.

Answer: A

Rationale:
This scan depicts a tooth displaced into the infratemporal fossa. Maxillary third molars that
are superiorly positioned may have only a thin layer of bone separating them from the
infratemporal space. Venous bleeding from the pterygoid plexus of veins often makes
visualization of the tooth difficult.

Reference:
Bouloux GF et al. Complications of Third Molar Surgery. Oral and Maxillofacial Surg
Clin N Am 19 (2007), p. 122.

American Board of Oral and Maxillofacial Surgery 86


2008 Oral and Maxillofacial Surgery Self Assessment Tool (OMSSAT)

Pogrel M. Complications of Third Molar Surgery. Oral and Maxillofacial Surg Clin N Am
1990; 2:441.

American Board of Oral and Maxillofacial Surgery 87


2008 Oral and Maxillofacial Surgery Self Assessment Tool (OMSSAT)

The best approach for surgical exposure of an impacted tooth for orthodontic bracketing is:

A. complete exposure of the CEJ.

B. partial exposure of the crown and avoiding exposure of the CEJ.

C. complete exposure of the CEJ and 1 mm of surrounding alveolar bone.

D exposure of the crown until the greatest diameter of the crown is revealed regardless of the
CEJ.

Answer: B

Rationale:
The surgical exposure of an impacted tooth should be carried out conservatively so that
only enough bone and soft tissue is removed to place on orthodontic bracket. Damaging
effects to the periodontium have been shown to be more frequent with exposure of the CEJ.

Reference:
Zeitler DL. Management of Impacted Teeth other than Third Molars, pp.133-134. In
Miloro M ed. Oral and Maxillofacial Surgery, Vol. 1, BC Becker, 2004.

Kohavi D, Becker A, Silverman Y. Surgical exposure, orthodontic movement, and final


tooth position as factors in periodontal breakdown of treated palatally impacted canines.
Am J Orthod 1984; 85:72-77.

American Board of Oral and Maxillofacial Surgery 88


2008 Oral and Maxillofacial Surgery Self Assessment Tool (OMSSAT)

The posterior extension of an incision designed for removal of an impacted mandibular third
molar is extended laterally because:

A. this design places the incision in keratinized tissue.

B. this design avoids injury to the buccal artery.

C. this design avoids injury to the lingual nerve.

D. this design reduces postoperative trismus.

Answer: C

Rationale:
The incision design for removal of impacted third molar flares laterally to avoid possible
injury to the lingual nerve. The mandibular ramus flares laterally and the lingual nerve
extends above the lingual crest 10% of the time.

Reference:
Ness GM, Peterson LJ. Impacted Teeth, p. 144. In Miloro M ed. Oral and Maxillofacial
Surgery, vol. 1, BC Becker, 2004.

Miloro M, Halkias LE, Slone HW, Chakeres DW. Assessment of the lingual nerve in the
third molar region using magnetic resonance imaging. J Oral Maxillofac Surg
1997;52:134-7.

American Board of Oral and Maxillofacial Surgery 89


2008 Oral and Maxillofacial Surgery Self Assessment Tool (OMSSAT)

While attempting to extract an impacted tooth #16, the tooth is suddenly displaced, and is no
longer visible or palpable. The patient now has limited mandibular opening. What is the most
likely position of tooth #16?

A. In the maxillary sinus

B. In the buccal space

C. In the body of the zygoma

D. In the infratemporal space

Answer: D

Rationale:
Upper third molars can be displaced distally into the infratemporal space if excessive distal
elevation is used without placement of retraction distal to the tooth. In this particular case,
the new finding of restricted opening gives a clue that the tooth is not in the maxillary
sinus, but most probably impinging on the coronoid process, and thus is in the
infratemporal fossa.

Reference:
Dentoalveolar Surgery in Peterson's Principles of Oral and Maxillofacial Surgery, Miloro,
M. et al., Editors, Second Edition, BC Decker Inc, 2004, pp. 152.

Fonseca, et al. Oral and Maxillofacial Surgery: Anesthesia/Dentoalveolar Surgery/Office


Management. (Vol 1). W.B. Saunders Company. Philadelphia. 2000. p426-427.

American Board of Oral and Maxillofacial Surgery 90


2008 Oral and Maxillofacial Surgery Self Assessment Tool (OMSSAT)

What is the most frequent postoperative complication of third molar extraction?

A. Localized alveolar osteitis

B. Subperiosteal abscess

C. Inferior alveolar nerve injury

D. Lingual nerve injury

Answer: A

Rationale:
In a prospective study involving 63 surgeons, 3,760 patients, and 8,333 third molars; it was
found that the incidence of localized alveolar osteitis was 12% in mandibular molars.
Other complication rates were: infection 1%, IAN injury 1.1-1.7%, Lingual nerve injury
0.3%.

Reference:
Haug, R, et al. The American Association of Oral and Maxillofacial Surgeons Age-Related
Third Molar Study. JOMS 63:1106-1114, 2005

Dentoalveolar Surgery in Peterson's Principles of Oral and Maxillofacial Surgery, Miloro,


M. et al., Editors, Second Edition, BC Decker Inc, 2004, pp. 151.

American Board of Oral and Maxillofacial Surgery 91


2008 Oral and Maxillofacial Surgery Self Assessment Tool (OMSSAT)

What is the frequency with which the lingual nerve runs superior to the lingual crest of the
mandible?

A. < 5%

B. 10%

C. 30%

D. 50%

Answer: B

Rationale:
In a clinical study utilizing magnetic resonance imaging of the mandibles of normal human
subjects, the lingual nerve was found to be on average 2.8mm inferior to the lingual crest
and 2.5mm medial to the lingual plate. In 10% of the group, the lingual nerve was above
the lingual crest and 25% were in direct contact to the lingual plate.

Reference:
Behnia H et al, An anatomic study of the lingual nerve in the third molar region.J Oral
Maxillofacial Surg, 2000. 58:649.

Miloro M et al, Assessment of the lingual nerve in the third molar region using magnetic
resonance imaging. J Oral Maxillofacial Surg, 1997. 55:134.

American Board of Oral and Maxillofacial Surgery 92


2008 Oral and Maxillofacial Surgery Self Assessment Tool (OMSSAT)

Which radiographic finding is most highly associated with inferior alveolar nerve exposure
during surgical removal of mandibular third molars?

A. Darkening of the third molar tooth root

B. Narrowing of the third molar tooth root

C. Deflection of the third molar roots

D. Diversion of the inferior alveolar canal

Answer: A

Rationale:
IAN exposure is associated with increased incidence of IAN injury1. The radiographic
finding linked with the most relative risk is darkening of the third molar tooth root 2. All
other answers are associated with IAN exposure, but to a lesser amount.

Reference:
Valmaseda-Castellon E, Berini-Aytes L, Gay-Escoda C. Inferior alveolar nerve damage
after lower third molar surgical extraction: a prospective study of 1117 surgical extractions.
Oral Surg Oral Med Oral Pathol Oral Radiol Endod. 2001; 92:377-383.

Sedaghatfar M, August MA, Dodson TB. Panoramic radiographic findings as predictors of


inferior alveolar nerve exposure following third molar extraction. J Oral Maxillofac Surg.
2005; 63:3-7.

American Board of Oral and Maxillofacial Surgery 93


2008 Oral and Maxillofacial Surgery Self Assessment Tool (OMSSAT)

When performing a ridge splitting technique to expand the edentulous ridge for insertion of an
interpositional bone graft prior to implant placement, what is the preferred preoperative
minimum ridge width?

A. 1 mm

B. 2 mm

C. 3 mm

D. 4 mm

Answer: C

Rationale:
The ridge splitting technique is used to expand the edentulous ridge for implant placement
or insertion of an interpositional bone graft. This technique is only suitable for enhancing
ridge width. There must be adequate available bone height for implant placement, and no
vertical bone defect should be present. Although skilled surgeons may be able to expand
very thin ridges, a minimum width of 3.0 mm is preferred. Splitting ridges narrower than
this is technique sensitive and can result in bone fractures and resorption.

Reference:
Misch C. Implant site development using ridge splitting techniques. Oral Maxillofacial
Surg Clin N Am (2004) p.65.

Fonseca, et al. Oral and Maxillofacial Surgery: Reconstructive and Implant Surgery: (Vol
7). Pages 219-221.

American Board of Oral and Maxillofacial Surgery 94


2008 Oral and Maxillofacial Surgery Self Assessment Tool (OMSSAT)

When performing alveolar ridge development using orthodontic forced eruption, what is the
recommended amount of tooth movement per month?

A. 1.0 2.0 mm

B. 2.1 3.0 mm

C. 3.1 4.0 mm

D. 4.1 5.0 mm

Answer: A

Rationale:
The orthodontist needs to know what the clinician plans to gain by the forces applied to the
tooth or teeth in the treatment area. Forces for crown lengthening would be rapid. By
contrast, forced eruption for implant site development would be slower, approximately 1 to
2 mm per month. Caution should be taken to avoid moving the root too rapidly. In
addition, the length of root in bone may affect the rate of movement and amount of applied
force necessary. In a situation of severe bone loss, the remaining root in bone may be
minimal (providing less resistance) and may move faster.

Reference:
Hinds K. Alveolar ridge development with forced eruption and distraction of retained
natural dentition. Oral Maxillofacial Surg Clin N Am 16 (2004) p.76 - 78.

Mantzikos T, Shamus I. Forced eruption and implant site development: an osteophysiologic


response. Am J Orthod Dentofacial Orthop 1999; 115(5):58391.

American Board of Oral and Maxillofacial Surgery 95


2008 Oral and Maxillofacial Surgery Self Assessment Tool (OMSSAT)

The optimal rate of distraction that is recommended while performing the alveolar distraction
technique for implant site development is:

A. 0.5 mm per day.

B. 1.0 mm per day.

C. 2.0 mm per day.

D. 2.5 mm per day.

Answer: B

Rationale:
The rate of distraction is the distance the bone is lengthened each day. Various rates have
been studied. A rate of less than 0.5 mm per day carries the risk of premature ossification,
whereas a distraction rate of 2.0 mm per day is associated with increased fibroconnective
tissue formation and decreased bone formation. A rate of 1 mm per day (usually done in at
least two equal distraction movements per day) is considered optimal for bone formation.

Reference:
Batal H, Cottrell D. Alveolar distraction osteogenesis for implant site development. Oral
Maxillofacial Surg Clin N Am 16 (2004) p.93.

Saulaic,N, Martn,M, Camacho, and Garca, A Complications in Alveolar Distraction


Osteogenesis: A Clinical Investigation

Journal of Oral and Maxillofacial Surgery, Volume 65, Issue 2, February 2007, Pages 267-
274

American Board of Oral and Maxillofacial Surgery 96


2008 Oral and Maxillofacial Surgery Self Assessment Tool (OMSSAT)

Following completion of alveolar distraction, the stabilizing device is maintained for:

A. 2 4 weeks.

B. 5 7 weeks.

C. 8 12 weeks.

D. 13 17 weeks.

Answer: C

Rationale:
The ability of the distractor to stabilize the newly formed bone within the distraction gap is
key to the formation of a healthy regenerate. Unstable devices are associated with
increased endochondral bone formation and delayed bone formation within the distraction
gap. Stable devices lead to direct osteogenesis without intervening cartilage formation.

Reference:
Batal H, Cottrell D. Alveolar distraction osteogenesis for implant site development. Oral
Maxillofacial Surg Clin N Am 16 (2004) p.93 - 94.

Saulacic, N, Martn, M, Camacho, and Garca, A Complications in Alveolar Distraction


Osteogenesis: A Clinical Investigation

Journal of Oral and Maxillofacial Surgery, Volume 65, Issue 2, February 2007, Pages 267-
274

American Board of Oral and Maxillofacial Surgery 97


2008 Oral and Maxillofacial Surgery Self Assessment Tool (OMSSAT)

The phase in alveolar distraction osteogenesis for implant site development, which occurs at the
end of distraction until the device is removed is known as the:

A. latency period.

B. distraction period.

C. consolidation period.

D. activation period.

Answer: C

Rationale:
The consolidation period follows active distraction and continues until device removal.
The length of the consolidation period is influenced by the age of the patient, distance and
time of distraction, and the amount of surgical trauma at the time of surgery. In cases of
increased surgical trauma, recommendations are for increasing the time of the latency
period and the consolidation period. The latency period is the time between device
placement/osteotomy and activation of the distractor.

The distraction period is the classic term for the time frame during which the distraction
device is activated and the gap between the osteotomy segments is expanded. The
activation period could also describe the distraction period, but is not classic terminology.

Reference:
Batal H, Cottrell D. Alveolar distraction osteogenesis for implant site development. Oral
Maxillofacial Surg Clin N Am 16 (2004) p.94.

Peterson, LJ, Ellis, E, Hupp, JR, Tucker, MR. Contemporary Oral and Maxillofacial
Surgery 4th Edition, Mosby, St. Louis, 2003, p. 582-584.

American Board of Oral and Maxillofacial Surgery 98


2008 Oral and Maxillofacial Surgery Self Assessment Tool (OMSSAT)

Osteogenesis occurs with which of the following grafting materials?

A. Allogeneic bone grafts

B. Xenographic bone grafts

C. Alloplastic bone grafts

D. Autogenous bone grafts

Answer: D

Rationale:
Osteogenesis refers to the growth of bone from viable cells transferred within the
graft. Autogenous bone is the only graft material available with osteogenic properties.

Allogeneic bone grafts (allografts) are treated in bone banks in a variety of methods,
resulting in different mineralized, freeze-dried, solvent-dehydrated, or demineralized states.
Allografts such as demineralized freeze-dried bone and solvent-dehydrated mineralized
bone have been advocated for use in extraction sites because of their osteoconductive
nature and the characteristic that they will resorb and be replaced within a relatively short
period of time.

Xenografts are graft materials harvested from a species other than human, typically bovine,
and are processed to remove the antigenicity by a variety of chemical and preparation
techniques. The absence of proteins results in minimal immune response in vivo.

Alloplasts include forms of calcium phosphate materialseither dense or porous


hydroxylapatite, hard tissue replacement, and bioactive glass. These materials have proved
useful for retaining alveolar bulk but can be slow to resorb because of their chemical
characteristics. Recent advances in adding materials or changing the chemical
characteristics of these materials, however, recently have been shown to provide
maintenance of form and also allow for bone formation.

Reference:
Misch, CE, Contemporary Implant Dentistry 2nd edition, page 455, Mosby 1999.

Block, MS, Treatment of the single tooth extraction site. Oral and Maxillofacial Surgery
Clinics of North America, Volume 16, Issue 1, February 2004, Pages 41-63

American Board of Oral and Maxillofacial Surgery 99


2008 Oral and Maxillofacial Surgery Self Assessment Tool (OMSSAT)

When compared to subepithelial connective tissue grafts, free gingival grafts:

A. result in less scarring.

B. revascularize more rapidly.

C. are less predictable for root coverage.

D. provide superior color matching.

Answer: C

Rationale:
Free gingival grafts are primarily intended for increasing the zone of keratinized tissue.
Root coverage, especially in wider areas of recession, is limited. Color match is inferior
and scarring evident with free gingival grafts. Subepithelial connective tissue grafts, due to
dual blood supply in the soft-tissue pouch, revascularize more rapidly.

Reference:
Block M S et al, Atlas of the Oral and Maxillofacial Surgery Clinics of North America.
Soft Tissue Esthetic Procedures for Teeth Implants, A Subepithelial Connective Tissue
Graft Procedure for Optimum Root Coverage. Pages 11-28, Volume Seven, Number Two,
WB Saunders, 1999.

Miloro M et al, Peterson's Principles of Oral and Maxillofacial Surgery. Pages 205-221,
Second Edition, BC Decker, 2004.

American Board of Oral and Maxillofacial Surgery 100


2008 Oral and Maxillofacial Surgery Self Assessment Tool (OMSSAT)

Which of the following best characterizes the closed technique of creating a recipient site for a
subepithelial connective tissue grafts?

A. It better preserves blood supply at the recipient site

B. It is preferred when abutment or root exposure is greater than 4mm apico-coronally

C. It allows for significant coronal advancement when vertical soft tissue augmentation is
desired

D. The width of the recipient site should equal that of the exposed root or abutment surface

Answer: A

Rationale:

\
Closed "pouch" technique for the preparation of a recipient site for a subepithelial
connective tissue graft. A, split thickness dissection (shaded area). B, Graft
immobilization apically & coronally.

Since no releasing incision is created, the closed technique better preserves blood supply.
The closed technique is preferred over the open technique when root or abutment exposure
is LESS than 4mm apicocoronally. The open technique allows for significant coronal
advancement; the closed technique does not. The width of the recipient site for the closed
technique should be three times that of the exposed surface to be covered. The recipient
site must be designed such that it can contribute adequate peripheral blood supply to
sustain the graft. Dissection must extend well beyond the width of soft tissue being
corrected.

Reference:
Miloro M et al, Peterson's Principles of Oral and Maxillofacial Surgery. Pages 214-215,
Second Edition, BC Decker, 2004.

Block M S et al, Atlas of the Oral and Maxillofacial Surgery Clinics of North America.
Soft Tissue Esthetic Procedures for Teeth and Implants, Subepithelial Connective Tissue
Grafting with Dental Implants. Pages 95-107, Volume Seven, Number Two, WB
Saunders, 1999.

American Board of Oral and Maxillofacial Surgery 101


2008 Oral and Maxillofacial Surgery Self Assessment Tool (OMSSAT)

The palatal roll technique for soft tissue augmentation around implants:

A. is only useful with small gingival defects.

B. can correct horizontal defects up to 5mm.

C. can be used to reconstruct the papilla.

D. should be performed one month prior to uncovering an implant.

Answer: A

Rationale:
A palatal roll procedure is accomplished at the time of uncovering an implant that has a
small gingival defect that requires augmenting. It can only be used to aid with augmenting
small horizontal and buccal width defects.

Reference:
Block M, Atlas of the Oral and Maxillofacial Surgery Clinics of North America, Vol 7
Number 2, Sept 1999, pg 109-116.

Soft Tissue Management in Implant Therapy in Peterson's Principles of Oral and


Maxillofacial Surgery, Miloro, M. et al., Editors, Second Edition, BC Decker Inc, 2004, pp.
205-221.

American Board of Oral and Maxillofacial Surgery 102


2008 Oral and Maxillofacial Surgery Self Assessment Tool (OMSSAT)

Connective tissue grafts are less predictable if performed at the time of:

A. extraction.

B. cortico-cancellous grafting.

C. implant placement.

D. implant uncovering.

Answer: B

Rationale:
Part of a connective tissue graft's blood supply is obtained from the underlying of the
periosteum. All surgical procedures interrupt this blood supply to some degree. The
placement of a block graft, which involved the elevation of the periosteum off the native
bone bed, has the highest potential to interfere with the blood supply and therefore has been
found to have a higher complication rate with less predictability compared to the other
listed procedures.

Reference:
Block M, Atlas of the Oral and Maxillofacial Surgery Clinics of North America, Vol 7
Number 2, Sept 1999, pp 95-107.

Soft Tissue Management in Implant Therapy in Peterson's Principles of Oral and


Maxillofacial Surgery, Miloro, M. et al., Editors, Second Edition, BC Decker Inc, 2004, pp.
209-221.

American Board of Oral and Maxillofacial Surgery 103


2008 Oral and Maxillofacial Surgery Self Assessment Tool (OMSSAT)

Following site development for implant placement by orthodontic separation of maxillary


anterior teeth, what happens to the buccolingual width of the edentulous alveolus over time?

A. Less than 1% of the ridge resorbs over a period of 4 years

B. The ridge narrows by 34% over 5 years

C. More resorption occurs in adolescents than in adult patients

D. The ridge resorbs over 5 years whether or not fixtures are placed

Answer: A

Rationale:
A study by Spear, Mathews, and Kokich evaluated the long term width of the alveolar ridge
after space had been opened for missing maxillary lateral incisors in adolescent orthodontic
patients. The amount of bone loss was less than 1% over 4 years. Previous studies have
shown that if maxillary anterior teeth are extracted, the alveolar ridge will narrow by 34%
over a 5 year period.

Reference:
Spear FM, Mathews DM, Kokich VG. Interdisciplinary management of single-tooth
implants. Semin Orthod. 1997; 3(1):45-72.

Kokich V: Maxillary lateral incisor implants: planning with the aid of orthodontics. J Oral
Maxillofac Surg. 2004 Sep; 62(9 Suppl 2):48-56.

American Board of Oral and Maxillofacial Surgery 104


2008 Oral and Maxillofacial Surgery Self Assessment Tool (OMSSAT)

When conventional bone-added / socket lift osteotome technique is used, what raises the floor of
the sinus?

A. The bur used to drill the pilot hole

B. The hydraulic pressure of the graft material

C The smallest osteotome

D The largest osteotome

Answer: B

Rationale:
In this sinus elevation technique, drills or osteotomes are used to reach the level of the
sinus floor. An osteotome is used to fracture the sinus floor immediately prior to implant
of graft placement. Graft material or the implant itself raises the sinus floor as the final
step.

Reference:
Hahn J. Clinical uses of osteotomes. J Oral Implantol, 1999; 25:23-29.

Al-Maseeh J, Levin B, Symeonides E. The osteotome technique: a classification for


technique approach and clinical case reports. Compen Contin Educ Dent. 2005; 26:551-
556.

American Board of Oral and Maxillofacial Surgery 105


2008 Oral and Maxillofacial Surgery Self Assessment Tool (OMSSAT)

When compared to areas grafted with palatal autografts, areas grafted with acellular dermal
matrix allografts demonstrate which of the following qualities?

A. Less graft shrinkage and greater amounts of keratinized tissue

B. More graft shrinkage and greater amounts of keratinized tissue

C. Less graft shrinkage and lesser amounts of keratinized tissue

D. More graft shrinkage and lesser amounts of keratinized tissue

Answer: D

Rationale:
Acellular dermal matrix grafts act as a biologically compatible framework into which
fibroblasts and epithelial cells can migrate and adhere, thus repopulating and incorporating
the cells into the material. These migrating tissues replace the dermal matrix, causing
increased graft shrinkage and ultimately resulting in lesser amounts of keratinized tissue at
the recipient site.

Reference:
Yan JJ. Tsai AY. Wong MY. Hou LT. Int J Periodontics Restorative Dent. 2006 June;
26(3):287-92

McGuire MK. Nunn ME. Evaluation of the safety and efficacy of periodontal applications
of a living tissue-engineered human fibroblast-derived dermal substitute. I. Comparison to
the gingival autograft: a randomized controlled pilot study. Journal of Periodontology.
76(6):867-80, 2005 Jun.

American Board of Oral and Maxillofacial Surgery 106


2008 Oral and Maxillofacial Surgery Self Assessment Tool (OMSSAT)

Where keratinized gingival tissue exists, the best surgical option to thicken tissue around an
implant to minimize facial metal show:

A. laterally repositioned flap.

B. semilunar flap.

C. free gingival graft.

D. subepithelial connective tissue graft.

Answer: D

Rationale:
There are several indications for the use of the subepithelial connective tissue graft and
dental implant sites. Thickening gingiva to eliminate metal show from the underlying
dental implant is one of these indications. The subepithelial connective tissue graft can
thicken the gingiva one to 3 mm, depending on the thickness of the graft and
contracture/shrinkage of the graft with healing.

Reference:
Block M S et al, Atlas of the Oral and Maxillofacial Surgery Clinics of North America.
Soft Tissue Esthetic Procedures for Teeth Implants, Subepithelial Connective Tissue
Grafting with Dental Implants. Pages 95-107, Volume Seven, Number Two, WB
Saunders, 1999.

Block M S et al, Atlas of the Oral and Maxillofacial Surgery Clinics of North America.
Soft Tissue Esthetic Procedures for Teeth Implants, A Subepithelial Connective Tissue
Graft Procedure for Optimum Root Coverage. Pages 11-28, Volume Seven, Number Two,
WB Saunders, 1999.

American Board of Oral and Maxillofacial Surgery 107


2008 Oral and Maxillofacial Surgery Self Assessment Tool (OMSSAT)

The biologic width refers to the distance between the:

A. gingival margin and the crestal bone.

B. base of the sulcus and the alveolar crest.

C. gingival margin and the junctional epithelium.

D. base of the sulcus and the cementoenamel junction.

Answer: B

Rationale:
The biologic width is defined as the distance from the base of the sulcus to the crest of the
alveolar ridge. Accurate sulcus depth can be problematic to measure with implants since
there is no direct connective tissue fiber insertion into the implant surface; so in theory a
periodontal probe (especially of excessive pressure is used) can pass through the top of the
implant epithelial attachment (the bottom of the sulcus) to the top of the alveolar ridge
crest. Biologic width (usually at least 2mm) is maintained between any microgap in the
fixture/abutment interface.

Reference:
Misch, CE, Dental Implant Prosthetics, pages 74-75, Mosby, Inc. 2005.

Fonseca, R J et al, Oral and Maxillofacial Surgery Volume Seven, Soft Tissue
Considerations, page 341, WB Saunders 2000.

American Board of Oral and Maxillofacial Surgery 108


2008 Oral and Maxillofacial Surgery Self Assessment Tool (OMSSAT)

For a subantral osseous augmentation, non-resorbable HA is added to autogenous bone in order


to:

A. add bulk to the graft.

B. improve initial implant stability.

C. decrease infection rate.

D. improve osteogenesis.

Answer: A

Rationale:
When mixed to a 1:1 ratio with an allograft or autograft, nonresorbable hydroxyapatite
helps compensate for the nature loss of bone volume that is seen with the grafts alone. It is
also stable in its volume independently and therefore acts as a latticework providing
structural support.

Reference:
Babbush, CA, Dental Implants: The Art and Science, WB Saunders, 2001, pp 158-9
Block, MS: Treatment of the single tooth extraction site, Oral Maxillofacial Surg Clin N
Am 16 (2004) 4163

American Board of Oral and Maxillofacial Surgery 109


2008 Oral and Maxillofacial Surgery Self Assessment Tool (OMSSAT)

Local contraindications to a four-millimeter diameter posterior single tooth implant include:

A. an adjacent tooth that requires a crown.

B. mesiodistal bone width <7mm.

C. +1 mobility of an adjacent tooth.

D. buccolingual bone width <7mm.

Answer: B

Rationale:
Contraindications to implant placement in this situation include inadequate bone volume of
<6mm in the buccolingual direction and <7mm in the mesiodistal direction. Also, more
than one adjacent tooth of a moderate to advanced mobility is considered unacceptable.
Both adjacent teeth requiring crowns is a relative indication for a fixed partial denture
restoration (influenced by the long-term prognosis of the adjacent tooth or teeth.)

Reference:
Misch, CE, Dental Implant Prosthetics, Mosby, 2005, pp 354-5

Manual of Dental Implants: David P. Sarment, D.D. S., M.S., page 14

Fonseca, et al. Oral and Maxillofacial Surgery: Reconstructive and Implant Surgery: (Vol
7). Pages 211-242.

American Board of Oral and Maxillofacial Surgery 110


2008 Oral and Maxillofacial Surgery Self Assessment Tool (OMSSAT)

A flapless approach in the placement of dental implants has shown to:

A. have a positive effect on interdental papilla preservation.

B. have a failure rate of 10% below a flap approach.

C. be of greater benefit in the vertically atrophic maxilla.

D. minimize buccal fenestration apically.

Answer: A

Rationale:
The flapless technique preserves the interdental papilla though has not been shown to have
a consistently predicable esthetic benefit as compared to a flap approach. The inability to
directly visualize the alveolus can present problems with undetected fenestration and
dehiscence.

Reference:
Parel SM and Schow SR: Early Clinical Experience With a New One-Piece Implant
System in Single Tooth Sites, J Oral Maxillofac Surg 63:2-10, 2005

Becker W. Goldstein M. Becker BE. Sennerby L. Minimally invasive flapless implant


surgery: a prospective multicenter study. Clinical Implant Dentistry & Related Research. 7
Suppl 1:S21-7, 2005.

American Board of Oral and Maxillofacial Surgery 111


2008 Oral and Maxillofacial Surgery Self Assessment Tool (OMSSAT)

The osteotome expansion technique for preparation of an implant osteotomy site:

A. requires a broad alveolar ridge to receive the implant.

B. cannot be used with soft bone.

C. compresses bone immediately adjacent to the implant.

D. requires sequential drill preparation prior to its use.

Answer: C

Rationale:
Osteotomes are ideal for the use in soft maxillary bone. This technique can be used to split
or widen a narrow ridge to receive an implant. The osteotomes condense and laterally
compress the soft bone at the osteotomy site. Only a small 1.5mm drill (pilot) needs to be
used prior to use with an osteotome.

Reference:
Fonseca: Oral & Maxillofacial Surgery, Volume Number 7. Pages 111-114.

Misch, CE: Dental Implant Site Preservation and Development,Oral and Maxillofacial
Surgery Clinics of North America Vol16, Issue 1, February 2004, pp 70-72

American Board of Oral and Maxillofacial Surgery 112


2008 Oral and Maxillofacial Surgery Self Assessment Tool (OMSSAT)

What is minimum amount of buccallingual bone width to allow placement of a 4mm diameter
implant?

A. 5mm

B. 6mm

C. 7mm

D. 8mm

Answer: B

Rationale:
At least 1mm of buccal and 1mm of lingual bone are required; therefore a 6mm width for a
4mm diameter implant. With regards to the mesial distal width, 7mm is required. 4mm for
the implant and 1mm of bone both along the mesial and distal surfaces with the additional
knowledge that the periodontal ligaments of the adjacent teeth are 0.5mm.

Reference:
Manual Dental Implants: David P. Sarment, D.D. S., M.S., page 14.

Misch, CE, Dental Implant Prosthetics, Mosby, 2005, pp 354-5

Fonseca, et al. Oral and Maxillofacial Surgery: Reconstructive and Implant Surgery: (Vol
7). Pages 211-242.

American Board of Oral and Maxillofacial Surgery 113


2008 Oral and Maxillofacial Surgery Self Assessment Tool (OMSSAT)

Which of the following infections has a DNA virus as its etiologic agent?

A. Mononucleosis

B. Scrofula

C. AIDS

D. Herpangina

Answer: A

Rationale:
It is estimated that 79% of Mononucleosis from an Epstein - Barr virus (EBV) infection; 21
% from a cytomegalovirus (CMV) infection. Both are DNA viruses.

Scrofula (tuberculosis cervical lymphadenitis) is a secondary infection of the cervical


lymph node chain, associated with active pulmonary tuberculosis (Mycobacterium
tuberculosis) and results in a draining lesion. AIDS is transmitted by Human
immunodeficiency virus (HIV) and is a RNA retrovirus. Patients present with opportunistic
infections. Herpangina is a Coxsackie virus infection, and is characterized by vesicular
eruptions on the fauces and palate. Coxsackie viruses are RNA viruses.

Reference:
Bergman, SA In Topazian, RG, Goldberg, MH, Hupp, JR, editors: Oral and Maxillofacial
Infections, ed 4, Philadelphia, 2002, W B Saunders Co., pp 243-278.

McKenna, SJ In Topazian, RG, Goldberg, MH, Hupp, JR, editors: Oral and Maxillofacial
Infections, ed 4, Philadelphia, 2002, W B Saunders Co., pp 456-467.

Hupp, JR In Topazian, RG, Goldberg, MH, Hupp, JR, editors: Oral and Maxillofacial
Infections, ed 4, Philadelphia, 2002, W B Saunders Co., pp 112-125.

Rubinovitch et al in Cohen & Powderly: Infectious Diseases, 2nd ed., Copyright 2004
Elsevier, p 200.

American Board of Oral and Maxillofacial Surgery 114


2008 Oral and Maxillofacial Surgery Self Assessment Tool (OMSSAT)

A patient who has had a splenectomy within the last 6 months is most susceptible to which of the
following infectious agents?

A. Treponema pallidum

B. Haemophilus influenzae

C. Pneumocystis carinii

D. Mycobacterium tuberculosis

Answer: B

Rationale:
Patients who have had a splenectomy are most commonly susceptible to infection by
encapsulated bacteria. Haemophilus influenzae is the only bacteria listed which is
encapsulated. Following splenectomy, it is recommended that patients be administered the
Pneumnovax vaccine to provide active immunity to the encapsulated pneumococcus
bacteria.

Reference:
McKenna, SJ In Topazian, RG, Goldberg, MH, Hupp, JR, editors: Oral and Maxillofacial
Infections, ed 4, Philadelphia, 2002, W B Saunders Co., pp 456-467

CDC, Morbidity & Mortality Weekly Report: Prevention of Pneumococcal Disease


Recommendations of the Advisory Committee on Immunization Practices (ACIP) April 4,
1997 / Vol. 46 / No. RR-8

American Board of Oral and Maxillofacial Surgery 115


2008 Oral and Maxillofacial Surgery Self Assessment Tool (OMSSAT)

Below is a gram stain of drainage from a neck wound. Which of the following is the most
appropriate initial oral antibiotic therapy?

A. Metronidazole

B. Trimethoprim-sulfamethoxazole

C. Penicillin

D. Vancomycin

Answer: B

Rationale:
The gram stain of gram-positive cocci in clusters is consistent with staphylococcus, which
is aerobic. Metronidazole has antimicrobial activity against anaerobic bacteria. Penicillin
has some antimicrobial against staphylococcus that does not produce penicillinase. The
gram stain does not provide any insight as to whether the bacteria produce penicillinase, so
this would be a poor choice. Vancomycin does have excellent antibacterial properties
against staphylococcus, but it is not absorbed when taken orally. Trimethoprim-
sulfamethoxazole, although typically prescribed for its activity against gram negative
bacteria, also has excellent antibacterial properties against staphylococcus, is well absorbed
orally, and is the most appropriate initial antibiotic of those listed.

Reference:
McCarter, YS In Topazian, RG, Goldberg, MH, Hupp, JR, editors: Oral and Maxillofacial
Infections, ed 4, Philadelphia, 2002, W B Saunders Co., pp 47-50.

Hupp, JR In Topazian, RG, Goldberg, MH, Hupp, JR, editors: Oral and Maxillofacial
Infections, ed 4, Philadelphia, 2002, W B Saunders Co., pp 112-125.

American Board of Oral and Maxillofacial Surgery 116


2008 Oral and Maxillofacial Surgery Self Assessment Tool (OMSSAT)

A patient with a severe cervicofacial infection is noted to have tea-colored urine. The presence
of which of the following on urinalysis would suggest necrotizing fasciitis?

A. Bilirubin

B. Hemoglobin

C. Myoglobin

D. Haptoglobin

Answer: C

Rationale:
Necrotizing fasciitis can be associated with rhabdomyolysis and secretion of myoglobin in
the urine. Although severe infections can be associated with hemolysis and subsequent
hemoglobinuria, and increased urine bilirubin, these parameters are not suggestive of
underlying muscle inflammation and necrosis. Haptoglobin binds hemoglobin and is too
large a molecule to be filtered n the urine.

Reference:
Marx: Rosen's Emergency Medicine: Concepts and Clinical Practice, 5th ed., Copyright
2002 Mosby, Inc. pp.1763-1766

Hoffman: Hematology: Basic Principles and Practice, 3rd ed., Copyright 2000 Churchill
Livingstone, Inc., p. 408.

American Board of Oral and Maxillofacial Surgery 117


2008 Oral and Maxillofacial Surgery Self Assessment Tool (OMSSAT)

Antibiotic prophylaxis prior to dentoalveolar surgery is recommended for which disorder?

A. Mitral valve prolapse

B. Mitral valve prolapse with regurgitation

C. Artificial heart valves

D. Calcified aortic stenosis

Answer: C

Rationale:
According to the newest AHA guidelines (11) from (4-19-2007):
Antiobiotic prophylaxis is required prior to dental surgery only for the following
conditions:
1. Artificial heart valves
2. A history of having had bacterial endocarditis
3. Certain specific, serious congenital (present from birth) heart conditions, including:
a) Unrepaired or incompletely repaired cyanotic congenital heart disease, including
those with palliative shunts and conduits
b) A completely repaired congenital heart defect with prosthetic
Material or device, whether placed by surgery or by catheter intervention, during
the first six months after the procedure
c) Any repaired congenital heart defect with residual defect at the site or adjacent to
the site of a prosthetic patch or a prosthetic device
d) A cardiac transplant which develops a problem in a heart valve.

Reference:
Wilson W, Taubert KA, Gewitz M, Lockhart PB, Baddour LM, Levison M, Bolger A,
Cabell CH, Takahashi M, Baltimore RS, Newburger JW, Strom BL,
Tani LY, Gerber M, Bonow RO, Pallasch T, Shulman ST, Rowley AH, Burns
JC, Ferrieri P, Gardner T, Goff D, and Durack DT. Prevention of Infective Endocarditis.
Guidelines from the American Heart Association.
A Guideline from the American Heart Association Rheumatic Fever,
Endocarditis, and Kawasaki Disease Committee, Council on Cardiovascular
Disease in the Young, and the Council on Clinical Cardiology, Council on Cardiovascular
Surgery and Anesthesia, and the Quality of Care and Outcomes Research Interdisciplinary
Working Group. Circulation 2007

American Board of Oral and Maxillofacial Surgery 118


2008 Oral and Maxillofacial Surgery Self Assessment Tool (OMSSAT)

Which factor decreases a patients risk for post-operative mandibular fracture after third molar
removal?

A. Female gender

B. Age greater than 25 years-old

C. Full dentition

D. Distoangular position of the third molar

Answer: A

Rationale:
The main factors that increase the risk for post-operative mandible fracture after third
molar removal are: 1) age greater than 25 years-old
2) male gender
3) full dentition

Reference:
Libersa P, Roze D, Cachart T, Libersa JC. Immediate and late mandibular fractures after
third molar removal. J Oral Maxillofac Surg. 2002; 60:163-165; discussion 165-166.

Krimmel M, Reinert S. Mandibular fracture after third molar removal. J Oral Maxillofac
Surg. 2000; 58:1110-1112.

American Board of Oral and Maxillofacial Surgery 119


2008 Oral and Maxillofacial Surgery Self Assessment Tool (OMSSAT)

Which of the following best characterizes the basement membrane surface of Alloderm (acellular
dermal matrix), when used as an interpositional graft for root coverage?

A. Retains reddish coloration after contact with the patients blood

B. Facilitates epithelial cell migration

C. Should be placed away from the exposed surface intended for coverage

D. Promotes revascularization

Answer: B

Rationale:
The basement membrane surface of the Alloderm graft facilitates epithelial cell migration
and attachment. The connective tissue side contains vascular channels that allow for
cellular infiltration and revascularization. The basement membrane surface should be
placed in contact with the exposed root surface when attempting to achieve root coverage,
as this is an epithelial surface. The connective tissue surface retains a reddish coloration
after contact with the patient's blood; the basement membrane side remains white.

Reference:
Miloro M et al, Peterson's Principles of Oral and Maxillofacial Surgery. Pages 220-221,
Second Edition, BC Decker, 2004.

Babbush CA et al, Dental Implants. The Art and Science. Implant Periabutment Tissue.
Pages 127-129, WB Saunders, 2001.

American Board of Oral and Maxillofacial Surgery 120


2008 Oral and Maxillofacial Surgery Self Assessment Tool (OMSSAT)

A semilunar flap technique for coverage of a root with marginal tissue recession is indicated in
which of the following situations?

A. Absence of interdental papilla

B. Thin scalloped periodontium

C. Lack of severe facial ridge curvature

D. Inadequate zone of keratinized tissue

Answer: C

Rationale:
A semilunar flap technique will not recreate the dental papilla and it exhibits poor success
in patients with thin scalloped periodontium. If there is not an adequate zone of keratinized
tissue to start, the procedure cannot be accomplished. A severe facial curvature of the bone
will also prevent success of the technique.

Reference:
Nasr H, Atlas of the Oral and Maxillofacial Surgery Clinics of North America, Vol 7
Number 2, Sept 1999, pg 29-37

Tarnow, D, Solving Restorative Esthetic Dilemmas with the Semilunar Coronally


Positioned Flap Journal of Esthetic and Restorative Dentistry 6 (2) 1994, 6164.

American Board of Oral and Maxillofacial Surgery 121


2008 Oral and Maxillofacial Surgery Self Assessment Tool (OMSSAT)

Which of the following is a limitation of the palatal connective tissue graft technique?

A. High incidence of poor healing

B. Dependence on smooth palate donor site

C. Graft availability is dependent on donor site thickness

D. High incidence of neurovascular injury

Answer: C

Rationale:
Connective tissue grafting extremely useful in that is does not depend upon a smooth palate
and heals very well. The incidence of neurovascular injury is also very low if harvested in
the classic manner (anterior to the maxillary first molar.) Depending upon the thickness of
a particular patient's tissue, the amount of graft available may be minimal and therefore
some patients may require secondary grafting several months later.

Reference:
Sclar A, Alpha Omegan, Volume 93, number3, Aug/Sept 2000, pg 38-46.

Fonseca, et al. Oral and Maxillofacial Surgery: Reconstruction and Implant Surgery. (Vol
7). WB Saunders Company. Philadelphia. 2000, pp 335-8.

American Board of Oral and Maxillofacial Surgery 122


2008 Oral and Maxillofacial Surgery Self Assessment Tool (OMSSAT)

What is the minimum amount of time a parotid stent be kept in place after a ductal repair?

A. 2-4 weeks

B. 6-8 weeks

C. 10-12 weeks

D. 16-18 weeks

Answer: A

Rationale:
Deep lacerations of the cheek can injure the parotid duct and facial nerve branches. The
duct should be explored. Proximal and distal ends are identified, and a stent is sutured to
the intraoral mucosa to prevent accidental displacement while the duct heals and is
removed 2 to 4 weeks. This is sufficient time to allow for re-epithelialization of the
severed duct. If the proximal portion of the duct cannot be located, a pressure dressing is
applied to decrease the chances of a sialocele. Multiple aspirations of accumulated saliva
may be a necessary part of this regimen. Use of antisialogogues (such as oral
glycopyrrolate) is always a recommended adjunct when repairing a severed salivary gland
duct.

Reference:
Karas, ND. Surgery of the salivary ducts. Atlas Oral Maxillofac Surg Clin North Am.
1998 Mar;6(1):99-116

Ward Booth, P. Maxillofacial Trauma. Primary management of soft tissue trauma and
nerve reconstruction. 213-255Churchill Livingstone 2003.

American Board of Oral and Maxillofacial Surgery 123


2008 Oral and Maxillofacial Surgery Self Assessment Tool (OMSSAT)

What is the largest percent loss of an upper lip avulsive defect that can be closed primarily
without compromising function and aesthetics?

A. 20 %

B. 30 %

C. 40 %

D. 50 %

Answer: B

Rationale:
Because of great tissue elasticity, an avulsive defect of approximately 30% of the upper or
lower lip can be reconstructed with primary closure without compromising function and
esthetics. Defects greater than 30% require local and regional flaps in order to prevent
microstomia.

Reference:
Naumann, H.H. Head and Neck Surgery. Volume 1: Face, Nose and facial Skull, Part I.
Surgical Management of skin defects of the scalp, forehead, cheeks and lips. P 41-94.
Thieme 1995.

Mathes, S. Plastic Surgery. Volume 3: The Head and Neck, Part 2. Second Edition.
Elsevier 2006.

American Board of Oral and Maxillofacial Surgery 124


2008 Oral and Maxillofacial Surgery Self Assessment Tool (OMSSAT)

How long after inferior canaliculus laceration repair and intubation should the stent remain in
place in the adult patient?

A. 1-2 weeks

B. 4-6 weeks

C. 7-10 weeks

D. 12-16 weeks

Answer: D

Rationale:
Inferior canaliculus injuries need repair within 24-48 hours in order to prevent epiphora.
Repair of this kind of injuries is usually carried out by loop intubation with the punctate
being initially cannulated with silastic stents. The stents extend from the puncta through
the nasolacrimal duct and emerge in the inferior meatus, and should remain in place for at
least 3 months in the adult in cases of pediatric injuries, the same procedure is performed;
however, the stent can be removed in a shorter amount of time.

Reference:
Ward Booth, P. Maxillofacial Trauma Trauma. Primary management of soft tissue trauma
and nerve reconstruction. 213-255 Churchill Livingstone 2003.

Miloro, M. et al, Peterson's Principles of Oral and Maxillofacial Surgery. Soft Tissue
injuries, p357-370. Second Edition, BC Decker 2004.

American Board of Oral and Maxillofacial Surgery 125


2008 Oral and Maxillofacial Surgery Self Assessment Tool (OMSSAT)

A patient has an upper eyelid laceration with fat herniating from the wound. Injury to which of
the following can be eliminated?

A. Levator palpebrae superiorus

B. Globe

C. Sub-orbicularis oculi fat

D. Retro-orbicularis oculi fat

Answer: C

Rationale:
SOOF, or the sub-orbicularis oculi fat is located in the lower lid region between the
periosteum and the orbicularis oculi muscle. It should not be involved in an upper lid
injury. All other choices might occur in an upper eye lid laceration.

Reference:
Turk JB, Goldman A. SOOF lift and lateral retinacular canthoplasty. Facial Plast Surg. 17,
37, 2001.

Hwang SH, Hwang K, Jin S, et al. Location and Nature of Retro-Orbicularis Oculus Fat
and Suborbicularis Oculi Fat. J Craniofac Surg. 2007 Mar; 18(2):387-390.

American Board of Oral and Maxillofacial Surgery 126


2008 Oral and Maxillofacial Surgery Self Assessment Tool (OMSSAT)

What is the minimal pressure (in pounds per square inch) required during irrigation to
mechanically disrupt bacterial adherence to a wound surface?

A. 7 lbs (3.18 kg)

B. 10 lbs (4.55 kg)

C. 15 lbs (6.82 kg)

D. 20 lbs (9.09 kg)

Answer: A

Rationale:
To be clinically effective, irrigants must be delivered with a fluid jet impacting a wound
with 7 lb of psi. This pressure is the adequate for removing adherent bacteria from a
wound. This amount of pressure can be generated from a pulsatile irrigation apparatus.

Reference:
Fonseca, Oral & Maxillofacial Trauma, Vol II, 2005, chapter 25, Management of soft
tissue injuries 751-820.

Tobin GR. Closure of contaminated wounds. Surg Clin N Am, 64, 639-652, 1984.

American Board of Oral and Maxillofacial Surgery 127


2008 Oral and Maxillofacial Surgery Self Assessment Tool (OMSSAT)

Whitnalls ligament attaches to which of the following structures?

A. Whitnalls tubercle

B. Lockwoods suspensory ligament

C. Lateral horn of levator aponeurosis

D. Orbital lobe of the lacrimal glands

Answer: D

Rationale:
There are 4 structures attaching to the lateral retinaculum of the Whitnall's tubercle:
Lockwood's suspensory ligament, lateral horn of levator aponeurosis, check ligament of
lateral rectus muscle, and posterior limb of lateral canthal tendon. Whitnall's ligament, the
suspensory ligament of the upper lid, attaches around the orbital lobe of the lacrimal gland.

Reference:
Jelks GW, Glat PM, Jelks EB, et al. The inferior retinacular lateral canthoplasty: A new
technique. Plast Reconstr Surg 100; 1997: 1262-1275.

American Board of Oral and Maxillofacial Surgery 128


2008 Oral and Maxillofacial Surgery Self Assessment Tool (OMSSAT)

Peterson, Principles of Oral & Maxillofacial Surgery, 2004, Blepharoplasty, 1317-1344

American Board of Oral and Maxillofacial Surgery 129


2008 Oral and Maxillofacial Surgery Self Assessment Tool (OMSSAT)

The first evidence of systolic hypotension is seen in which class of hemorrhagic shock?

A. Class I

B. Class II

C. Class III

D. Class IV

Answer: C

Rationale:
Class III statatic hemorrhagic shock is distinguished by 30%-40% blood loss and the first
indication of hypotension. Class I and II hemorrhagic shock do not display any reduction
of blood pressure.

Reference:
Advanced Trauma Life Support. American College of Surgeons Committee on Trauma.
Shock, 87-107, 2005

Krausz MM. Initial resuscitation of hemorrhagic shock. World J Emerg Surg. 2006 Apr
27;1.

American Board of Oral and Maxillofacial Surgery 130


2008 Oral and Maxillofacial Surgery Self Assessment Tool (OMSSAT)

Which of the following is considered the least beneficial transport medium in the management of
avulsed permanent teeth?

A. Milk

B. Saliva

C. Hanks solution

D. Blood

Answer: D

Rationale:
Acceptable transport media for avulsed permanent teeth include Hank's balanced salt
solution, milk, saliva (vestibule), saline, and water (if none of the above is available).
Blood is not an acceptable form of transport media according to the recommended
guidelines of the American Association of Endodontists since it does not replenish PDL
cellular metabolites. Hank's solution is a balanced salt solution with a physiologic pH.

Reference:
Recommended Guidelines of the American Association of Endodontists

Peterson's Principles of Oral and Maxillofacial Surgery Vol 1; Management of Alveolar


and Dental fractures, BC Decker 2004

American Board of Oral and Maxillofacial Surgery 131


2008 Oral and Maxillofacial Surgery Self Assessment Tool (OMSSAT)

Which of the following is contraindicated true regarding replantation of avulsed teeth?

A. Primary teeth can be replanted within 1-2 hours following avulsion

B. Ankylosis following replantation of teeth is a rare complication

C. Avulsed teeth should be followed for at least 5 years to determine outcome of teeth

D. Rigid splinting is recommended in most cases following replantation of avulsed teeth

Answer: C

Rationale:
Primary teeth should not be replanted following avulsive injuries. Space maintenance is
however recommended in these situations. Replantation of avulsed primary teeth may
cause risk of pulp necrosis and interference with development of succedaneous teeth.

Reference:
Recommended Guidelines of the American Association of Endodontists

Fonseca Oral and Maxillofacial Trauma Vol 1; Diagnosis and Management of


Dentoalveolar Injuries p 427-478, Elsevier Saunders 2005

American Board of Oral and Maxillofacial Surgery 132


2008 Oral and Maxillofacial Surgery Self Assessment Tool (OMSSAT)

Which of the following can occur following placement of a tissue expander in the scalp?

A. Epidermal hypoplasia

B. Increase in dermal thickness

C. Atrophy of fat

D. Hyperplasia of skeletal muscle

Answer: C

Rationale:
Following placement of a tissue expander in the body, the following histologic changes
occur: thickened epidermis, decrease in thickness of dermis, no changes in hair follicles or
sebaceous glands, decrease in thickness of skeletal muscle, increase in capillaries, and fat
atrophy.

Reference:
Marks MW, Argenta LC. Skin expansion in reconstructive surgery. Facial Plastic Surg,
1988, 301-311.

Hoffman JF. Tissue expansion in the head and neck. Facial Plast Surg Clin North Am.
2005 May,:315-24.

American Board of Oral and Maxillofacial Surgery 133


2008 Oral and Maxillofacial Surgery Self Assessment Tool (OMSSAT)

Scar modification using a 60 Z-Plasty technique will increase the over length of the laceration
by how much?

A. 35%

B. 45%

C. 75%

D. 95%

Answer: C

Rationale:
Z-plasty is used to rearrange a wound. A 30' Z-plasty will increase the overall length of a
wound by 25%, a 45' Z-Plasty by 50%, and a 60' Z-plasty by 75%. The long axis of the
wound is rotated 90 degrees and the entire length of the incision is lengthened compared to
the length of the excised scar.

Reference:
Hove CR, William EF 3rd, Rogers BJ. Z-plasty: a concise review. Facial Plast Surg. 2001
Nov: 289-94.

Davis WE, Boyd JH. Z-Plasty. Otolaryngol Clin N Am 1990, 23:880-885.

American Board of Oral and Maxillofacial Surgery 134


2008 Oral and Maxillofacial Surgery Self Assessment Tool (OMSSAT)

What is the most commonly isolated organism found in mammalian animal bites?

A. Escherichia coli

B. Clostridium tetani

C. Pasteurella multocida

D. Fusobacterium nucleatum

Answer: C

Rationale:
Although multiple organisms have been isolated from animal bites, 50-75% of all bites
contain Pasteurella multocida. This aerobic organism is especially prevalent in cat bites.
Beta lactam antibiotics are indicated for the treatment of such wounds. Initial management
of such wounds should include Debridement and irrigation.

Reference:
Presutti RJ. Bite wounds. Early treatment and prophylaxis against infectious complications.
Postgrad Med. 1997 Apr; 101(4):243-4, 246-52, 254.

Fonseca. Oral and Maxillofacial Trauma. Chapter 27, 843-862. 2005.

Stefanopoulos PK, Tarantzopoulou AD. Facial bite wounds: management update.


Int J Oral Maxillofac Surg. 2005; 34:464-72.

American Board of Oral and Maxillofacial Surgery 135


2008 Oral and Maxillofacial Surgery Self Assessment Tool (OMSSAT)

What is the mechanism for development of coagulopathy following diffuse axonal injury (DAI)?

A. Lack of production of factor V

B. Release of tissue thromboplastin

C. Release of antithrombin III

D. Excessive production of prothrombin

Answer: B

Rationale:
DAI is a common occurrence following traumatic deceleration injuries of the brain.
Initially, DAI does not show specific CT or MRI findings. However with time, diffuse
edema if seen on CT scans. Release of tissue thromboplastin by damaged brain matter
signals the clotting cascade leading to depletion of coagulation factors. Coagulopathy is a
common finding following DAI.

Reference:
Advanced Trauma Life Support. American College of Surgeons Committee on Trauma.
Head Trauma, 181-206, 2005

Greenberg MS. Handbook of Neurosurgery. Head Trauma, 690-753, 1997

American Board of Oral and Maxillofacial Surgery 136


2008 Oral and Maxillofacial Surgery Self Assessment Tool (OMSSAT)

Which artery supplies the condylar head following a subcondylar fracture?

A. Masseteric

B. Medial pterygoid

C. Lateral pterygoid

D. Buccal

Answer: C

Rationale:
The axial blood supply to the mandibular condyle is via the lateral pterygoid artery. This
artery supplies the muscle which is intimately associated and attached to the condylar head
and neck.

Reference:
Fonseca. Oral and Maxillofacial Trauma. Basic Anatomy of the Head and Neck, 281-328,
2005.

Janfaza P et al. Surgical anatomy of the head and neck. Scalp, Cranium and Brain, p 49-
148, 2001.

American Board of Oral and Maxillofacial Surgery 137


2008 Oral and Maxillofacial Surgery Self Assessment Tool (OMSSAT)

How long should subluxed permanent teeth be treated with a flexible splint?

A. 1-2 weeks

B. 3-4 weeks

C. 5-6 weeks

D. 7-8 weeks

Answer: A

Rationale:
A flexible splint (acid-etched) should be used for 7 to 10 days on subluxed permanent
teeth. A short period of time is preferred to a longer period of time in order to prevent
future complications such as ankylosis. Subluxation is the defined as movement of the teeth
in any direction while concussion is the physiologic/pathologic sequela of a subluxation
that affects the pulpal tissue and surround PDL.

Reference:
Kaban LB, et. al. Pediatric Oral and Maxillofacial Surgery. Ch. 25 Facial Trauma II:
Dentoalveolar Injuries and Mandibular Fractures, Baumann A, et. al. p.446.

Daniels A, Backland L: Traumatic Dental Injuries: Current Treatment Concepts. JADA


129 (10):1401-14 1998OK

American Board of Oral and Maxillofacial Surgery 138


2008 Oral and Maxillofacial Surgery Self Assessment Tool (OMSSAT)

Which of the following is true regarding avulsed teeth?

A. Permanent teeth with apical foramina less than 1 mm diameter have a better prognosis than
those with greater an 1 mm diameter

B. Avulsed permanent teeth stored in Hanks solution have a more guarded prognosis than
those stored in saline

C. Replanted permanent teeth should be treated with a rigid splint

D. Avulsed primary teeth are not replanted

Answer: D

Rationale:
Permanent teeth with open apices greater than 1 mm diameter have a much better prognosis
than those with closed apices since there is increased potential for reestablishment of pulpal
circulation. Hank's solution is considered the ideal physiologic medium for avulsed teeth.
It contains sodium chloride, calcium chloride, potassium chloride and magnesium sulfate.
Avulsed primary teeth are never replanted since they have poor prognosis and may cause
ankylosis of the permanent tooth.

Reference:
Fonseca. Oral and Maxillofacial Trauma. Diagnosis and management of dentoalveolar
injuries, p 427-478, 2005.

OMS Reference Guide, Trauma/Emergencies, p 149-182, 2007.

American Board of Oral and Maxillofacial Surgery 139


2008 Oral and Maxillofacial Surgery Self Assessment Tool (OMSSAT)

Which type of injury to teeth carries the highest degree of pulpal necrosis?

A. Extrusion

B. Intrusion

C. Lateral luxation

D. Lingual luxation

Answer: B

Rationale:
Intrusive type injuries to teeth will cause greater compression and inflammatory injury of
the periapical tissues and therefore greater compromise of the pulpal vasculature and blood
flow causing an incidence of 65-90% pulpal necrosis. Extrusion can cause pulpal necrosis
in 64% of the time; Luxations account for the lowest incident of pulpal necrosis.

Reference:
Fonseca. Oral and Maxillofacial Trauma. Diagnosis and management of dentoalveolar
injuries, p 427-478, 2005.

Peterson. Principles of Oral and Maxillofacial Surgery. Management of alveolar and dental
fractures, p 383-400, 2006.

Andreasen JO. Luxation of permanent teeth due to trauma: a clinical and radiographic
follow-up study of 189 injured teeth. Scan J Dent Res 1970, 78: 273.

American Board of Oral and Maxillofacial Surgery 140


2008 Oral and Maxillofacial Surgery Self Assessment Tool (OMSSAT)

Which of the following root fractures has the best prognosis?

A. Horizontal fracture in the apical 1/3 of the root

B. Horizontal fracture in the coronal 1/3 of the root

C. Horizontal fracture in the middle portion of the root

D. Vertical fracture of the root

Answer: A

Rationale:
Fractures in the apical 1/3 of roots have the best prognosis for survival since the apical
portion of the root is completely embedded in alveolar bone and surrounding PDL.Serial
follow up evaluations are indicated. In many instances, endodontic therapy may not be
indicated. Vertical root fractures require extraction since the entire pulp chamber is injured
in this type of fracture.

Reference:
Fonseca. Oral and Maxillofacial Trauma. Diagnosis and management of dentoalveolar
injuries, p 427-478, 2005.

Peterson. Principles of Oral and Maxillofacial Surgery. Management of alveolar and dental
fractures, p 383-400, 2006.

American Board of Oral and Maxillofacial Surgery 141


2008 Oral and Maxillofacial Surgery Self Assessment Tool (OMSSAT)

What is the recommended treatment for permanent teeth with vertical root fractures?

A. Splinting the teeth for 2 weeks

B. Splinting the teeth for 6 weeks

C. No treatment is necessary

D. Extraction

Answer: D

Rationale:
All primary and permanent teeth with vertical root fractures must be extracted. These teeth
have poor prognosis.

Reference:
Fonseca. Oral and Maxillofacial Trauma. Diagnosis and management of dentoalveolar
injuries, p 427-478, 2005.

Peterson. Principles of Oral and Maxillofacial Surgery. Management of alveolar and dental
fractures, p 383-400, 2006.

American Board of Oral and Maxillofacial Surgery 142


2008 Oral and Maxillofacial Surgery Self Assessment Tool (OMSSAT)

Damage to which portion of frontal sinus would most likely involve the naso-frontal ducts?

A. Posterior wall

B. Medial portion of floor

C. Anterior wall

D. Lateral portion of floor

Answer: B

Rationale:
Damage to the medial portion of the frontal sinus most often involves damage to the naso-
frontal ducts. This is an important area to evaluate intraoperatively to evaluate the patency
of the naso-frontal ducts. Other areas mentioned do not involve the NFD's. Posterior wall
fractures involve the brain, and the lateral aspect of the sinus can involve the orbital roof.

Reference:
Fonseca. Oral and Maxillofacial Trauma. Evaluation and management of frontal sinus
injuries, p 721-736, 2005.

Peterson. Principles of Oral and Maxillofacial Surgery. Management of frontal sinus and
naso-orbitalethmoid complex fractures, 491-508, 2004.

American Board of Oral and Maxillofacial Surgery 143


2008 Oral and Maxillofacial Surgery Self Assessment Tool (OMSSAT)

During a Gillies approach to access the zygomatic arch, the plane of dissection is between which
two anatomic layers?

A. Superficial and deep layers of deep temporalis fascia

B. Temporoparietal fascia and superficial layer of deep temporalis fascia

C. Subcutaneous fat layer and temporoparietal fascia

D. Deep layer of deep temporalis fascia and temporalis muscle

Answer: D

Rationale:
The Gillies approach to reduce a zygomatic arch fracture utilizes a dissection between the
deep layer of the deep temporalis fascia and the temporalis muscle. The plane of dissection
is sub-fascial, but supra-muscular. Dissection in the temporoparietal fascia may damage
the facial nerve and will not reach the zygomatic arch. Since the two layers of the deep
temporal fascia split to envelop the arch, dissection must be maintained between the muscle
and deep layer of the deep temporal fascia in order to place the elevator deep to the arch.

American Board of Oral and Maxillofacial Surgery 144


2008 Oral and Maxillofacial Surgery Self Assessment Tool (OMSSAT)

Reference:
Fonseca. Oral and Maxillofacial. Trauma. Evaluation and management of frontal sinus
injuries, p 721-736, 2005.

Peterson. Principles of Oral and Maxillofacial Surgery. Management of frontal sinus and
naso-orbitalethmoid complex fractures, 491-508, 2004.

American Board of Oral and Maxillofacial Surgery 145


2008 Oral and Maxillofacial Surgery Self Assessment Tool (OMSSAT)

The marginal mandibular nerveis:

A. anterior to the facial artery, but not the vein, in 100% of cases.

B. anterior to the facial vein, but not the artery, in 100% of cases.

C. below the inferior border of the mandible in 19% of the cases where the nerve is posterior
to the facial vessels.

D. below the inferior border of the mandible in 5% of the cases where the nerve is anterior to
the facial vessels.

Answer: C

Rationale:
The classic 1961 study by Dingman and Grabb showed the relationship of the marginal
mandibular branch of the facial nerve in relation to the facial vessels. In 81% of the cases,
the nerve was superior to the inferior border of the mandible posterior to the vessels. In
19% of the cases, the nerve ran up to 1 centimeter inferior to the inferior border of the
mandible posterior to the vessels, and in 100% of cases, when the nerve was anterior to the
vessels, the marginal mandibular branch ran above the inferior border of the mandible.
Since the marginal mandibular branch runs from posterior to anterior, it cross the vertically
running facial artery and vein and therefore always is found to run anterior to both facial
vessels.

Reference:
Dingman RO, Grabb WC. Surgical anatomy of the mandibular ramus of the facial nerve based
on the dissection of 100 facial halves. Plastic Reconstrutive Surgery Vol 29, 1962.

Ziarah HA, Atkinson ME. The surgical anatomy of the mandibular distribution of the facial
nerve. Br J Oral Surg. 1981 Sep; 19(3):159-70.

American Board of Oral and Maxillofacial Surgery 146


2008 Oral and Maxillofacial Surgery Self Assessment Tool (OMSSAT)

The Keen technique is utilized to reduce fracture of what structure?

A. Mandibular condyle

B. Medial canthal tendon

C. Coronoid process

D. Zygomatic arch

Answer: D

Rationale:
Keen and Carmedy-Baxon approaches are the two classic methods of approaching and
reducing a zygomatic arch trans-orally. Buccal sulcus (vestibular) is the Keen approach,
while the lateral coronoid approach (along the ascending ramus) is known as the Carmedy-
Baxon. These two approaches allow reduction of a fractured arch The Keen approach
allows access to the infraorbital rim and nasomaxillary region as well while the Carmedy-
Baxon approach is somewhat more limited in exposure.

Reference:
Quinn JH. Lateral coronoid approach for intraoral reduction of fractures of the zygomatic
arch. J Oral Surg 35; 1977.

Courtney DJ. Upper buccal sulcus approach to management of fractures of the zygomatic
complex: a retrospective study of 50 cases. Br J Oral Maxillofac Surg, 37, 1999.

American Board of Oral and Maxillofacial Surgery 147


2008 Oral and Maxillofacial Surgery Self Assessment Tool (OMSSAT)

What is the most important bony suture for future growth when treating a naso-septal fracture in
the pediatric population?

A. Nasomaxillary

B. Nasofrontal

C. Septovomerine

D. Frontoethmoidal

Answer: C

Rationale:
The septovomerine junction is considered a growth center in the pediatric nose. Therefore,
adequate reduction of this region is imperative to decrease risk of premature ossification
which can lead to future growth disturbance.

Reference:
Haug RH.. Maxillofacial injuries in the pediatric patient. Oral Surg, Oral Medicine, Oral
Path, 90, 126, 2000.

Rock WP, Brain DJ. The effects of nasal trauma during childhood upon growth of the nose
and midface. Br J Orthod 10: 38, 1983.

American Board of Oral and Maxillofacial Surgery 148


2008 Oral and Maxillofacial Surgery Self Assessment Tool (OMSSAT)

What is the most important factor in re-establishment of the vertical facial height in the
management of pan-facial fractures?

A. Reduction of mandibular condyle fractures

B. Reduction of fronto-zygomatic sutures

C. Establishing proper zygomatic projection

D. Establishing proper dental occlusion

Answer: A

Rationale:
There are several key landmarks when treating pan-facial injuries. While re-establishing
proper occlusion is a key component, the posterior facial height and ramus/condyle units
must be aligned properly in order to gain vertical height of the face. This is accomplished
by proper reduction of mandibular condyles after proper occlusion is established. If teeth
are not present, then anatomic reduction of the mandible and/or maxilla must be performed
in order to establish proper vertical height. Once proper occlusion and vertical ramus
height are established, alignment of vertical and horizontal facial bony buttresses is
accomplished.

Reference:
Peterson. Principles of Oral and Maxillofacial Surgery. Management of panfacial fractures,
547-562, 2004.

Markowitz BL, Manson PN. Panfacial fractures: organization of treatment. Clin Plast
Surg 16; 105, 1989.

American Board of Oral and Maxillofacial Surgery 149


2008 Oral and Maxillofacial Surgery Self Assessment Tool (OMSSAT)

Between which two anatomic layers is the safest plane of dissection during the initial elevation
of a coronal flap?

A. Subcutaneous fat and galea

B. Pericranium and galea

C. Subcutaneous fat and temporoparietal fascia

D. Galeal fat pad and temporal fat pad

Answer: B

Rationale:
The safest plane of dissection in a coronal flap elevation is the subgaleal plane located
between the galea and the pericranium. At this level, the frontal branch is above (superior)
the level of the dissection and thus is less susceptible to injury.

Reference:
Frodel JL, Marentette LJ. The coronal approach. Anatomic and technical considerations
and morbidity. Arch Otolaryngol Head Neck Surg. 1993 Feb; 119(2):140.

Liebman EP, Webster RC, Berger AS, et al. The frontalis nerve in the temporal brow lift.
Arch Otolaryngol 1982 Apr; 108(4):232-5.

American Board of Oral and Maxillofacial Surgery 150


2008 Oral and Maxillofacial Surgery Self Assessment Tool (OMSSAT)

What is the most frequently fractured area of the edentulous mandible?

A. Condyle

B. Subcondyle

C. Symphysis

D. Body

Answer: D

Rationale:
43.5% of all fractures of the edentulous mandible occur in the body region. This is
compared to only 33% in the dentate patient. The body fracture is the most frequently
fractured segment of the edentulous mandible.

Reference:
Bruce RA, Strachan DS. Fractures of the edentulous mandible: the Chalmers j. Lyons
Academy stud. J Oral Surg, 34:9073, 1976.

Peterson 2nd edition, Principles of Oral and Maxillofacial Surgery, Principles of


management of mandibular fractures, 401-434, 2004

Fonseca, Oral and Maxillofacial Trauma, Mandibular fractures, P 479-522, 2005

American Board of Oral and Maxillofacial Surgery 151


2008 Oral and Maxillofacial Surgery Self Assessment Tool (OMSSAT)

What is the mostly commonly used endoscope (offset degree of angulation and diameter) for
repair of mandibular condyle fractures?

A. 30 degrees, 2mm

B. 0 degrees, 2 mm

C. 30 degrees, 4mm

D. 0 degrees, 4 mm

Answer: C

Rationale:
Although endoscopically assisted condyle repairs are not as commonly performed as the
more traditional methods, the most commonly used endoscope is a 30', 4 mm endoscope.
This technique requires making a Rison incision and inserting an endoscope while reducing
the fracture. Application of fixation can be done directly or via a trocar system.

Reference:
Martin M, Lee C. Endoscopic mandibular condyle fracture repair. Atlas Oral Maxillofac
Surg Clin N Am, 11, 2003.

Troulis MJ. Endoscopic open reduction and internal rigid fixation of subcondylar
fractures. J Oral Maxillofac Surg 62, 2004.

American Board of Oral and Maxillofacial Surgery 152


2008 Oral and Maxillofacial Surgery Self Assessment Tool (OMSSAT)

Which of the following differentiates between superior orbital fissure syndrome and orbital apex
syndrome?

A. Ophthalmoplegia

B. Loss of vision

C. Ptosis of the upper lid

D. Anesthesia of the forehead

Answer: B

Rationale:
Loss of vision is the defining clinical sign which differentiates superior orbital fissure
syndrome from orbital apex syndrome. All other signs and symptoms listed are seen in
both conditions. Ophthalmoplegia occurs due to involvement of cranial nerves III, IV, VI.
Anesthesia occurs due to the involvement of V1. Ptosis occurs due to loss of sympathetic
tone to the Muller's muscle; since the terminal sympathetic fibers travel with the
ophthalmic branch of the trigeminal nerve (V1.)

Reference:
Peter ward Booth (ed); Maxillofacial trauma; Periorbital and intraorbital trauma and orbital
reconstruction, P 205-222, Churchhill Livingstone; 2003

Eo S, Kim JY, Azari K. Temporary orbital apex syndrome after repair of orbital wall
fracture. Plast Reconstr Surg. 2005 Oct; 116(5):85e-89e

American Board of Oral and Maxillofacial Surgery 153


2008 Oral and Maxillofacial Surgery Self Assessment Tool (OMSSAT)

Which of the following can cause binocular diplopia?

A. Retinal detachment

B. Lens dislocation

C. Corneal scarring

D. Alteration in globe position

Answer: D

Rationale:
Binocular diplopia is more common than monocular diplopia; however both can result
from traumatic insult to the globe. Retinal detachment, lens dislocation, cataracts, and
corneal scarring are causes of monocular diplopia. Globe position changes can lead to
binocular diplopia.

Reference:
Peter ward Booth (ed); Maxillofacial trauma Periorbital and intraorbital trauma and orbital
reconstruction, P 205-222, Churchhill Livingstone; 2003

Michael Miloro (ed); Peterson's principles of Oral and Maxillofacial Surgery 2nd edition,
Orbital and ocular trauma, P 463-490, BC Decker Inc, 2004

American Board of Oral and Maxillofacial Surgery 154


2008 Oral and Maxillofacial Surgery Self Assessment Tool (OMSSAT)

What is the mean distance of the optic canal from the posterior ethmoidal foramen?

A. 1mm

B. 5mm

C. 10 mm

D. 12mm

Answer: B

Rationale:
When measuring from the anterior lacrimal crest the mean distance of the anterior and
posterior ethmiodal foramen are 24 and 36mm respectively, the optic canal is a mean of
5mm posterior to the posterior ethmiodal foramen (or a mean of 42mm from the anterior
lacrimal crest).

Reference:
Michael Miloro (ed); Peterson's principles of Oral and Maxillofacial Surgery 2nd edition,
BC Decker Inc, Orbital and ocular trauma, P 463-490,2004,

Ray Fonseca (ed); Oral and Maxillofacial Surgery, volume 3; Orbital Trauma, p 205-244,
WB Saunders, 2000

American Board of Oral and Maxillofacial Surgery 155


2008 Oral and Maxillofacial Surgery Self Assessment Tool (OMSSAT)

What is the greatest width of an upper eye lid defect that can be repaired by primary closure
without compromising function?

A. 10%

B. 15%

C. 20%

D. 25%

Answer: D

Rationale:
Eyelid injuries involving less then 25% can be closed primarily, those which are 25-50%
can be repaired with local tissue advancement. Those greater then a 50% defect will
require a flap or skin graft which replaces both the anterior and posterior lamellae.

Reference:
Michael Miloro (ed); Peterson's principles of Oral and Maxillofacial Surgery 2nd edition,
BC Decker Inc, Orbital and ocular trauma, P 463-490, 2004

Ray Fonseca (ed); Oral and Maxillofacial Surgery, volume 3; Orbital Trauma, p 205-
244WB Saunders, 2000

American Board of Oral and Maxillofacial Surgery 156


2008 Oral and Maxillofacial Surgery Self Assessment Tool (OMSSAT)

A 30 year-old male has bilateral lower leg deformities following a motor vehicle collision. He is
anxious, with the follwing vital signs: BP = 130/100, pulse = 110, respiratory rate = 28What is
his class of blood loss?

A. Class I

B. Class II

C. Class III

D. Class IV

Answer: B

Rationale:
Class II Hemorrhage represents volume loss of 750 to 1500ml of blood. Clinical symptoms
include tachycardia, tachypnea and a decrease in pulse pressure. This decrease in pulse
pressure is primarily related to a rise in the diastolic component due to an increase in
circulating catecholamines which increase the vascular tone and resistance. Systolic
pressure changes minimally in early hemorrhagic shock.

Reference:
Advanced Trauma Life Support Student Manual. Shock, 87-107, American College of
Surgeons. Sixth Edition. 1997.

Miloro, M. et al, Peterson's Principles of Oral and Maxillofacial Surgery, Second Edition,
Initial management of the trauma patient, p 327-356, BC Decker 2004.

American Board of Oral and Maxillofacial Surgery 157


2008 Oral and Maxillofacial Surgery Self Assessment Tool (OMSSAT)

A 25 year-old male presents unconscious following a fall. Neurological evaluation shows that he
withdraws from pain, there is no verbal response, and there is no eye opening. What is his
Glasgow Coma Score?

A. 4

B. 5

C. 6

D. 7

Answer: C

Rationale:
Glasgow Coma Scale is based on three variables: best motor response, best verbal
response and eye opening. (GCS = M+V+E) Scores range from 3 to 15. In this case
withdrawing from pain represents 4, no verbal response represents 1 and no eye opening
represents 1. GCS = (M4+V1+E1) = 6

Reference:
Advanced Trauma Life Support Student Manual. Shock, 87-107, American College of
Surgeons. Sixth Edition. 1997.

American Board of Oral and Maxillofacial Surgery 158


2008 Oral and Maxillofacial Surgery Self Assessment Tool (OMSSAT)

Miloro, M. et al, Peterson's Principles of Oral and Maxillofacial Surgery, Second Edition,
Initial management of the trauma patient, p 327-356, BC Decker 2004.
.

American Board of Oral and Maxillofacial Surgery 159


2008 Oral and Maxillofacial Surgery Self Assessment Tool (OMSSAT)

After a severe head trauma, the intracranial pressure of 45 year-old male is 30mm Hg. Other vital
signs include pulse 90, respiratory rate 20, and blood pressure 130/85. What is the cerebral
perfusion pressure of this patient?

A. 50

B. 60

C. 70

D. 80

Answer: C

Rationale:
Cerebral blood flow, maintained by autoregulation, depends on cerebral perfusion pressure
and intracranial pressure. Cerebral perfusion pressure is the mean arterial blood pressure
minus intracranial pressure. (CPP = MAP ICP). Normal CPP in an adult is >50 mm Hg.

One method for the calculation of the mean arterial pressure is diastolic pressure times 2
plus systolic pressure all divided by three.
{MAP = [(2 x Diastolic Pressure) + Systolic Pressure] / 3}
=[(2 x 85) + 130] / 3
=100

CPP = 100 30
CPP = 70

Reference:
Advanced Trauma Life Support Student Manual. Shock, 87-107American College of
Surgeons. Sixth Edition. 1997.

Way, L et al. Current Surgical Diagnosis and Treatment. Fluid and electrolyte
management, 129-142, Eleventh Edition. McGraw Hill. 2003.

American Board of Oral and Maxillofacial Surgery 160


2008 Oral and Maxillofacial Surgery Self Assessment Tool (OMSSAT)

A 2 year-old male presents with second degree burns of his entire head. What percentage of his
body surface area is involved?

A. 4.5%

B. 9%

C. 10%

D. 18%

Answer: D

Rationale:
The Rule of Nines is a useful and practical guide to determine the extent of the burn and
fluid resuscitation. The infant's head represents a larger proportion of the surface area. The
percentage of total body surface of the infant's head is twice that of the normal adult (which
is 9%).

Reference:
Advanced Trauma Life Support Student Manual. Pediatric trauma, p 289-311, American
College of Surgeons. Sixth Edition. 1997.

Way, L et al. Current Surgical Diagnosis and Treatment. Fluid and electrolyte
management, 129-14 Eleventh Edition. McGraw Hill. 2003.

American Board of Oral and Maxillofacial Surgery 161


2008 Oral and Maxillofacial Surgery Self Assessment Tool (OMSSAT)

Which of the following is a component of Becks triad?

A. Wide pulse pressure

B. Decreased central venous pressure

C. Increased systolic arterial pressure

D. Muffled heart tones

Answer: D

Rationale:
Beck's triad for diagnosis of cardiac tamponade consists of venous pressure elevation,
decline in arterial pressure and muffled heart tones. Wide pulse pressure is seen in shock.

Reference:
Advanced Trauma Life Support Student Manual. Thoracic trauma, p 125-141, American
College of Surgeons. Sixth Edition. 1997.

Miloro, M. et al, Peterson's Principles of Oral and Maxillofacial Surgery, Second Edition,
Initial management of the trauma patient, p 327-356BC Decker 2004.

American Board of Oral and Maxillofacial Surgery 162


2008 Oral and Maxillofacial Surgery Self Assessment Tool (OMSSAT)

What is the most common midfacial fracture in the pediatric population?

A. Orbital roof

B. Orbital floor

C. Zygomatico-maxillary

D. Nose

Answer: D

Rationale:
The nose is a prominent structure in children and the nasal bones are fragile. Nasal
fractures are the most common midfacial skeleton injury in children. The low incidence of
midfacial fractures in children can also be explained by the elasticity of the child's facial
bones, the retrusive position of the maxilla, nose and infraorbital rims and the anatomic
protection afforded by the cranium.

Reference:
Kaban LB, et. al. Pediatric Oral and Maxillofacial Surgery. Ch. 24 Facial Trauma I:
Midfacial Fractures, Baumann A, et. al. p.426.

2 Posnick JC. Craniofacial and Maxillofacial Surgery in Children and Young Adults. Ch
30 Primary Craniomaxillofacial Fracture Management, Posnick JC. p.720.

American Board of Oral and Maxillofacial Surgery 163


2008 Oral and Maxillofacial Surgery Self Assessment Tool (OMSSAT)

What is the most commonly fractured area of the mandible in pediatric patients?

A. Symphysis

B. Parasymphysis

C. Coronoid process

D. Condyle

Answer: D

Rationale:
Condyle fractures have been reported to occur most commonly in pediatric patients (15-
60%). With increasing age, the angle is involved more commonly.

Anatomic Distribution of Mandibular Injury in Pediatric Patient


Condyle 15-60%
Alveolus 8-50.6%
Body 6-44%
Symphysis 2-40%
ParaSymph 23-33%
Angle 3-27%
Ramus 1-10%
Coronoid 0-19%

Reference:
Kaban LB, et. al. Pediatric Oral and Maxillofacial Surgery. Ch. 25 Facial Trauma II:
Dentoalveolar Injuries and Mandibular Fractures, Baumann A, et. al. p.446.

Haug R, Foss J. Maxillofacial injuries in the pediatric patient. Oral Surgery, Oral
Medicine, Oral Pathology, Oral Radiology & Endodontics, 90:2, 2000, p.131.

Rowe NL, Williams JL. Maxillofacial Injuries. Ch 13 Maxillofacial Injuries in Children,


James D. p.388.

American Board of Oral and Maxillofacial Surgery 164


2008 Oral and Maxillofacial Surgery Self Assessment Tool (OMSSAT)

What explains the relative paucity of pediatric midfacial trauma compared to adults?

A. Large paranasal sinuses

B. Rigid facial bones

C. Prominent midfacial structures

D. Size of cranium

Answer: D

Rationale:
The low incidence of midfacial fractures in children can be explained by the elasticity of
the child's facial bones, the retrusive position of the maxilla, nose and infraorbital rims and
the anatomic protection afforded by the cranium.

Reference:
Kaban LB, et. al. Pediatric Oral and Maxillofacial Surgery. Ch. 24 Facial Trauma I:
Midfacial Fractures, Baumann A, et. al. p.425.

Posnick JC. Craniofacial and Maxillofacial Surgery in Children and Young Adults. Ch 30
Primary Craniomaxillofacial Fracture Management, Posnick JC. p.704, 716.

American Board of Oral and Maxillofacial Surgery 165


2008 Oral and Maxillofacial Surgery Self Assessment Tool (OMSSAT)

Which of the following is a consideration for open reduction and internal fixation in pediatric
patients with condyle fractures?

A. Minor malocclusion in the mixed dentition

B. Minor malocclusion in the primary dentition

C. Persistent malocclusion in the permanent dentition

D. To restore vertical height of the ramus regardless of age or occlusion

Answer: C

Rationale:
Open reduction internal fixation (ORIF) is rarely indicated in the pediatric population.
Most condylar fractures can be treated with a combination of intermaxillary fixation or
analgesics and soft diet. Minor occlusal discrepancies will equilibrate with continued
growth and dental eruption. Children in the permanent dentition who have persistent
malocclusion at the time of initial treatment or after a period of intermaxillary fixation
should be considered for ORIF to restore ramal height and occlusal function.

Reference:
Kaban LB, et. al. Pediatric Oral and Maxillofacial Surgery. Ch. 25 Facial Trauma II:
Dentoalveolar Injuries and Mandibular Fractures, Baumann A, et. al. p.457.

Rowe NL, Williams JL. Maxillofacial Injuries. Ch 13 Maxillofacial Injuries in Children,


James D. p.393.

American Board of Oral and Maxillofacial Surgery 166


2008 Oral and Maxillofacial Surgery Self Assessment Tool (OMSSAT)

How do pediatric hemorrhagic shock and circulatory decompensation differ from the adult?

A. There are superior compensatory mechanisms in pediatric patients

B. Hemorrhagic shock occurs gradually in children despite rapid blood loss

C. Hemorrhagic shock occurs more suddenly, as vasoconstriction, tachycardia, and


myocardial contractility can mask signs and symptoms

D. Hemorrhagic shock rarely occurs in the pediatric population because of a higher body
surface area to mass ratio

Answer: C

Rationale:
Management of the pediatric trauma patient differs from that of the adult. Lack of
compensatory reserves in the pediatric patients can lead to a sudden cardiovascular
collapse. This occurs rather quickly since tachycardia, vasoconstriction and increased
myocardial contractility will initially mask the presenting signs and symptoms. Body
surface ratio does not affect shock response, but does affect both heat and insensible fluid
loss in the pediatric patient.

Reference:
Haug R, Foss J. Maxillofacial injuries in the pediatric patient. Oral Surgery, Oral
Medicine, Oral Pathology, Oral Radiology & Endodontics, 90:2, 2000, p.128.

Miloro M, et. al. Peterson's Principles of Oral and Maxillofacial Surgery. Ch 27 Pediatric
Craniomaxillofacial Fracture Management, Posnick JC, et. al. p.528.

American Board of Oral and Maxillofacial Surgery 167


2008 Oral and Maxillofacial Surgery Self Assessment Tool (OMSSAT)

A trauma patient has multiple facial lacerations heavily contaminated with soil. He has
completed a tetanus immunization series but has not received a booster for the past 7 years. How
should this patient be managed?

A. Tetanus toxoid booster injection

B. Tetanus booster and tetanus immunoglobulin

C Tetanus immunoglobulin

D. Tetanus booster, tetanus immunoglobulin plus full course of immunization

Answer: A

Rationale:
In a "clean" (minimally or noncontaminated) would in a previously immunized patient who
has not received a booster in the past 10 years, a booster dose of 0.5 ml of toxoid booster is
recommended. This timeline decreases to 5 years for a "dirty" or contaminated wound. If
there is no history of immunization or an unknown history, the tetanus immunization series
is begun; with the addition of tetanus immune globulin (TIG) for dirty wounds.

American Board of Oral and Maxillofacial Surgery 168


2008 Oral and Maxillofacial Surgery Self Assessment Tool (OMSSAT)

Reference:
Fonseca Oral & Maxillofacial Trauma, 3rd edition; Management of soft tissue injuries, P
751-820, Gomella Clinician's Pocket Reference, 9th Edition; p 330

American Board of Oral and Maxillofacial Surgery 169


2008 Oral and Maxillofacial Surgery Self Assessment Tool (OMSSAT)

Indication for removal of teeth in the line of mandibular fractures includes which of the
following?

A. Complete bony impacted wisdom teeth in the line of fracture

B. Teeth with exposed root apices in the fracture site

C. Teeth with coronal fractures

D. Immobile teeth in line of fracture

Answer: B

Rationale:
Relative indications for removal of teeth in the line of mandibular fractures include:
Presence of obvious pathology (caries, periodontal disease)
Gross mobility of involved teeth
Teeth that prevent adequate reduction of fracture
Teeth with fractured roots
Teeth with exposed apices

Reference:
Peterson 2nd edition, Principles of management of mandibular fractures. P 401-434,

Fonseca, Oral and Maxillofacial Trauma, Mandibular fractures, P 479-522, 2005,

American Board of Oral and Maxillofacial Surgery 170


2008 Oral and Maxillofacial Surgery Self Assessment Tool (OMSSAT)

Which of the following is a relative contraindication for closed reduction of a mandible fracture?

A. Poorly controlled seizure disorder

B. Poorly controlled renal disease

C. Well-controlled psychiatric disorder

D. Well-controlled chronic obstructive pulmonary disease

Answer: A

Rationale:
Poorly controlled seizure disorders, mental retardation, and psychiatric disorders are
relative contraindication for closed reduction of mandible fractures in which
maxillomandibular fixation would be poorly tolerated. \. If possible, an attempt should be
made to perform an open reduction with rigid fixation for these patients.

Reference:
Peterson 2nd edition, Principles of management of mandibular fractures. P 401-434, 2004

Fonseca, Oral and Maxillofacial Trauma, Mandibular fractures, P 479-522, 2005

American Board of Oral and Maxillofacial Surgery 171


2008 Oral and Maxillofacial Surgery Self Assessment Tool (OMSSAT)

What is the recommended treatment for a patient with an intracapsular condylar head fracture
and normal occlusion?

A. Observation

B. 2 weeks of intermaxially fixation

C. 6 weeks of intermaxially fixation

D. 8 weeks of intermaxillary fixation

Answer: A

Rationale:
Intracapsular condylar head fractures with a normal occlusion should be observed. These
patients do not need intermaxillary fixation since there is no change in occlusion. The
management also includes soft dies, pain control and range of motion exercise to regain
normal interincisal opening after the initial healing period.

Reference:
Peterson 2nd edition, Principles of management of mandibular fractures. P 401-434, 2004

Fonseca, Oral and Maxillofacial Trauma, Mandibular fractures, P 479-522, 2005

American Board of Oral and Maxillofacial Surgery 172


2008 Oral and Maxillofacial Surgery Self Assessment Tool (OMSSAT)

Which of the following is a contraindication for exploration of an orbital floor fracture?

A. Enophthalmos

B. Grade III Hyphema

C. Persistant diplopia

D. Limitation of extraocular muscle function

Answer: B

Rationale:
Hyphema is a contraindication to orbital exploration due to the possibility of re-bleeding
into the anterior chamber, which can lead to corneal staining and glaucoma.

Reference:
Peterson 2nd edition, Principles of Oral & Maxillofacial Surgery, Orbital and ocular
trauma, P 463-490, 2004

Brandt MT, Haug RH. Traumatic hyphema: a comprehensive review. J Oral Maxillofac
Surg, 59, 1462, 2001.

American Board of Oral and Maxillofacial Surgery 173


2008 Oral and Maxillofacial Surgery Self Assessment Tool (OMSSAT)

Which of the following is least likely to cause diplopia?

A. Chemosis

B. Hematoma

C. Extraocular muscle entrapment

D. Abducens nerve injury

Answer: A

Rationale:
Chemosis is edema of palpebral and bulbar conjuctiva. This does not lead to diplopia.
Other conditions including hematoma, muscle entrapment and a nerve injury will cause
double vision.

Reference:
Peterson 2nd edition, Principles of Oral and Maxillofacial Surgery, Orbital and ocular
trauma, P 463-490, 2004

Fonseca, Oral and Maxillofacial Trauma, ophthalmic consequences of maxillofacial


trauma, p 693-720, 2005

American Board of Oral and Maxillofacial Surgery 174


2008 Oral and Maxillofacial Surgery Self Assessment Tool (OMSSAT)

Which of the following is the clearest indication for open reduction of a mandibular condyle
fracture?

A. Unilateral condylar fractures with comminuted upper face fracture

B. Displacement into middle cranial fossa

C. Medial extracapsular dislocation of the condylar head

D. Isolated condylar neck fracture

Answer: B

Rationale:
Zide's criteria for open reduction of condyle fractures includes the following relative and
absolute indications:

American Board of Oral and Maxillofacial Surgery 175


2008 Oral and Maxillofacial Surgery Self Assessment Tool (OMSSAT)

Reference:
Zide MF. Open reduction of mandibular condyle fractures. Indications and technique. Clin
Plast Surg. 1989 Jan; 16(1):69-76.

Peterson 2nd edition, , Principles of management of mandibular fractures. P 401-434,


2004

Fonseca, Oral and Maxillofacial Trauma, Mandibular fractures, P 479-522, 2005

American Board of Oral and Maxillofacial Surgery 176


2008 Oral and Maxillofacial Surgery Self Assessment Tool (OMSSAT)

A patient has a deep, vertical laceration on his face extending from the mid lower lid to the
ipsilateral oral commissure. Physical examination reveals weakness of the buccal branches of
the facial nerve. How should this patient be managed?

A. Explore the wound to identify and repair the injured nerve

B. Irrigate and close the wound without nerve exploration

C. Repair the laceration and wait two weeks for return of function, if no improvement in
function, then explore and repair

D. Consider nerve grafting from the great auricular nerve

Answer: B

Rationale:
Facial nerve injuries medial to a vertical line from the lateral canthus do not need to be
explored because there is increased terminal facial nerve branch arborization and therefore
multiple small branches exist. Exploration of such small branches is not practical. This
wound should be managed as any other facial laceration.

Reference:
Peterson 2nd edition, Principles of Oral & Maxillofacial Surgery, Soft tissue injuires, P
357-370, 2004

Fonseca, Oral and Maxillofacial Trauma, Applied surgical anatomy of the head and neck, p
281-328, 2005,

American Board of Oral and Maxillofacial Surgery 177


2008 Oral and Maxillofacial Surgery Self Assessment Tool (OMSSAT)

Following repair of pan-facial fractures, a patient develops diplopia, orbital proptosis, and
reports an unusual buzzing sound. What is the most likely diagnosis?

A. Cavernous sinus thrombosis

B. Orbital apex syndrome

C. Carotid-cavernous fistula

D. Temporal arteritis

Answer: C

Rationale:
Carotid-cavernous fistulae (CCF) are potential complications of craniofacial trauma. Signs
and symptoms are related to the abnormal flow of blood (shunting) between a high flow
(internal carotid) to a low flow system (cavernous sinus). Patients often complain of a
swollen red eye, orbital pain, loud buzzing sound and swishing sound, proptosis and visual
changes. Although cavernous sinus thrombosis, orbital apex syndrome and temporal
arteritis may have visual disturbances, none will display an unusual buzzing sound. CCF
can occur within days to weeks following trauma.

Reference:
Origitano TC, Al-Meffy O. Aneurysms of the cavernous sinus: treatment options and
consideration. In: Youmans RJ. Neurological Surgery. Philadelphia: WB Saunders,
1995, 1320-1335.

Fattahi T, Brandt MT, Jenkins WS, et al. Traumatic carotid-cavernous fistula:


pathophysiology and treatment. J Craniofacial Surg, 14:1, 2003.

American Board of Oral and Maxillofacial Surgery 178


2008 Oral and Maxillofacial Surgery Self Assessment Tool (OMSSAT)

Injury to which zone of the neck is most amenable to physical examination?

A. Zone 1

B. Zone 2

C. Zone 3

D. Zone 4

Answer: B

Rationale:
Penetrating neck injuries can have significant morbidity and mortality. Zone 1 extends
from the clavicle to the cricoid. Zone 2 is from the cricoid to the inferior border of the
mandible. Zone 3 is from the mandibular angle to the skull base. Physical examination of
zone 2 is most easily accomplished due to its location and ease of access. While studies in
addition to physical exam may be indicated in Zone 2 penetrating injuries, Zones 1 and 3
more often require further diagnostic tools such as an arteriogram or esophagoscopy.

Reference:
Advanced Trauma Life Support Student Manual. Initial assessment and management, P
21-46, American College of Surgeons. Sixth Edition. 1997.

Miloro, M. et al, Peterson's Principles of Oral and Maxillofacial Surgery, Second Edition,
Initial management of the trauma patient, p 327-356, BC Decker 2004, chapter 18

American Board of Oral and Maxillofacial Surgery 179


2008 Oral and Maxillofacial Surgery Self Assessment Tool (OMSSAT)

What is the most likely position of a displaced mandibular condylar fracture?

A. Superior and lateral

B. Superior and medial

C. Anterior and medial

D. Anterior and lateral

Answer: C

Rationale:
Condylar displacement usually occurs in an anterior and medial direction secondary to the
lateral ptyergoid muscle pull. The patient will have a premature bite on the ipsilateral side
and upon opening will deviate to the ipsilateral side as well.

Reference:
Peterson 2nd edition, Principles of Oral and Maxillofacial Surgery, Principles of
management of mandibular fracture, p 401-434, 2004

Fonseca, Oral and Maxillofacial Trauma, Mandibular fractures, p 479-522, 2005

American Board of Oral and Maxillofacial Surgery 180


2008 Oral and Maxillofacial Surgery Self Assessment Tool (OMSSAT)

Formation of a cartilaginous callus following an initial formation of a hematoma marks what


type of bone healing?

A. Initial

B. Primary

C. Secondary

D. Tertiary

Answer: C

Rationale:
Bone healing has been divided into 2 categories: primary and secondary. Primary bone
healing is characterized by less than 1 mm gap between bone edges, no bony mobility and
no callus formation. This type of healing occurs following placement of load-bearing
plates. Secondary bone healing occurs with formation of a hematoma, fibroblast

American Board of Oral and Maxillofacial Surgery 181


2008 Oral and Maxillofacial Surgery Self Assessment Tool (OMSSAT)

infiltration, cartilaginous and bony callus formation, and then remodeling. This type of
healing occurs following closed reduction of fractures.

Reference:
Peterson 2nd edition, Principles of Oral and Maxillofacial Surgery, Principles of
management of mandibular fracture, p 401-434, 2004

Fonseca, Oral and Maxillofacial Trauma, Mandibular fractures, p 479-522, 2005

Abubaker OA. Oral and Maxillofacial Surgery Secrets. Maxillofacial trauma, 197-206
2001

American Board of Oral and Maxillofacial Surgery 182


2008 Oral and Maxillofacial Surgery Self Assessment Tool (OMSSAT)

Maximum interincisal opening of 5 mm immediately following 3 weeks of maxillo-mandibular


fixation is best explained by?

A. Bony ankylosis

B. Fibrous ankylosis

C. Muscle splinting

D. Hypertrophy of muscles of mastication

Answer: C

Rationale:
In the mandible, 3 weeks of IMF for an adult condyle fracture is just slightly longer than
usual. However, it is not long enough to cause a fibrous or bony ankylosis. Hypomoblity
following closed reduction is due to muscle splinting and guarding. This responds very
well to physiotherapy. Ankylosis is a major complication of prolonged IMF, especially in
children.

Reference:
Peterson 2nd edition, Principles of Oral and Maxillofacial Surgery, Principles of
management of mandibular fracture, p 401-434, 2004

Fonseca, Oral and Maxillofacial Trauma, Mandibular fractures, p 479-522, 2005

American Board of Oral and Maxillofacial Surgery 183


2008 Oral and Maxillofacial Surgery Self Assessment Tool (OMSSAT)

Which patient is best treated with a superficial chemical peel:

A. 45 year-old with actinic keratoses and lentigines.

B. 70 year-old with severe photoaging and wrinkles.

C. 28 year-old with a localized hypertrophic skin scars.

D 16 year-old with acne scarring and active herpes labialis.

Answer: A

Rationale:
Chemical peels are a well-accepted form of treatment for skin rejuvenation. They have the
advantages of a long-standing safety and efficacy record, performed with ease, low cost,
and relatively quick recovery time. Various acidic and basic compounds are used to
produce a controlled skin injury and can be classified as superficial, medium, and deep
chemical peels according to their level of penetration, tissue destruction, and inflammation.
Superficial peels involve penetration of only the epidermis and papillary dermis and are
indicated in the treatment of mild acne and its post-inflammatory erythema, mild
photoaging (Glogau I, II), and epidermal growths like actinic keratoses and lentigines as
well as melasma and other pigmentary dyschromias. Multiple sessions are usually required
for optimal results. A 70-year old patient with severe photoaging and dynamic wrinkles is a
Glogau Class IV patient, and superficial peel treatment will not have any beneficial effect.
Hypertrophic skin scars are usually treated with intralesional steroid injections, laser
therapy, dermabrasion, or surgical scar revision. Active herpes labialis is a contraindication
to chemical and laser skin rejuvenation treatment.

American Board of Oral and Maxillofacial Surgery 184


2008 Oral and Maxillofacial Surgery Self Assessment Tool (OMSSAT)

Reference:
Glogau classification of photoaging: Reproduced from: Monheit GD: Skin rejuvenation
procedures. Principles of Oral and Maxillofacial Surgery. 2nd edition, Volume 2, BC
Decker Inc, London, P 1419:

Demas PN, Braun TW: Chemical skin resurfacing. Esthetic surgery of the aging face.
Oral and Maxillofacial Surgery Clinics of North America. Vol 6(8), 1998. P 2

Monheit GD: Skin rejuvenation procedures. Principles of Oral and Maxillofacial Surgery.
2nd edition, Volume 2, BC Decker Inc, London, P 1419

American Board of Oral and Maxillofacial Surgery 185


2008 Oral and Maxillofacial Surgery Self Assessment Tool (OMSSAT)

A facial skin chemical peel technique combining 30% trichloroacetic acid and Jessners solution
is expected to penetrate to which level?

A. Epidermis

B. Papillary dermis

C. Reticular dermis

D. Subcutaneous tissue

Answer: C

Rationale:
a) Epidermis superficial layer of skin and it is penetrated by all agents. Lower
concentrations of superficial peeling agents may be confined to this layer.
b) Papillary dermis this level penetration is attained following application of superficial
peeling agents like TCA (10-30%), Glycolic acid (10-30%) and Jessner's solution.
c) Reticular dermis The upper reticular dermis is penetrated by medium depth peeling
agents (TCA 30-50%, Jessner's plus 30 %TCA, 88% Phenol) and the mid-reticular dermis
is penetrated by deep peeling agents like Baker's phenol and Litton's phenol).
d) Subcutaneous tissue: This depth would suggest a full thickness skin injury that would
result in significant scarring. There are no formulated peeling agents designed for
predictable penetration to this level.

Reference:
Cross section of skin with superficial, medium and deep wound areas: Reproduced from
Monheit GD: Skin rejuvenation procedures. Principles of Oral and Maxillofacial Surgery.
2nd edition, Volume 2, BC Decker Inc, London, P 1420

American Board of Oral and Maxillofacial Surgery 186


2008 Oral and Maxillofacial Surgery Self Assessment Tool (OMSSAT)

Nahai F, Baker TJ, Stuzin JM. Chemical Peel. The Art of Aesthetic Surgery. St Louis:
Quality Medical Publishing, 2005, p 386.

Demas PN, Braun TW: Chemical Skin Resurfacing. Esthetic Surgery of the Aging face.
Oral and Maxillofacial Surgery Clinics of North America, Vol6 (2), 1998. P 1

American Board of Oral and Maxillofacial Surgery 187


2008 Oral and Maxillofacial Surgery Self Assessment Tool (OMSSAT)

A 38 year-old female patient presents with erythema and mild induration of the perioral region
and along the inferior border of the mandible, three weeks after a medium depth chemical peel.
She gives a history of using topical tretinoin and isotretinoin for up to four months prior to the
peel. Which of the following is the most likely diagnosis and recommended treatment?

A. Viral Infection prescribe an antiviral agent

B. Bacterial Infection prescribe an antibiotic

C. Fungal Infection prescribe an antifungal agent

D. Hypertrophic scarring topical and/or injectable corticosteroids

Answer: D

Rationale:
This presentation represents hypertrophic scarring. Viral infections present within 2 weeks
as clusters of painful ulcerations. Similarly, bacterial infections are more symptomatic and
present with exudative, pustular eruptions; fungal infections are often seen as diffuse
erythematous and pruritic areas, and KOH staining is useful for diagnosis. Patients who
have had treatment with Isotretinoin (Accutane) and Tretinoin (Retin-A) are predisposed to
complications like hypertrophic scarring due to thinness of skin and reduction of adnexal
structures. Scarring typically presents as erythematous patches that are indurated and
papular in appearance that classically involves the peri-oral and inferior border areas of the
mandible. Occlusive silicone skin dressings may also be helpful.

Reference:
Nahai F, Baker TJ, Stuzin JM. Chemical Peel. The Art of Aesthetic Surgery. St Louis:
Quality Medical Publishing, 2005, p 421.

Demas PN, Braun TW: Chemical Skin Resurfacing. Esthetic Surgery of the Aging face.
Oral and Maxillofacial Surgery Clinics of North America, Vol6 (2), 1998. P 21

American Board of Oral and Maxillofacial Surgery 188


2008 Oral and Maxillofacial Surgery Self Assessment Tool (OMSSAT)

Which of the following peeling agents is directly associated with cardiac events like multiple
premature ventricular contractions (PVCs) during chemical skin resurfacing?

A. 50% glycolic acid

B. Bakers phenol

C. 35 % Trichloracetic acid

D. Jessner's solution

Answer: B

Rationale:
30% -70% Glycolic acid is used as superficial chemical peel and is not known to cause
cardiac toxicity. Glycolic acid decreases the cohesion of corneocytes in the stratum
corneum at low concentrations, and causes complete epidermolysis at higher
concentrations. Baker's Phenol is hepatotoxic, nephrotoxic, cardiotoxic and a respiratory
depressant. Phenol peels should be conducted over a 1-2 hour period with continual IV
fluid administration and EKG monitoring. The administrator continuously mixes the
solution to ensure emulsification and maintain standard concentration. Approximately 15
minutes are permitted between each facial unit application, and patients are usually
monitored for two hours postoperatively. 35% Trichloroacetic acid is used for a medium
depth peel and is not cardiotoxic. TCA is often mixed with solid CO2 or Jessner's solution
as these combination therapies form an effective treatment of photoaging. Jessner's
solution contains resorcinol, salicylic acid, lactic acid, and ethanol, and is also not
cardiotoxic. It provides for a superficial depth peel by itself or a medium depth peel in
combination with other agents (e.g., TCA).

Reference:
Nahai F, Baker TJ, Stuzin JM. Chemical Peel. The Art of Aesthetic Surgery. St Louis:
Quality Medical Publishing, 2005, p 427.

Demas PN, Braun TW: Chemical Skin Resurfacing. Esthetic Surgery of the Aging face.
Oral and Maxillofacial Surgery Clinics of North America, Vol 6(2), 1998. P 8-12

Monheit GD: Skin rejuvenation procedures. Principles of Oral and Maxillofacial Surgery.
2nd edition, Volume 2, BC Decker Inc, London, P 141

American Board of Oral and Maxillofacial Surgery 189


2008 Oral and Maxillofacial Surgery Self Assessment Tool (OMSSAT)

Which of the following is an appropriate preoperative treatment regimen in patients undergoing


laser skin resurfacing of the face?

A. Topical tretinoin to increase fibroblastic activity and collagen production

B. Alpha hydroxy acid therapy to inhibit melanocytes and hyperpigmentation

C. Hydroquinone therapy to enhance penetration of other topical agents

D. Acyclovir to prevent milia formation

Answer: A

Rationale:
Pre-treatment with specific medications is essential prior to laser facial skin resurfacing
surgery. This multiple step regimen is directed specifically to prepare the skin and
associated structures to optimally respond to laser surgery. Tretinoin is used to normalize
keratin and increase fibroblast collagen production to allow for faster wound healing.
Alpha hydroxy acids (Glycolic acid) are used to achieve keratinolysis to enhance
penetration of other topical agents. Hydroquinone is used to inhibit melanocyte activity and
minimize post inflammatory hyperpigmentation. Acyclovir is used to decrease the potential
for postoperative herpetic infections. Milia are keratin retention cysts resulting from
plugged hair follicles, and are not caused by viral infections

Reference:
S. Obagi Pre and postlaser skin care. Oral and Maxillofacial Surgery Clinics North
America 16, 2004 181-187.

Chisholm BB: CO2 Laser skin resurfacing for the aging face. Oral and Maxillofacial
Surgery Clinics of North America. Vol 6(8), 1998. P 23-38

American Board of Oral and Maxillofacial Surgery 190


2008 Oral and Maxillofacial Surgery Self Assessment Tool (OMSSAT)

Ten days following a CO2 laser skin resurfacing procedure, a patients presents with pain, fever,
malaise, and the following non-purulent erythematous erosions:

A. milia.

B. viral infection.

C. post inflammatory hyperpigmentation.

D. fungal infection.

Answer: B

Rationale:
Milia are keratin filled cysts formed due to clogged pores. They are not associated with
fever, malaise and are also not generally uniformly distributed all over the face. Viral
lesions after laser surgery are usually herpetic erosions, and treatment with antivirals until
re-epithelialization is complete is indicated to reduce the incidence of scarring. Post
inflammatory hyperpigmentation is not painful, and does not present with fever, malaise or

American Board of Oral and Maxillofacial Surgery 191


2008 Oral and Maxillofacial Surgery Self Assessment Tool (OMSSAT)

erosion. A normal examination at the 10-day postoperative interval should not look like
this and patients should not have constitutional symptoms like pain, fever, and malaise.

Reference:
P. Demas and J. Bridenstine. Diagnosis and Treatment of Postoperative Complications
after Skin Resurfacing. J Oral Maxillofac Surg 57 (1999) 837- 841.

Chisholm BB: CO2 Laser skin resurfacing for the aging face. Oral and Maxillofacial
Surgery Clinics of North America. Vol 6(8), 1998. P 23-38

American Board of Oral and Maxillofacial Surgery 192


2008 Oral and Maxillofacial Surgery Self Assessment Tool (OMSSAT)

The following photograph shows a patient with pigmentary changes two weeks after C02 laser
therapy. Which of the following correctly describes the initial management of this condition?

A. Topical steroid therapy

B. Avoidance of sunscreens

C. Camouflage make-up

D. Avoidance of topical retinoids

Answer: C

Rationale:
Temporary hyperpigmentation changes after laser surgery are more frequently encountered
in Fitzpatrick III (35%) and IV (75%) skin-types. Treatment includes topical application of
hyqroquinone, topical retinoids (tretonoin), and alpha hydroxy acids (glycolic acid).
Hydroquinone acts by inhibiting melanocytes, while glycolic acid promotes keratinolysis.
Topical retinoids increase fibroblastic activity and collagen production. Immediate steroid
therapy is not advocated. Sunscreens and decreasing exposure to ultraviolet light are
advisable. Camouflage make up can be used and may be required long-term in cases of
prolonged or permanent hyperpigmentation.

Reference:
P. Demas and J. Bridenstine. Diagnosis and Treatment of Postoperative Complications
after Skin Resurfacing. J Oral Maxillofac Surg 57 (1999) 837- 841.

American Board of Oral and Maxillofacial Surgery 193


2008 Oral and Maxillofacial Surgery Self Assessment Tool (OMSSAT)

Chisholm BB: CO2 Laser skin resurfacing for the aging face. Oral and Maxillofacial
Surgery Clinics of North America. Vol 6(8), 1998. P 23-38

American Board of Oral and Maxillofacial Surgery 194


2008 Oral and Maxillofacial Surgery Self Assessment Tool (OMSSAT)

A patient reports that when her skin is exposed to sun, she usually burns, and tans with difficulty.
This patient is best classified as a Fitzpatrick skin type:

A. I.

B. II.

C. III.

D. IV.

Answer: B

Rationale:
The Fitzpatrick classification is useful in diagnosis and treatment planning before laser skin
resurfacing procedures. The classification is based on factors including skin color and
complexion, and its response to sun exposure. Five distinct classes are noted based on the
response to sun exposure: Type I: skin always burns and never tans. Type II: skin usually
burns and tans with difficulty. Type III: skin exhibits a mild burn and average tan. Type
IV: skin rarely burns and tans with ease. Type V: skin very rarely burns and tans
extremely easily.

Ideal patients for carbon dioxide laser procedures include Fitzpatrick type I and II patients.
Type III and IV patients are more prone to complications like hyperpigmentation after CO2
laser skin resurfacing.

Reference:
Nahai F, Baker TJ, Stuzin JM. Skin Treatments. The Art of Aesthetic Surgery. St Louis:
Quality Medical Publishing, 2005, P 391.

Chisholm BB. CO2 laser skin resurfacing of the aging face. Esthetic surgery of the aging
face. Oral and Maxillofacial Surgery Clinics of North America. Vol 6(8), 1998. P 32

American Board of Oral and Maxillofacial Surgery 195


2008 Oral and Maxillofacial Surgery Self Assessment Tool (OMSSAT)

Which of the following procedures is most appropriate for treatment of mild-moderate facial
rhytids and blotchy generalized dyspigmentation in a Fitzpatrick skin type II patient?

A. CO2 laser skin resurfacing

B. Dermabrasion

C. Microdermabrasion

D. Trichloroacetic acid-Jessners solution peel

Answer: D

Rationale:
Fitzpatrick class I and II patients are good candidates for CO2 laser skin resurfacing.
However, laser surgery is usually indicated for more severe facial rhytids and more severe
solar elastosis, and is not very effective in treating skin dyspigmentation. Dermabrasion is
generally performed with rotary instruments like the wire brush and diamond fraise. The
main indication for dermabrasion is acne scarring or other facial scars that involve focal
and not generalized areas of the face. The technique of microdermabrasion is considered a
very superficial procedure since it removes the stratum corneum and outer epidermis. It has
only limited indications like treatment of very fine rhytids and extremely mild
hyperpigmentation. It is a procedure that has to be repeated every two weeks or so in
combination with other agents. 35% -50 % trichloroacetic acid Jessner's solution
combination is ideal for treating mild-moderate facial rhytids, actinic damage and
dyspigmentation. Level of penetration is better controlled with this combination regimen as
compared to TCA alone.

Reference:
Nahia F, Baker TJ, Stuzin JM. Chemical Peel. The Art of Aesthetic Surgery. St Louis:
Quality Medical Publishing, 2005, p367, 447.

Demas PN, Braun TW: Chemical Skin Resurfacing. Esthetic Surgery of the Aging face.
Oral and Maxillofacial Surgery Clinics of North America, Vol 6 (2), 1998. P 21

Chisholm BB. CO2 laser skin resurfacing of the aging face. Esthetic surgery of the aging
face. Oral and Maxillofacial Surgery Clinics of North America. Vol 6(8), 1998. P 32

American Board of Oral and Maxillofacial Surgery 196


2008 Oral and Maxillofacial Surgery Self Assessment Tool (OMSSAT)

Which of the following statements correctly describes the Mustarde technique for otoplasty?

A. Cartilage excision technique in a stepwise fashion to reduce a hypertrophic conchal wall

B. Postauricular approach to removal vestigial posterior auricular muscle and its ligament
down to the mastoid fascia

C. Creation of a new antihelical fold by cartilage weakening and mattress sutures

D. Creation of a new antihelical fold at the expense of sharp cartilaginous ridges seen through
the thin anterior auricular skin

Answer: C

Rationale:
The Mustarde technique involves cartilage weakening and placement of a series of
horizontal mattress sutures to create an antihelical fold. The Davis technique corrects
conchal hypertrophy by stepwise cartilage excision. The Converse-Wood Smith technique
involves several full-thickness cuts through the cartilage and suturing to form an
anitihelical fold; these sharp ridges can often be felt/seen through the thin anterior skin. The
Furnas technique has a high incidence of relapse and is a post auricular approach, where
the muscle is removed and sutures placed from the mastoid fascia to the ear cartilage.

Reference:
Owsley TG, Tejera TJ: Otoplastic surgery for the protruding ear. In: Fonseca R, Baker, S,
Wolford LM (eds). Oral and Maxillofacial Surgery. Vol 6, WB Saunders, Philadelphia
2000, p 408

Bauer BS: Correction of the constricted ear. Plast Surg Tech 1:2, 153-160, 1995

American Board of Oral and Maxillofacial Surgery 197


2008 Oral and Maxillofacial Surgery Self Assessment Tool (OMSSAT)

Perichondritis after otoplasty is most commonly caused by which of the following organisms:

A. streptococcus pyogenes, Escherichia coli, Hemophylus influenza.

B. streptococcus pyogenes, Neisseria gonorrhea, Bacteroides species.

C. staphylococcus aureus, Hemophylus influenza, Bacteroides species.

D. staphylococcus aureus, Escherichia coli, Pseudomonas aerugionsa.

Answer: D

Rationale:
Perichondritis occurs in the early postoperative period, and is usually related to an
undetected or inadequately treated hematoma. Symptoms include pain, erythema, fever,
and discharge. Cartilage wounds can be slow to heal and may demand aggressive therapy,
including surgical debridement and drainage. While cultures are recommended, empirical
therapy of auricular chondritis should include coverage for Pseudomonas, which may be
the most common cause. Given limited options for outpatient therapy, oral ciprofloxacin
has been recommended as the drug of choice. Some practitioners recommend oral
dicloxacilin or cephalexin as initial treatment. Significant infections may warrant
intravenous antibiotic treatment. Massive cartilage destruction and severe ear deformities
can result, despite aggressive treatment

Reference:
Owsley TG, Tejera TJ: Otoplastic surgery for the protruding ear. In: Fonseca R, Baker, S,
Wolford LM (eds). Oral and Maxillofacial Surgery. Vol 6, WB Saunders, Philadelphia
2000, P 408

Tanzer, RC: Congenital deformities, Deformities of the auricle. In: Reconstructive Plastic
Surgery, 2nd edition, W.B. Saunders, Philadelphia, 1997, P 1671

American Board of Oral and Maxillofacial Surgery 198


2008 Oral and Maxillofacial Surgery Self Assessment Tool (OMSSAT)

The main sensory innervation of the auricle is via the:

A. auriculotemporal nerve.

B. cervical plexus.

C. lesser occipital nerve.

D. greater auricular nerve.

Answer: D

Rationale:
The sensory nerve supply is primarily from the anterior and posterior branches of the
greater auricular nerve. This nerve travels 8 mm posterior to the postauricular crease. The
greater auricular nerve originates from the second and third cervical nerves, winds around
the posterior border of the sternomastoid, and, after perforating the deep fascia, ascends
upon that muscle beneath the platysma into the parotid gland, where it divides into anterior
and posterior branches. The posterior (mastoid) branch supplies the skin over the mastoid
process and on the back of the auricle, except at its upper part; a filament pierces the
auricule to reach its lateral surface, where it is distributed to the lobule and lower part of
the concha. Thus, it provides sensory innervation for the skin over parotid gland and
mastoid process, and both surfaces of the outer ear. The auriculotemporal and lesser
occipital nerves supply the conchal cavity and external auditory meatus, and auricular
branches of the vagus may supply the posterior wall of the external auditory meatus.

Reference:
Oswley, TG, Tejera TJ: Otoplastic surgery for the protruding ear. In: Fonseca R, Baker S,
Wolford LM (eds). Oral & Maxillofacial surgery Vol 6, WB Saunders, Philadelphia 2000.
P 408

Tanzer, RC: Congenital deformities, Deformities of the auricle. In: Reconstructive Plastic
Surgery, 2nd edition, W.B. Saunders, Philadelphia, 1997, P 1671

American Board of Oral and Maxillofacial Surgery 199


2008 Oral and Maxillofacial Surgery Self Assessment Tool (OMSSAT)

Which of the following is a major mechanism for nasal tip support?

A. Size and shape of nasal bones

B. Medial crural feet attachment to the nasal septum

C. Anterior nasal spine

D. Soft tissue thickness of the ala

Answer: B

Rationale:
Nasal tip supporting mechanisms can be divided into primary and secondary support
mechanisms:
Primary/Major
1. Attachment of the caudal septum and medial crura (interrupted by a complete
transfixion incision)
2. Fibrous attachments between the upper and lower lateral nasal cartilages (interrupted by
intercartilaginous incisions)
3. Size, shape, and strength of the lower lateral cartilages

Secondary/Minor
1. Interdomal ligament
2. Sesamoid complex, which, in effect attaches the lower lateral cartilages to the
piriform rim
3. Cartilaginous septal dorsum
4. Anterior nasal spine
5. Membranous septum
6. Alar cartilages with their overlying skin

Reference:
Kennedy BD: Indications and Techniques for Rhinoplasty. Principles of Oral and
Maxillofacial Surgery. Peterson LJ (ed). JB Lippincott Co, 1992. P 1724-5

Koehler J, Waite PD: Basic principles of rhinoplasty. Principles of Oral and Maxillofacial
Surgery, 2nd ed, BC Decker Inc., London 2004, p. 1350

American Board of Oral and Maxillofacial Surgery 200


2008 Oral and Maxillofacial Surgery Self Assessment Tool (OMSSAT)

This open roof nasal deformity is best corrected by:

A. nasal septoplasty.
.
B. shield graft placement.

C. lateral nasal osteotomies.

D. shaving upper lateral cartilages.

Answer: C

Rationale:
An open roof deformity is created during dorsal hump reduction surgery. It gives the
appearance of a broad, flattened nasal bridge from the frontal view. This condition can is
corrected with nasal osteotomies. The osteotomies are performed bilaterally and the nasal
bridge/dorsum narrowed by infracturing of the nasal bones. Bilateral lateral osteotomies are
commonly used, and occasionally medial osteotomies can also be performed. Care must be
taken during treatment as over-narrowing may cause nasal obstruction. . Repositioning the
nasal septum or septoplasty has no effect on eliminating this condition. Shield grafts are
used for nasal tip definition. Shaving of upper lateral cartilages may worsen the open roof
deformity.

Reference:

American Board of Oral and Maxillofacial Surgery 201


2008 Oral and Maxillofacial Surgery Self Assessment Tool (OMSSAT)

Kennedy BD: Cosmetic Rhinoplasty. In: Fonseca R, Baker S, Wolford LM (eds). Oral &
Maxillofacial surgery Vol 6, WB Saunders, Philadelphia 2000. P 332

Bruce N. Epker: Cosmetic Oral & Maxillofacial Surgery. Oral and Maxillofacial Surgery
Clinics of North America Saunders & Company, May 1990, P 289-338

American Board of Oral and Maxillofacial Surgery 202


2008 Oral and Maxillofacial Surgery Self Assessment Tool (OMSSAT)

This patient requires narrowing and improved definition of her nasal tip. This result can best be
accomplished by:

A. external rhinoplasty with shield grafts.

B. septoplasty with Weir excision procedure.

C. internal rhinoplasty with lateral nasal osteotomies.

D. transfixion incision and lowering of the septal angle.

Answer: A

Rationale:
The nasal tip is a soft tissue structure. Nasal tip alterations and refinements are best
accomplished by soft tissue surgery and with carefully placed cartilage grafts. An open
technique gives greater surgical access and direct visibility, and is especially indicated for
complex cases. It involves bilateral marginal incisions and a transcolumellar incision
followed by skeletonization and exposure. The complaints of this patient are best addressed
with an external rhinoplasty, cartilage trimming, and placement of customized cartilage
grafts to define the nasal tip (see figure below)

American Board of Oral and Maxillofacial Surgery 203


2008 Oral and Maxillofacial Surgery Self Assessment Tool (OMSSAT)

A Weir procedure is used for correcting the width of alar base, specifically alar base width
reduction. It involves excision of a small wedge of vestibular mucosa and skin. The
excision is usually conservative and rarely greater than 3 mm in width. Lateral nasal
osteotomies would narrow the nose, and address bony defects but not asymmetric nasal tip
deformities. They are indicated to narrow the nose when the width of nasal dorsum and
bridge area is more than 80% of the nasal alar base width. Lowering of the septal angle
with a complete transfixion incision would reduce tip projection and likely produce
undesirable effects.

Reference:
Tardy ME: Rhinoplasty: The Art and Science. WB Saunders Philadelphia, 1997. P 374.

Bruce N. Epker; Cosmetic Oral & Maxillofacial Surgery. Oral and Maxillofacial Surgery
Clinics of North America, May 1990, P 289-338.

Joseph Niamtu, III, DDS: Cosmetic Facial Surgery. Oral and Maxillofacial Surgery
Clinics of North America, November 2000, P 739-754.

American Board of Oral and Maxillofacial Surgery 204


2008 Oral and Maxillofacial Surgery Self Assessment Tool (OMSSAT)

The fibrous connection between the upper lateral cartilages and lower lateral cartilages of the
nose is termed:

A. scroll area.

B. rhinion.

C. internal nasal valve.

D. supratip break.

Answer: A

Rationale:
The scroll is the area of attachment of the lower lateral and upper lateral cartilages. Four
configurations are common: interlocked (52%), overlapped (20%), end-end (17%), and
opposed (11%). The scroll provides important support to the nasal tip and an
intercartilaginous incision during an endonasal rhinoplasty procedure violates this area,
thereby having effects on nasal tip position. The rhinion is the junction between the nasal
bones and the upper lateral cartilage, i.e., the bony and cartilaginous parts of the nasal
bridge. The soft tissue covering and skin of the nose is thinnest at this point. The internal
nasal valve is formed by the junction of the caudal edge of the upper lateral cartilages with
the dorsal septum. The normal value of this is angle is approximately 10 degrees, and
narrowing the valve can have adverse effects on the functional nasal airway surgery by
causing nasal obstruction. The supratip break is a subtle dorsal depression just cephalic to
the nasal tip at the junction of the nasal dorsum and nasal tip.

Figure reproduced from: Koehler J, Waite PD: Basic principles of rhinoplasty. Principles
of Oral and Maxillofacial Surgery, 2nd ed, BC Decker Inc., London 2004, P1348

American Board of Oral and Maxillofacial Surgery 205


2008 Oral and Maxillofacial Surgery Self Assessment Tool (OMSSAT)

Reference:
Kennedy BD: Cosmetic Rhinoplasty. In Fonseca RJ (ed):, Oral and Maxillofacial Surgery.
Philadelphia, PA, Saunders, 2000, Vol 6, P 310

Toriumi, DM: Structure approach in rhinoplasty. University of Illinois, Facial Plastic


Surgery Clinics of North America, Vol 13, No. 1, 2003. P127

Koehler J, Waite PD: Basic principles of rhinoplasty. Principles of Oral and Maxillofacial
Surgery, 2nd ed, BC Decker Inc., London 2004, P1348

American Board of Oral and Maxillofacial Surgery 206


2008 Oral and Maxillofacial Surgery Self Assessment Tool (OMSSAT)

Skeletonization of the nose with an external rhinoplasty approach is achieved by placement of


what bilateral incisions?

A. Marginal

B. Transfixion

C. Intercartiliginous

D. Killian

Answer: A

Rationale:
Bilateral marginal incisions are connected by a transcolumellar incision to form the basic
incisions for an external rhinoplasty approach. This connection permits exposure to the
lower lateral cartilages and nasal dorsum. A marginal incision parallels the caudal edges
of the lower lateral cartilages. It involves placement of a curved incision a few millimeters
from the nasal aperture margins. A transfixion incision is performed at the junction of the
inferior aspect of the nasal septum and superior aspect of the medial crura, and can be made
partially or completely or only on one side (hemitransfixion). An intercartiliginous incision
forms the basis of an internal (endonasal) rhinoplasty approach and is made between the
cephalic margin of the lower lateral cartilage and caudal margin of the upper lateral
cartilage. The Killian incision is not used in rhinoplasty, but is a traditional incision design
for nasal septoplasty. It refers to an incision made several millimeters cephalad to the
caudal edge of the septum, and can be extended to the nasal floor, if required.

Reference:
Fonseca RJ: Cosmetic Rhinoplasty, in Oral and Maxillofacial Surgery. Philadelphia, PA,
Saunders, 2000, Vol 6, pg 314

Toriumi, DM: Structure approach in rhinoplasty. University of Illinois, Facial Plastic


Surgery Clinics of North America, Vol 13, No. 1, 2003.

Koehler J, Waite PD: Basic principles of rhinoplasty. Principles of Oral and Maxillofacial
Surgery, 2nd ed, BC Decker Inc., London 2004, p. 1354

American Board of Oral and Maxillofacial Surgery 207


2008 Oral and Maxillofacial Surgery Self Assessment Tool (OMSSAT)

Cephalic trimming of the lower lateral cartilages of the nose during rhinoplasty has which of the
following effects on the nasal tip:

A. upward rotation.

B. downward rotation.

C. widening.

D. no effect.

Answer: A

Rationale:
Although cartilage preservation is emphasized, cartilage resection is required in certain
instances. There are 3 basic principles of cartilage excision in the nasal tip region: complete
strip technique (Figure a), a weakened complete strip technique, and an interrupted strip
technique. (Figure b) Following strip removal, at least 6 mm width of lower lateral
cartilages must be retained to ensure adequate tip support.

Cephalic trimming of the lower lateral cartilages provides for some upward tip rotation as
the cephalic edge and its connection to the upper lateral cartilages is removed. The rotation
is greatest in the interrupted strip technique. Additionally, cephalic trimming also narrows
the nasal tip, thereby giving it more definition and refinement. Depending on the medial
extension of excision into the dome area, cephalic trim can actually cause a mild reduction
in tip projection.

Figure a

American Board of Oral and Maxillofacial Surgery 208


2008 Oral and Maxillofacial Surgery Self Assessment Tool (OMSSAT)

Figure b

Lined areas depict areas of cephalic cartilage trimming and the dotted areas show
preservation of cartilage for support

Figures adapted from: Taylor CO: Surgery of the nasal tip. In, Waite PD(ed): Atlas of Oral
and Maxillofacial Surgery Clinics of North America: Rhinoplasty, W.B. Saunders,
Philadelphia, 1995. P 58-59

Reference:
Fonseca RJ: Cosmetic Rhinoplasty, in Oral and Maxillofacial Surgery. Philadelphia, PA,
Saunders, 2000, Vol 6, pg 325

Anderson JR, Ries WR: Rhinoplasty: Emphasizing the external approach. The American
Academy of Facial Plastic and Reconstructive Surgery. Thieme Inc., New York. 1986. P
98-106

Koehler J, Waite PD: Basic principles of rhinoplasty. Principles of Oral and Maxillofacial
Surgery, 2nd ed, BC Decker Inc., London 2004, p. 1362

American Board of Oral and Maxillofacial Surgery 209


2008 Oral and Maxillofacial Surgery Self Assessment Tool (OMSSAT)

Which of the following pairs of nasal deformity and indicated treatment option (cartilage
grafting) pairings is correct?

A. Poor nasal tip supportspreader graft

B. Internal nasal valve collapsecolumellar strut graft

C. Saddle nose deformityumbrella graft

D. Amorphous nasal tipshield graft

Answer: D

Rationale:
Nasal tip support is enhanced by columellar strut grafts, which can be placed between the
right and left medial crura of the lower lateral cartilages, and abutted against the nasal
spine. The internal nasal valve is essential for a good functional airway, and if narrowed,
can be opened/improved with spreader grafts, which are placed between the nasal septum
and upper lateral cartilages. A minimum of 10-15 degree angle must be maintained in the
valve area to ensure patency of the airway. A nasal tip graft such as a shield graft is used to
give more definition to an amorphous nasal tip. Shield grafts usually consist of a
trapezoidal shaped cartilage graft that is sutured in the tip area. The superior margins are
notched and beveled to reduce sharp transition points. Placement via a closed rhinoplasty
approach utilizes marginal incisions and development of a soft tissue pocket anterior and
inferior to the medial crura. With an open technique, the graft is directly sutured to the
domal area. Umbrella grafts refer to a combination of a collumellar strut graft and a tip
graft. These grafts are indicated in cases where there is lack of tip definition and tip support
due to weakness in the region of the medial crura. A saddle nose deformity is corrected by
augmentation with cartilage or alloplastic materials, but not with umbrella grafts

American Board of Oral and Maxillofacial Surgery 210


2008 Oral and Maxillofacial Surgery Self Assessment Tool (OMSSAT)

Figure reproduced from: Taylor CO: Surgery of the nasal tip. In, Waite PD(ed): Atlas of
Oral and Maxillofacial Surgery Clinics of North America: Rhinoplasty, W.B. Saunders,
Philadelphia, 1995.

Reference:
Fonseca RJ: Cosmetic Rhinoplasty, in Oral and Maxillofacial Surgery. Philadelphia, PA,
Saunders, 2000, Vol 6, pg 334-335

Werther JR: Posttraumatic Rhinoplasty in Oral and Maxillofacial Surgery Knowledge


Update, 1995, Vol 1, Part II, pg 53-59

American Board of Oral and Maxillofacial Surgery 211


2008 Oral and Maxillofacial Surgery Self Assessment Tool (OMSSAT)

A patient presents with a complaint of nasal obstruction one year after suffering nasal trauma.
Intranasal exam reveals septal deviation and dislocation off the anterior nasal spine. Bilateral
internal and external nasal valve collapse and right inferior turbinate hypertrophy are noted.
During septorhinoplasty, the surgeon must:

A. remove all portions of deviated cartilaginous and bony septum in order to improve the
airway.

B. not harvest septal cartilage and bone for grafting due to the posttraumatic etiology.

C. leave an adequate dorsal and caudal strut of cartilage for nasal dorsum and tip support.

D. incise both sides of the mucosa to develop a communication between the right and left
nares for adequate septal straightening.

Answer: C

Rationale:
The surgeon must maintain an L-strut of cartilage for nasal support. Usually,
approximately 1 cm of cartilage is left on the dorsal and caudal aspects to maintain support.
All portions of deviated septum need not be removed; instead, one can score or selectively
remove cartilage to straighten the remaining portions. In this patient, cartilage and bone
harvested from the septum will be valuable for grafting to improve the functional airway
with cartilage grafts. It is always advantageous to leave at lease one side of the septal
mucosa intact to minimize the incidence of septal perforations.

Figure reproduced from: Taylor CO: Surgery of the nasal tip. In, Waite PD(ed): Atlas of
Oral and Maxillofacial Surgery Clinics of North America: Rhinoplasty, W.B. Saunders,
Philadelphia, 1995.

American Board of Oral and Maxillofacial Surgery 212


2008 Oral and Maxillofacial Surgery Self Assessment Tool (OMSSAT)

Reference:
Werther JR: Posttraumatic Rhinoplasty in Oral and Maxillofacial Surgery Knowledge
Update, 1995, Vol 1, Part II, P 55-59

Anderson JR, Ries WR: Rhinoplasty: Emphasizing the external approach. The American
Academy of Facial Plastic and Reconstructive Surgery. Thieme Inc., New York. 1986. P
98-106

American Board of Oral and Maxillofacial Surgery 213


2008 Oral and Maxillofacial Surgery Self Assessment Tool (OMSSAT)

Which graft improves external nasal valve function?

A. Dorsal nasal

B. Shield

C. Alar batten

D. Columellar strut

Answer: C

Rationale:
Augmentation of the dorsum of the nose does not have any direct beneficial effects on the
functional nasal airway. Similarly, a camouflage graft over the left nasal bony pyramid or a
shield tip graft can help improve residual asymmetry but will not improve the function of
the airway. Spreader grafts can improve the internal nasal valve, and alar batten grafts in
the supra-alar creases will improve the external nasal valve function. Additionally, a
columellar strut graft may provide some elevation and support to the ptotic nasal tip and
should subsequently improve the long-term airway function in this patient with poor tip
support.

American Board of Oral and Maxillofacial Surgery 214


2008 Oral and Maxillofacial Surgery Self Assessment Tool (OMSSAT)

Figures depict placement of an alar batten graft via a marginal incision


Figures taken from: The Rhinoplasty Center. An educational guide to the possibilities of
Rhinoplasty.

Reference:
Werther JR: Posttraumatic Rhinoplasty in Oral and Maxillofacial Surgery Knowledge
Update, 1995, Vol 1, Part II, pg 55-59

Fonseca RJ: Cosmetic Rhinoplasty, in Oral and Maxillofacial Surgery. Philadelphia, PA,
Saunders, 2000, Vol 6, pg 334-335

American Board of Oral and Maxillofacial Surgery 215


2008 Oral and Maxillofacial Surgery Self Assessment Tool (OMSSAT)

This patient presents with a post-traumatic nasal deformity. Which of the following is correct
regarding nasal bone osteotomies on this patient?

A. Start the lateral osteotomies below the inferior turbinate to prevent airway collapse

B. Avoid extending the lateral osteotomies into the radix to prevent a rocker deformity

C. Avoid performing lateral and medial osteotomies because of history of trauma

D. Perform an intermediate osteotomy on the left side to help obtain symmetry

Answer: B

Rationale:
It is advisable to avoid extending the lateral osteotomy into the thick nasal bone at the radix
because this can cause a rocker deformity. Starting the lateral osteotomy above the
inferior turbinate preserves the triangular bone at the piriform rim and prevents collapse of
the inferior turbinate into the airway. Medial osteotomies would allow free and full
mobilization of each side of the bony pyramid and improve the asymmetry. An
intermediate osteotomy on this patient's right side (longer side) and not the left (shorter)
side would allow shortening of the nasal pyramid and make the nose more symmetric

Reference:
Werther JR: Posttraumatic Rhinoplasty in Oral and Maxillofacial Surgery Knowledge
Update, 1995, Vol 1, Part II, pg 55-59

Fonseca RJ: Cosmetic Rhinoplasty, in Oral and Maxillofacial Surgery, W.B. Saunders,
2000, Vol 6, pg 334-335
.

American Board of Oral and Maxillofacial Surgery 216


2008 Oral and Maxillofacial Surgery Self Assessment Tool (OMSSAT)

The tripod concept in rhinoplasty refers to:

A. medial crura and upper lateral cartilages.

B. medial crura and caudal septum.

C. medial crura and bilateral lateral crura.

D. bilateral lateral crura and scroll.

Answer: C

Rationale:
The tripod concept is very important in rhinoplasty. The medial crura and the lateral crura
of the lower lateral cartilages form the nasal tripod. The natural position of the nose is tilted
as the one limb formed by the united medial crura is always shorter than the other two
limbs, which are formed by the lateral crura. Modification of one of these limbs causes
significant effects on nasal tip position and profile. Knowledge of this concept allows
precise alterations in nasal tip projection and nasal tip rotation. For example, shortening of
one of the longer limbs (lateral crura) will shorten the nose, and lengthening the shorter
limb (medial crura) will increase tip projection.

Figure reproduced from: Taylor CO: Surgery of the nasal tip. In, Waite PD(ed): Atlas of
Oral and Maxillofacial Surgery Clinics of North America: Rhinoplasty, W.B. Saunders,
Philadelphia, 1995.

American Board of Oral and Maxillofacial Surgery 217


2008 Oral and Maxillofacial Surgery Self Assessment Tool (OMSSAT)

Reference:
Anderson JR, Ries WR: Rhinoplasty: Emphasizing the external approach. The American
Academy of Facial Plastic and Reconstructive Surgery. Thieme Inc., New York. 1986. P 64

Kennedy BD: Indications and Techniques for rhinoplasty. Principles of Oral and
Maxillofacial Surgery. Peterson LJ (ed). JB Lippincott Co, 1992. P 1724

American Board of Oral and Maxillofacial Surgery 218


2008 Oral and Maxillofacial Surgery Self Assessment Tool (OMSSAT)

Ideally, the width of the alar base in the white population should be within how many mm of the
intercanthal distance:

A. 1-2.

B. 3-4.

C. 5-6.

D. 7-8.

Answer: A

Rationale:
The alar base width is closely related to the intercanthal distance, and measures within 1-2
mm of the intercanthal distance for optimal facial balance of the middle third of the face.
Increased alar base width could signify maxillary elongation or posterior positioning of the
midface. Normal intercanthal distance is 34+/- 4 mm in Caucasians, and can vary with
racial norm. Generally, darker skinned individuals have larger values (telecanthism). The
width of the nasal body and tip are approximately 80% of the alar base width.

Reference:
Staffel, J.G., M.D., F.A.C.S., Basic Principles of Rhinoplasty, University of Texas
health Sciences Center @ San Antonio, Tx. 1996. P. 13.

Thomas RG: An evaluation of the soft tissue facial profile in North Amercian Black
women. American Journal of Orthodontics. Vol 76, 1979, P 84

American Board of Oral and Maxillofacial Surgery 219


2008 Oral and Maxillofacial Surgery Self Assessment Tool (OMSSAT)

Distortion and narrowing of the normal horizontal palpebral aperture occurs due to:

A. weakness of the orbital septum.

B. laxity of the medial and lateral canthal tendons.

C. prolapse of the lacrimal gland.

D. decrease of lower lid skin tone.

Answer: B

Rationale:
Weakness of the orbital septum and fat prolapse will cause the appearance of a lower lid
bag with no effect on horizontal aperture. In contrast, laxity or dehiscence of the canthal
tendons leads to a decrease in the distance between the medial and lateral commisures,
thereby causing not only narrowing of the horizontal palpebral aperture but also inferior
displacement of the lower lid margin and rounding of the canthal angles. Propapse of
lacrimal glands leads to distortion on the lateral aspect of the upper lid with no effect on
horizontal aperture. Loss of lower lid skin tone will cause a lower lid bag with no effect
on palpebral aperture.

Reference:
Karesh JW: Blepharoplasty; an overview. In: Hupp JR (ed). Esthetic Surgery for the Aging
Face. Atlas of Oral and Maxilllofacial Surgery Clinics of North America. Vol 6, Number 2,
W.B. Saunders Co, Philadelphia 1998. P 90-97

Jarecki HL, Lucarelli MJ, Lemke BN: Blepharoplasty. In: Peterson's Principles of Oral and
Maxillofacial Surgery. Miloro M (ed). BC Decker, London, 2004. 2nd edition, P1321, 1329
.

American Board of Oral and Maxillofacial Surgery 220


2008 Oral and Maxillofacial Surgery Self Assessment Tool (OMSSAT)

This condition is termed:

A. dermatochalasis.

B. blepharochalasis.

C. pseudoptosis.

D. steatoblepharon.

Answer: A

Rationale:
Dermatochalasis refers to excess upper lid skin. It is seen with aging due to collagen and
elastic fiber breakdown. Blepharochalasis refers to a rare, recurrent inflammatory condition
of the upper eyelids in young patients that result in upper lid skin redundancy due to
relaxation. Pseudoptosis is a condition where the eyebrow drops down and as a result, the
upper eyelid may be displaced inferiorly; this condition resembles true ptotis. However,
the two conditions must be differentiated as a ptotic eyelid usually needs levator
aponeurosis surgery, but a pseudoptotic eyelid primarily needs brow lift surgery.
Steatoblepharon refers to bulging of postseptal orbital fat tissue due to a weakened orbital
septum, resulting in a baggy eyelid cosmetic defect.

Reference:
Hornblass A. Ptosis and pseudoptosis and blepharoplasty. Clin Plast Surg 8: page 811,
1981

Karesh J: Blepharoplasty: An overview. Esthetic Surgery for the aging face. Atlas of Oral
and Maxillofacial Surgery 6 (8), 1998. P 87

American Board of Oral and Maxillofacial Surgery 221


2008 Oral and Maxillofacial Surgery Self Assessment Tool (OMSSAT)

The margin-reflex distance-1 (MRD-1) is measured between the:

A. eyelid margin to the corneal light reflex in primary gaze.

B. eyebrow margin to the corneal light reflex in primary gaze.

C. upper brow margin to the lower brow margin passing through the corneal light reflex in
primary gaze.

D. corneal light reflex in primary gaze to the lateral canthal commisure.

Answer: A

Rationale:
MRD is considered to be one of the most sensitive indicators of upper lid ptosis, and is of
paramount importance in preoperative evaluation. It can help distinguish lid ptosis from
blepharochalasis, as these conditions require different treatments for correction. MRD can
be divided into MRD-1 (upper lid to corneal reflex) and MRD-2 (lower lid to corneal
reflex). Normal MRD-1 is 2-5 mm (see figure) and normal MRD-2 is 5-6 mm. If MRD-1 >
5 mm, it may be indicative of thyrotoxicosis; values < 2 mm are seen in lid ptosis.

Figure reproduced from Jarecki HL, Lucarelli MJ, Lemke BN: Blepharoplasty. In:
Peterson's Principles of Oral and Maxillofacial Surgery. Miloro M (ed). BC Decker,
London, 2004. 2nd edition, P1328

Reference:

Werther JR. Aesthetic Blepharoplasty. In Fonseca's Oral & Maxillofacial Surgery; Volume
6, chapter 14. W.B. Saunders Company; 2000.

Karesh J: Blepharoplasty: An overview. Esthetic Surgery for the aging face. Atlas of Oral
and Maxillofacial Surgery 6 (8), 1998. P 87

American Board of Oral and Maxillofacial Surgery 222


2008 Oral and Maxillofacial Surgery Self Assessment Tool (OMSSAT)

A 56 year-old patient presents for evaluation of lower lid puffiness. What test can the surgeon
perform to differentiate between fat prolapse and edema?

A. Lid distraction test

B. Snap test

C. Needle aspiration

D. Ballottement

Answer: D

Rationale:
Facile differentiation of orbital fat from edema is achieved by gentle ballottement of the
lower eyelid; The technique of ballottement involves application of gentle digital pressure
to the globe, thereby displacing the globe posteriorly. If the fullness is due to fat prolapse,
ballottement of the globe will result in distinct anterior movement of the fullness;
contrastingly, in cases where edema is the etiology, ballottement of the globe does not lead
to anterior displacement of the fullness. . Additionally, the patient can be asked to look in
an upward direction; if fat prolapse through the orbital septum is the etiology of the
puffiness, superior gazing will accentuate the condition. Even in cases of severe interstitial
edema, the possibility of aspirating fluid with a needle is almost minimal, so this is not
advisable. The snap test is used to assess the risk of ectropion after eyelid surgery; the
lower lid is pulled away from the eye and allowed to snap-back, and a delay indicates
higher risk of ectropion. A normal lid returns to its position within approximately 1-3
seconds. The lid distraction test is used to assess the laxity of ligaments of the lower
eyelid. Here the lid is pulled away from the globe; in a normal eyelid, the lid can be pulled
away only less than 6 mm.

Reference:
Jarecki HL, Lucarelli MJ, Lemke BN. Blepharoplasty. In Peterson's Principles of Oral &
Maxillofacial Surgery. Volume 2, Chapter 64; BC Decker, Inc; second edition, 2004. P
1329

American Board of Oral and Maxillofacial Surgery 223


2008 Oral and Maxillofacial Surgery Self Assessment Tool (OMSSAT)

Werther JR. Aesthetic Blepharoplasty. In Fonseca's Oral & Maxillofacial Surgery; Volume
6, chapter 14. W.B. Saunders Company; 2000. p353

American Board of Oral and Maxillofacial Surgery 224


2008 Oral and Maxillofacial Surgery Self Assessment Tool (OMSSAT)

How many fat pads are present in the upper eyelid?

A. 1

B. 2

C. 3

D. 4

Answer: B

Rationale:
The upper eyelid contains 2 fat pads: the medial and central fat pads which may be
removed during upper eyelid blepharoplasty. The lower eyelid contains 3 fat pads: the
medial, central and lateral fat pads. In the upper eyelid, the lateral compartment is occupied
by the lacrimal gland, which should not be disturbed during upper eyelid blepharoplasty.

Reference:
Jarecki, HL, Lucarelli MJ, Lemke, BN: Blepharoplasty. Peterson's Principles of Oral and
Maxillofacial Surgery 2nd Edition. 2004: pp 1317-1344.

Karesh JW: Blepharoplasty. Esthetic surgery of the aging face. Atlas of the Oral and
Maxillofacial Surgery Clinics of North America. September 1998. P88

American Board of Oral and Maxillofacial Surgery 225


2008 Oral and Maxillofacial Surgery Self Assessment Tool (OMSSAT)

Vertical glabellar frown lines are caused by action of which muscle?

A. Frontalis

B. Procerus

C. Corrugator

D. Orbicularis

Answer: C

Rationale:
The frontalis muscle originates from the subgaleal scalp plane and inserts into the
orbicularis oculi. Unlike the multiple depressors of the brow, the frontalis muscle is the
only true brow elevator. This upward action maintains upward positioning of the brow, but
can cause horizontal creases over time. Vertical glabellar frown lines are usually caused by
the corrugator muscle. This muscle originates fro the frontal bone just above the nasal
bones and inserts into the dermis of the upper brow. It has 2 heads: oblique, and transverse,
which pull the medial eyebrows towards the midline with slight depression causing vertical
and oblique furrows respectively. Horizontal rhytids in the midline region are caused by the
procerus muscles, which orginate over the lower nasal bones and upper lateral alar
cartilages, and insert into the glabellar dermis. Together, the corrugator and procerus
muscles are the main depressors of the medial brow and are the most common muscles
treated with Botulinum toxin A to alleviate frown lines in the glabellar region. These are
the same muscles that are also surgically disrupted in brow and forehead lifts. The orbital
part of the orbicularis oculi originates from the medial canthal tendon and surrounding
bone, and inserts into portions of the adjacent depressors, the frontalis, and the dermis
below the brow.

Reference:

American Board of Oral and Maxillofacial Surgery 226


2008 Oral and Maxillofacial Surgery Self Assessment Tool (OMSSAT)

Wieder JM, Moy RL. Understanding Botulinum Toxin: Surgical Anatomy of the Frown,
Forehead, and Periocular Region. Dermatologic Surgery, 1998, 24, 1172-74.

Cuzalina A: Forehead and Brow procedures. Principles of Oral and Maxillofacial Surgery.
BC Decker, London. 2004. P 1387

American Board of Oral and Maxillofacial Surgery 227


2008 Oral and Maxillofacial Surgery Self Assessment Tool (OMSSAT)

This condition has occurred after botulinum toxin A (Botox) treatment of the periorbital region.
The most likely etiology is inadvertent treatment of which muscle?

A. Levator

B. Mullers

C. Orbicularis

D. Frontalis

Answer: A

Rationale:
The patient has lid ptosis. The levator palpebrae superioris is the main lid retractor. It arises
from the lesser wing of the sphenoid, deep in the bony orbit, and courses over the superior
rectus muscle and forms an aponeurosis with important bony, dermal and tarsal
attachments. True lid ptosis following Botox treatment is due to inadvertent treatment of
levator palpebrae superioris muscle. This complication usually self-corrects in 3-6 months
once the effects of Botox wear off. Muller's muscle is sympathetically innervated, arises
from the undersurface of the levator muscle, and attaches to the tarsus. It provides
approximately 2 mm of upper eyelid lift, especially when the sympathetic system is
stimulated. The orbicularis muscle has three parts: pretarsal, preseptal, and orbital. The
pretarsal and preseptal parts together are responsible for reflex lid closure. Botulinium
toxin-mediated incomplete paralysis of the frontalis muscle is often chosen to achieve a
natural effect, so that some forehead animation is retained while eliminating the frown.
However, the action of residual active frontalis muscle may cause excessively peaked
eyebrows.

Reference:
Coleman Moriarty K. Botulinum Toxin in Facial Rejuvenation. Mosby, 2004, 47-48.

Cuzalina A: Forehead and Brow procedures. Principles of Oral and Maxillofacial Surgery.
BC Decker, London. 2004. P 1387

American Board of Oral and Maxillofacial Surgery 228


2008 Oral and Maxillofacial Surgery Self Assessment Tool (OMSSAT)

In males, during brow lift surgery, the inferior border of the entire brow is best-positioned ____
mm above the inferior border of the superior orbital rim.

A. 2

B. 4

C. 6

D. 8

Answer: A

Rationale:
The eyebrows should form a graceful curvature above the supraorbital rim. In females, the
terminal ends are approximately 1 cm above the supraorbital rim, with the greatest height
of the eyebrow curvature in line with the lateral limbus. Typically, the brow is divided into
3 distinct regions in females: medial 1/3, apex, and tail. The medial portion is 1-2 mm
above the inferior border of the supraorbital rim this area. The apex is located 8-10 mm
above the inferior border of the lateral supraorbital rim, and the tail is located 10-15 mm
from the anterior border of the supraorbital rim. In males, the supraorbital rim is more
pronounced, with larger eyebrows, more horizontally oriented with less curvature as
compared to females.

American Board of Oral and Maxillofacial Surgery 229


2008 Oral and Maxillofacial Surgery Self Assessment Tool (OMSSAT)

Figures reproduced from: Evans TW: Browlift. In: Esthetic Surgery of the Aging Face.
Hupp JR (ed). Atlas of Oral and Maxillofacial Surgery Clinics of North America. Vol 6(8),
1998.

Reference:
Alexander RW: Cosmetic Surgery of the Periorbital Area: Blepharoplasty, Brow Lifting
and Forehead Lift. Oral Maxillofac Surg Clin of N Am 2(2): 413-423, 1990.

Cuzalina A: Forehead and Brow procedures. Principles of Oral and Maxillofacial Surgery.
BC Decker, London. 2004. P 1387

Evans TW: Browlift. In: Esthetic Surgery of the Aging Face. Hupp JR (ed). Atlas of Oral
and Maxillofacial Surgery Clinics of North America. Vol 6(8), 1998. P 23-38

American Board of Oral and Maxillofacial Surgery 230


2008 Oral and Maxillofacial Surgery Self Assessment Tool (OMSSAT)

The coronal incision used in an open forehead lifts is best used in a:

A. male patient with medium forehead hairline.

B. female patient with medium forehead hairline.

C. male patient with high hairline.

D. female patient with high hairline.

Answer: B

Rationale:
Coronal brow lift is one of the earliest procedures for brow and forehead lifting. An
incision is made in the scalp hair (the incision is made beveled parallel to the hair follicles,
but could still lead to hair loss) 1-3 cm behind the hairline. Dissection is performed in a
subgaleal or subperiosteal plane, which then connects to a lateral subtemporoparietal plane
dissection. Scalp tissue excision is done to elevate the brows.

The coronal incision is best indicated for females or non-balding males with a medium to
low forehead hairline. Trichophytic or pretrichial incisions are best used on patients with
high forehead hairlines, where further lengthening of the forehead/hairline is undesirable.
The pretrichial incision is made in front of the hairline and leaves a scar on the forehead in
front of the hairline. In contrast, the trichophytic incision is made just behind the hairline; it
is beveled so that the follicles in front of the hairline survive, and hair grows to camouflage
the incision scar. Generally, it is advisable to avoid visible scalp incisions in balding males
(medium to high forehead lines), and endoscopic procedures are usually the best choice in
these cases.

Reference:
Larrabee WF: Browlifts. In Atlas of Head & Neck Surgery-Otolaryngology. BJ Bailey
(ed) Lippincott-Raven, Philadelphia, 1996, pp. 502-505.

Cuzalina A: Forehead and Brow procedures. Principles of Oral and Maxillofacial Surgery.
BC Decker, London. 2004.

Evans TW: Browlift. Oral and Maxillofacial Surgery Clinics of North America.
Vol 6(8), 1998. P 23-38

American Board of Oral and Maxillofacial Surgery 231


2008 Oral and Maxillofacial Surgery Self Assessment Tool (OMSSAT)

Which of the following maneuvers is routinely performed in endoscopic brow lift surgery?

A. Release of periosteum and depressor muscles

B. Complete removal of glabellar motor innervation

C. Sectioning of supraorbital and supratrochelar nerves

D. Overcorrection of the medial brow contour

Answer: A

Rationale:
This question tests the surgeon's understanding of the results of the surgical manipulation
and the cause and effect of the manipulations. Because the sling of the posterior scalp
exerts a backward force on the brow, complete release of periosteal and muscular
attachments must be performed to allow full elevation of the brow and relaxation of the
scalp posterior to fixation points. Complete removal of motor innervation is not a goal, but
a complication of endoscopic brow surgery; temporary decrease in motor nerve innervation
of the glabellar region is a desired effect of botulinum toxin therapy. The supratrochelar
and supraorbital nerves supply sensory innervation, and should not be affected during
surgery. Lateral (and not medial) brow contour is most important and lateral brow ptosis
the main manifestation of brow ptosis. Over-correction medially would distort the normal
brow architecture.

Reference:
Cosmetic Occuloplastic Surgery, 3rd edition, Putterman, Allen M. and Warren, Linda A.
Saunders 199, pg 315

A Retrospective Comparison of Open and Endoscopic Brow-lifts, Puig, C., LaFerriere, K.,
Arch Facial Plastic Surgery 2002; 4: P 221,225

Cuzalina A: Forehead and Brow procedures. Principles of Oral and Maxillofacial Surgery.
BC Decker, London. 2004. P 1396

American Board of Oral and Maxillofacial Surgery 232


2008 Oral and Maxillofacial Surgery Self Assessment Tool (OMSSAT)

The most reliable indicator of brow ptosis in females is measurement of the distance between the
highest point on the brow and:

A. trichion.

B. mid-pupil.

C. mid-forehead.

D. nasion.

Answer: B

Rationale:
The trichion refers to the hairline, and since it is not a stable landmark, it should not be
used for diagnosis and treatment planning. The diagnosis of brow ptosis is best made by
measuring the distance between the highest point of the brow to the mid pupil. This
distance is measured during forward gaze on a diagonal line and is normally 25 mm in a
non-ptotic female brow, as measured by a Colon browmeter. The highest point of the brow
(apex) in females lies approximately halfway between the lateral limbus and lateral
canthus. The mid-forehead and nasion reference points are more subjective and
inconsistent and are not reliable.

Figure reproduced from: Evans TW: Browlift. In: Esthetic Surgery for the Aging Face.
Hupp JR (ed). Atlas of Oral and Maxillofacial Surgery Clinics of North America. Vol 6 (2),
1998. P 118

Reference:
Zukowski ML: Endoscopic Brow Surgery. Oral Maxillofac Surg Clin N Am 12(4):703-
703, 2000

Evans TW: Browlift. In: Esthetic Surgery for the Aging Face. Hupp JR (ed). Atlas of Oral
and Maxillofacial Surgery Clinics of North America. Vol 6 (2), 1998. P 118

American Board of Oral and Maxillofacial Surgery 233


2008 Oral and Maxillofacial Surgery Self Assessment Tool (OMSSAT)

Which of the following is the most ideal candidate for cervicofacial liposuction?

A. 50-year-old patient with skin laxity

B. 35-year-old patient with moderate jowling

C. 30-year-old patient with an anterior hyoid

D. 45-year-old patient with platysmal banding

Answer: B

Rationale:
The ideal candidate for liposuction is usually less than 40 years old, has good skin
elasticity, a favorable hyoid position, and localized fat deposits. A patient with excess skin
laxity may be better suited for rhytidectomy. Anterior or low hyoid position presents a
limitation to the desired result and may be better treated with open lipectomy and
genioplasty. Platysmal banding may be worsened by closed liposuction, and should be
addressed with open lipectomy and muscle plication procedures. A young patient with
isolated, moderate jowling may achieve the desired result with liposuction alone, as the
fatty deposits are likely supraplatysmal.

Reference:
Cuzalina, LA; Koehler, J. Submentoplasy and Facial Liposuction. Oral Maxillofacial Surg
Clin N Am 17:85-98, 2005.

Ziccardi, VB. Adjunctive Cervicofacial Liposuction. Atlas Oral Maxillofacial Surgery


Clinics of North America 8:2, 81-97, 2000.

Watts J: Cervicofacial Liposuction. Oral and Maxillofacial Surgery Clinics of North


America. Vol 12, 623-629, November 2000

Facial Suction Lipectomy: Fonseca's Oral and Maxillofacial Surgery Volume 6: 491-511

American Board of Oral and Maxillofacial Surgery 234


2008 Oral and Maxillofacial Surgery Self Assessment Tool (OMSSAT)

Liposuction of the jowl region is best approached from which incision:

A. submental.

B. submandibular.

C. infra-auricular.

D. nasolabial.

Answer: C

Rationale:
A submental crease incision is routinely used to perform liposculpting of the submental and
submandibular regions. Alternatively, submandibular crease incisions can also be made for
removal of fat in the submandibular and submental regions. The jowl region is located
posteriorly in the neck around the inferior border of the mandible. In order to minimize the
chances of damage to anatomic structures including facial nerve, it is recommended that
this region be approached from a posterior access like the infra-auricular region.
Additionally, this infra-auricular approach permits subdermal extension and suctioning
over the inferior border of the mandible and upper neck areas, which may lead to improved
facial contours by emphasizing the prominence of the jaw line. A small incision in the
perinasal area is rarely required, but can be used to access an area of facial adiposity which
can not be accessed by the above mentioned incisions.

Figures reproduced from: Alexander, RW. Liposculpture of the Cervicofacial Region. In:
Esthetic Surgery of the Aging Face. Hupp JR (ed). Atlas Oral Maxillofacial Surg Clin N
Am 6:2, 1998. P 79, 81

American Board of Oral and Maxillofacial Surgery 235


2008 Oral and Maxillofacial Surgery Self Assessment Tool (OMSSAT)

Reference:
Alexander, RW. Liposculpture of the Cervicofacial Region. In: Esthetic Surgery of the
Aging Face. Hupp JR (ed). Atlas Oral Maxillofacial Surg Clin N Am 6:2, 73-85, 1998.

Cuzalina, LA; Koehler, J. Submentoplasy and Facial Liposuction. Oral Maxillofacial Surg
Clin N Am 17:85-98, 2005.

Jugan,MJ. Liposculpting Procedures. Peterson's Principles of Oral and Maxillofacial


Surgery. BC Decker, Hamilton, 2004.

American Board of Oral and Maxillofacial Surgery 236


2008 Oral and Maxillofacial Surgery Self Assessment Tool (OMSSAT)

The most pertinent anatomic landmarks to be identified during cervicofacial liposuction are:

A. anterior belly of the digastric muscle, posterior belly of the digastric muscle, inferior
border of the mandible.

B. thyroid cartilage, hyoid bone, submental fat deposits.

C. inferior border of the mandible, anterior border of the sternocleidomastoid muscle, thyroid
cartilage.

D. inferior border of the mandible, hyoid bone, posterior belly of the digastric muscle.

Answer: C

Rationale:
Esthetically significant subdermal fat deposits occur in the central submental area and the
region of the mid-jowl. These deposits are often hereditary in nature, and may be resistant
to diet and exercise. This fat is usually well distributed in the neck area, but may be
relatively more increased in central neck, extending from the submandibular crease to the
area of the thyroid cartilage between the anterior borders of the bilateral sternomastoid
muscles. Risk to vascular and neurological structures is minimized if liposuction is
confined to the area inferior to the mandibular border, anterior to the sternocleidomastoid,
and superior to the thyroid cartilage. Submental liposuction should not extend beyond these
borders.

Figure reproduced from: Alexander, RW. Liposculpture of the Cervicofacial Region. In:
Esthetic Surgery of the Aging Face. Hupp JR (ed). Atlas Oral Maxillofacial Surg Clin N
Am 6:2, 1998. P 74

Reference:
Fattahi, TT. Management of Isolated Neck Deformity. Atlas Oral Maxillofacial Surg Clin
N Am 12:261-270, 2004.

American Board of Oral and Maxillofacial Surgery 237


2008 Oral and Maxillofacial Surgery Self Assessment Tool (OMSSAT)

Ziccardi, VB. Adjunctive Cervicofacial Liposuction. Atlas Oral Maxillofacial Surg Clin N
Am 8:2, 81-97, 2000.

American Board of Oral and Maxillofacial Surgery 238


2008 Oral and Maxillofacial Surgery Self Assessment Tool (OMSSAT)

Bleeding during cervicofacial liposuction performed in the correct plane is most likely due to
damage to the:

A. anterior jugular vein.

B. subdermal plexus.

C. facial vein.

D. retromandibular vein.

Answer: B

Rationale:
If liposuction is performed properly in the supraplatysmal layer, deeper vessels should be at
minimal risk. If the suction cannulas are inadvertently placed deep to the platysma muscle,
it may result in damage to the anterior jugular, facial, or retromandibular veins. While
bleeding is minimal with the tumescent technique, there may be damage to the subdermal
plexus.

Reference:
Abraham MT, Romo TR. Liposuction of the Face and Neck. Emedicine.com, 2005.

Jugan MJ. Liposculpting Procedures. In: Peterson's Principles of Oral and Maxillofacial
Surgery. BC Decker, Hamilton, 2004. P 1410-11.

American Board of Oral and Maxillofacial Surgery 239


2008 Oral and Maxillofacial Surgery Self Assessment Tool (OMSSAT)

Restylane (injectable hyaluronic acid):

A. is derived from multiple plant alkaloids.

B. requires allergy testing before human use.

C. lasts longer than bovine collagen.

D. has direct effects on GABA receptors.

Answer: C

Rationale:
Restylane is a very popular lip augmentation material. It is a naturally occurring substance
in humans (derivative of hyaluronic acid). The incidence of allergy is minimal as compared
to bovine injectable filler substances. Numerous studies have shown that Restylane
treatments can lasts up to three times longer than those after injections of bovine collagen.
This longevity is attributed to Restylane beng a hydrophilic molecule; it undergoes
isovolemic degradation, whereby water is drawn into the filler material as it degrades,
thereby allowing itself to maintain its shape longer. It has no specific action on GABA
receptors.

Reference:
Niamtu, J. New lip and wrinkle fillers. Oral and Maxillofacial Surg Clin N Am. 2005;
17:17-28

Kamer FM, Churukian MM: Clinical use of injectible collagen. Arch Otolaryngol 1984:
110: P 93-98

American Board of Oral and Maxillofacial Surgery 240


2008 Oral and Maxillofacial Surgery Self Assessment Tool (OMSSAT)

Lip augmentation using dermal or alloplastic materials is primarily performed to:

A. increase vermillion exposure.

B. decrease white roll definition.

C. round oral commisures.

D. eliminate cupids bow.

Answer: A

Rationale:
Esthetic lips are a result of a pleasing combination of volume, anatomic definition, pout,
and symmetry. The basic outline of an esthetic upper lip is the shape of an M and the
lower lip is curvilinear or parabolic. The upper lip contains one third of the total lip volume
and the lower lip adds two thirds of the volume of the lips. The use of autogenous or
alloplastic materials into the lips primarily achieves an increase in lip volume and
vermillion exposure. A well-defined Cupid's Bow, enhanced white roll, visble philtral
columns, and sharp commissures contribute to the esthetic lip and are targets of injectable
fillers.

Reference:
Sykes JM, Emery BE. Upper and lower lip augmentation with dermal autografts. Oper tech
otolaryngol head & neck surg. 1995; 6:307.

Niamtu, J. Advanta facial implants. Oral Maxillofacial Surg Clin N Am. 2005; 17:29.

American Board of Oral and Maxillofacial Surgery 241


2008 Oral and Maxillofacial Surgery Self Assessment Tool (OMSSAT)

Lip augmentation with bovine collagen is best accomplished by injection of the materials into
which plane?

A. Epithelial

B. Dermal

C. Subdermal

D. Intramuscular

Answer: B

Rationale:
Since the FDA's decision to withdraw approval of injectable silicone materials, bovine
collagen became popular as a lip filling material. Duration of clinical improvement
following injection varies with site selection, degree of muscular activity at site of
injection, and technique of injection. Results can last up to 2-3 years, although most
patients return for touch-ups six months to a year after initial surgery. A layering
technique has been shown to give best results. Proper placement involves sequential
injections of collagen only in the intradermal plane. The less antigenic, cross-linked,
glutaraldehyde treated collagen (Zyplast) is injected into the deep dermis, followed by
placement of Zyderm collagen into the superficial dermal plane.

Reference:
Kamer FM, Churukian MM: Clinical use of injectable collagen. Arch Otolaryngol 1984:
110: P 93-98

Sykes JM, Emery BE. Upper and lower lip augmentation with dermal autografts. Oper tech
otolaryngol head & neck surg. 1995; 6:307.

American Board of Oral and Maxillofacial Surgery 242


2008 Oral and Maxillofacial Surgery Self Assessment Tool (OMSSAT)

The nerve most commonly injured in face lift procedures is the:

A. buccal.

B. spinal accessory.

C. greater auricular.

D. marginal mandibular.

Answer: C

Rationale:
Injury to the facial serve causing paralysis is rare, and reported only to occur in 0.53 to
2.6% of patients. Eighty-five percent of motor nerve injuries resolve spontaneously, and
results of surgical repair are unpredictable and not very encouraging. Injury to the greater
auricular nerve is most common and occurs in up to 7% of patients. Temporary neuropraxia
usually resolves in 2-4 months, and causes numbness/paresthesia around the inferior
portion of the ear and surrounding skin. Transection of the great auricular nerve is best
treated with immediate microanastomosis.

Reference:
Waite PD, Cuzalina LA: Rhytidectomy (Face lift). In: Fonseca R, Baker S, Wolford LM
(eds). Oral & Maxillofacial Surgery, Vol 6, WB Saunders, Philadelphia, 2000. P378

Rees TD, Aston SJ, Thorne CH. Postoperative considerations and complications in
aesthetic plastic surgery. Rees TD and Latrenta GS. Second edition Vol II. WB Saunders
Company 1994. p 273

American Board of Oral and Maxillofacial Surgery 243


2008 Oral and Maxillofacial Surgery Self Assessment Tool (OMSSAT)

This 50 year-old female wishes to enhance her cervico-facial appearance with face lift surgery.
Which of the following surgical techniques is the best option?

A. Subcutaneous

B. Composite

C. Mini-lift

D. Endoscopic

Answer: B

Rationale:
The various procedures available for rhytidectomy can be classified anatomically according
to the depth of dissection from skin down to periosteum. A skin only face-lift is the
simplest procedure, but is usually not indicated. The SMAS face-lift involves surgery to
both the SMAS and the skin. This can be achieved by either reconstituting the SMAS after
removing a strip of redundant preauricular skin or by SMAS plication alone. The degree of
the SMAS flap elevation is variable from none, to a small amount, to extended sub SMAS
elevation to the lateral edge of the zygomaticus major muscle in the face. In the mini-lift,
the skin incision is limited and the SMAS is plicated with a series of sutures at the lower
face and neck area. In a deep plane face lift, the dissection is in a plane below the malar fat
pad. A composite facelift adds the dissection of SMAS flap of the inferior portion of the
orbicularis oculi to the dissection of the deep plane. The subperiosteal facelift elevates the
periosteum off the zygomatic arch and the anterior face of the maxilla in order to reposition
the whole unit superiorly.

American Board of Oral and Maxillofacial Surgery 244


2008 Oral and Maxillofacial Surgery Self Assessment Tool (OMSSAT)

SMAS face lit procedures need to be individualized; In this specific patient, a subcutaneous
facelift will not address underlying skeletal deformities, ptotic deep soft tissue structures or
change the skin texture. A mini-facelift technique will not address cervical area and malar
fat pad region predictably. An endoscopic face-lift option will not improve skin texture,
and will release periosteal connections with the superficial soft tissues enhancing just the
upper facial and midfacial areas. Best results are expected with use of a composite
rhytidectomy incorporating multiplanar dissections and SMAS procedures, which will give
better control of soft tissues and provide longer lasting results.

Reference:
Ghali GE, Evans TE. Rhytidectomy. Peterson's principles of Oral and Maxillofacial
Surgery. Second edition. 2004. Vol 2: 1371

Waite PD, Cuzalina LA. Rhytidectomy (face-lift). Fonseca Oral and Maxillofacial Surgery.
Cleft/ Craniofacial, Cosmetic Surgery. 2000. Vol 6:379

American Board of Oral and Maxillofacial Surgery 245


2008 Oral and Maxillofacial Surgery Self Assessment Tool (OMSSAT)

The procedure which trims and secures the posterior border of the superficial
musculoaponeurotic system (SMAS)-platysma muscle layer in a face-lift procedure is best
termed:

A. imbrication.

B. involution.

C. placation.

D. transposition.

Answer: A

Rationale:
With the current trend toward limited undermining in facelift surgery, identification and
effective use of the SMAS layer is pivotal for success and longevity of rhytidectomy. This
layer, initially described by Mitz and Peryonie in 1976, includes the superficial temporal
fascia superiorly, and extends into the superficial surface of the platysma muscle inferorly.
It lies external to the parotid capsule, and contains varying amounts of fat. After
development of skin and SMAS flaps, the SMAS layer can be manipulated by either
plication (suturing and folding without excision) or imbrication (excision and
repositioning) or a combination of both procedures.

Reference:
Chisholm BB: Surgical facial rhytidectomy. Oral Maxillofac Surg Clin North Am 12,
2000: 719-728

Ghali GE, Evans TE. Rhytidectomy. Peterson's principles of Oral and Maxillofacial
Surgery. Second edition. 2004. Vol 2: 1371

American Board of Oral and Maxillofacial Surgery 246


2008 Oral and Maxillofacial Surgery Self Assessment Tool (OMSSAT)

Relative to facelift surgery, the incidence of skin flap necrosis is found to be how much higher in
smokers when compared to non-smokers?

A. 5

B. 8

C. 12

D. 15

Answer: C

Rationale:
Some surgeons consider smoking as an absolute contraindication: however, most
recommend that smoking be stopped at least 2 6 weeks before surgery, and for at least 2
weeks after surgery. The overall complication rate for patients who continue to smoke is
more than twice when compared to patients who stop smoking prior to surgery. Bupropion
(Wellbutrin) is often used to help in the cessation of smoking. Nicotine patches and gum
may still pose the same hazards as smoking in terms of delivery of nicotine and its
vasoconstrictive effects.

Reference:
Waite PD, Cuzalina LA: Rhytidectomy (face lift). In: Fonseca R, Baker S, Wolford LM
(eds). Oral & Maxillofacial Surgery, Vol 6, WB Saunders, Philadelphia 2000. P368

August Meredith: Complications in oral and maxillofacial surgery. Oral and Maxillofacial
Surgery clinics of North America, May 2003 pages 278-280.

Schaner P. Evaluation of the esthetic surgery patient. Atlas of Oral and Maxillofacial
Surgery Clinics of North America. P121-30, 2004.

American Board of Oral and Maxillofacial Surgery 247


2008 Oral and Maxillofacial Surgery Self Assessment Tool (OMSSAT)

Macgregors patch refers to an area adjacent to the:

A. sternocleidomastoid and ear lobule, where important nerves are found.

B. zygomatic arch and prominence where a plexus of vessels is found.

C. antegonial notch and inferior mandible where the facial artery is found.

D. the preauricular area where the facial nerve crosses the zygomatic arch.

Answer: B

Rationale:
McGregor's patch is also known as the bloody gulch. It is named after the strong
zygomatico-dermal fibrous attachments that often present as skin dimpling or retraction. It
is also important because a plexus of vessels supplied by the facial artery and transverse
cervical artery becomes superficial in this area. Damage to these vessels can create
bleeding during development of the skin muscle flap in face-lift procedures. Additionally,
the buccal nerve lies just deep to this danger zone, and the zygomatic branch of the facial
nerve becomes more superficial in this area.

Reference:
Waite PD, Cuzalina LA: Rytidectomy (Face Lift). In: Fonseca R, Baker S, Wolford LM
(eds). Oral & Maxillofacial Surgery, Vol 6, WB Saunders, Philadelphia, 2000. P 366-7

Goldwyn RM. Late bleeding after rhytidectomy from injury to the superficial temporal
vessels. Plastic Reconst Surg 88(3): 443-445, 19
.

American Board of Oral and Maxillofacial Surgery 248


2008 Oral and Maxillofacial Surgery Self Assessment Tool (OMSSAT)

What is the minimum safe osteotomy distance inferior to the mental foramen (in millimeters)
during osseous genioplasty surgery?

A. 1

B. 2

C. 3

D. 4

Answer: D

Rationale:
The mental foramen is an important landmark during the design of anterior mandibular
horizontal osteotomy (genioplasty). The design should ideally extend posterior to the
mental foramen, in order to improve cosmetic results, increase bone contact surface area,
and increase stability of the osteotomized segment. The inferior alveolar nerve courses in
the mandibular canal and exits through the mental foramen. Studies have demonstrated that
the nerve often loops within a 4-6 mm zone anterior and inferior to the foramen in its canal
prior to exiting the bone through the foramen. Thus, the osteotomy cuts should be placed at
least 4mm, and preferably 6 mm away from the mental foramen in order to avoid the nerve
and its anterior and inferior looping before exits from the mandibular bone.

Reference:
Bloomquist DS, Lee JJ. Principles of mandibular orthognathic surgery. Peterson's
principles of Oral and Maxillofacial Surgery. Second edition. 2004. Vol 2: 1168-1173

Nishioka GJ, Mason M, Van Sickels JE. Neurosensory disturbance associated with the
anterior mandibular horizontal osteotomy. JOMS 1988: 46: 107

Hwang K, Lee WJ, Song YB, et al: Vulnerability of the inferior alveolar & mental nerve
during genioplasty: An anatomic study. J Craniofac Surg 2005: 16: 10

American Board of Oral and Maxillofacial Surgery 249


2008 Oral and Maxillofacial Surgery Self Assessment Tool (OMSSAT)

The minimum pore size for porous facial implants which allows for host resistance to bacterial
infection is:

A. < 1 micron.

B. 10 to 25 microns.

C. 30 to 50 microns.

D. >50 microns.

Answer: D

Rationale:
Porous implants have the potential for ingrowth of bacteria that can be introduced at the
time of surgery or post-operatively, due to tissue breakdown. This occurs when the pore
size is > 1 micron. Human host defenses including macrophages require a pore size of > 50
microns to enter and engulf bacteria that have infected the implant. Therefore, the ideal
porous implant would have pores smaller than 1 micron to avoid bacterial innoculation or
>50 microns to allow macrophages to engulf the bacteria.

Reference:
Cohen MS, Constantino PO, Friedman CD. Biology of implants used in head and neck
surgery. Fac Plast Surg Clin North Am. 1999; 9: 17

Eppley BL. Alloplastic implantation. Plast Reconstr Surg. 1999; 104:1671.

American Board of Oral and Maxillofacial Surgery 250


2008 Oral and Maxillofacial Surgery Self Assessment Tool (OMSSAT)

Which cephalometric landmark shows bone apposition after an advancement sliding horizontal
osseous genioplasty?

A. Pogonion (Po)

B. Gnathion

C. B point (B)

D. Menton (Me)

Answer: C

Rationale:
Multiple investigators have demonstrated that minimal soft tissue dissection gave a more
predictable hard and soft tissue response because of less bone resorption within the
advanced segment. With such minimal soft tissue dissection, no bony remodeling of
gnathion or menton was observed. Bony apposition occurs at B point and pogonion shows
osseous resorption.

Reference:
Betts NJ, Edwards SP. Soft tissue changes associated with orthognathic surgery. Peterson's
principles of Oral and Maxillofacial Surgery. Second edition. 2004. Vol 2: 1229-1233

Van Sickels JE, Smith CV, Tiner BD, Jones DL: Hard and soft tissue predictability with
advancement genioplasties. Oral Surg Oral Med Oral Pathol 1994 Mar; 77(3): 218-21

Polido WD, Bell WH. Long-term osseous and soft tissue changes after chin advancements.
J. Craniomaxillofac Surg. 1993, 21: 54-9

Which of the following statements regarding esthetic evaluation of the midface is true?

American Board of Oral and Maxillofacial Surgery 251


2008 Oral and Maxillofacial Surgery Self Assessment Tool (OMSSAT)

A. The zygomatic prominence should be located 2 cm inferior and 1.5- 2 cm lateral to the
lateral canthus of the eye.

B. The infraorbital rim should be 0 to 2 mm posterior to the cornea

C. Greater than 3-4 mm of sclera should be visible inferiorly between the limbus and the
lower eyelid.

D. The cheek prominence should be located 3 mm superior to the Frankfort horizontal plane.

Answer: A

Rationale:
The midface region is best evaluated in four basic views frontal in repose and smiling,
profile, three-quarter oblique and basal. Normal bizygomatic width should be 88.5 % +/-
4.6 % of the facial height (as measured from nasion to gnathion) in millimeters. The
zygomatic prominence is located 2cm inferior and 1.5 - 2 cm lateral to the lateral
canthus. The infraorbital rim should be 0-2mm anterior to the cornea. Normal scleral show
is less than 4mm. Cephalometric evaluation of the zygomatic region should include the use
of McNamara's nasion perpendicular, and the distance of the infraorbital rim to this
perpendicular line is approximately 5 mm. The zygomatic (cheek) prominence should be
located below the Frankfort Horizontal Plane, and not above it.

Reference:
Zide and Epker. Systematic Aesthetic Evaluation of the Cheeks for Cosmetic Surgery.
Cosmetic Oral & Maxillofacial Surgery, Oral & Maxillofacial Surgery Clinics of North
America. Vol. 2, No. 2, May 1990.

Eppley BL. Alloplastic implantation. Plast Reconstr Surg. 1999; 104:1671.

American Board of Oral and Maxillofacial Surgery 252


2008 Oral and Maxillofacial Surgery Self Assessment Tool (OMSSAT)

The most common etiology for hair loss in males is:

A. alopecia areata.

B. androgenetic alopecia.

C. traction alopecia.

D. psychological stress.

Answer: B

Rationale:
Androgentic alopecia is the most common condition causing male pattern baldness. It is
estimated that it affects approximately 60-80% of men, and a hormone
(Dihydrotestosterone) is associated with this cause of male baldness. Genetic predisposition
contributes to the number of testesterone receptors of follicular cells and activity of 5-alpha
reductase activity in different areas of the scalp. This enzyme reduces testesterone synthesis
and protein synthesis, thereby producing finer and finer hair, until hair is lost. Alopecia
areata is a relatively rare condition where round patches of hair loss appear suddenly. The
hair-growing tissue is attacked by the patient's own immune cells for unknown reasons.
Traction alopecia is associated with sustained tension on the scalp hair, and traction causes
hair to loosen from its follicular roots. Hair loss also occurs secondary to local follicular
inflammation and atrophy. Patients undergoing severe physiologic (e.g., with acute and
chronic systemic illness) changes or significant psychological stress can suffer from some
degree of hair loss, which is usually temporary.

Reference:
Cummings: Otolaryngology: Head & Neck Surgery, 4th ed. Page 673-680

Unger WP, Nordstrom REA: Hair transplantation. Marcel Dekker, New York 1998.

Hendler BH: Hair restoration surgery. Esthetic surgery of the aging face. Oral and
Maxllofacial Surgery Clinics of North America. Vol 6(8), 1998. P 40-1

American Board of Oral and Maxillofacial Surgery 253

Potrebbero piacerti anche